Links 7/16: Peter Linklage

Newspapers showed the picture of a man who died in the terrorist attacks on Paris last year. But somebody who is clearly the same man was shown as being killed in the Orlando nightclub shooting last month. And now the same person is in the body count for the attacks on Ataturk Airport in Istanbul. Who is the mysterious man who dies in every terrorist attack?

Systematic review: “Overall, results of these studies do not indicate a higher prevalence of eating disorders among fashion models compared to non-models.”

/r/AccidentalRenaissance: everyday photos which, when you think about it, look kind of like Renaissance paintings.

Pseudoerasmus reviews Empire of Cotton. Even though he’s not a fan of the book, just his hostile summary helped me understand some of what people mean when they say that “free trade” has set back the developing world.

In order to counteract my (and maybe your) usual bias: here’s somebody fired for doing a study that found that some people were racist.

Oh God oh God oh God functional brain imaging studies are awful – “If the whole-brain across-subject correlation analysis with 16 subjects considers 1000 possible correlations (considerably less than the number of voxels in a whole-brain analysis) the peak correlation coefficient is expected to be about 0.75, even if the true correlation is actually 0.” Best read alongside the old study that replicated various results about the brain in a dead salmon to show how easy it was to fake.

Supposedly most antidepressants don’t work in kids and teens, but Prozac does. But I find anything that discovers striking cross-SSRI differences a little hard to believe.

Psssst, wanna buy a slightly used Soviet surplus tank? What if I told you they cost less than a nice car?

The Kentucky meat shower was an unexplained event when meat fell from the sky like rain. “A letter from Dr. Allan McLane Hamilton appearing in the publication Medical Record [stated] that the meat had been identified as lung tissue from either a horse or a human infant, ‘the structure of the organ in these two cases being almost identical.'” Well that’s not creepy at all.

LWer and Future of Humanity Institute scholar Stuart Armstrong is in the news for a paper written together with Google AI scientists detailing an exciting new avenue for working on AI safety based on designing intelligences that will not resist their own shutdown. Related: scientists at Google, OpenAI, Stanford, and Berkeley publish a review of Concrete Problems In AI Safety.

China plans to evict 5000 monks from Larung Gar Buddhist Monastery for political reasons. If you’re wondering what kind of monastery has 5000 monks, take a look at the photos.

Related: Treasure-hunting is big part of Tibetan Buddhism, and monks inspired by mystical revelation will often go out and unearth treasures or manuscripts hidden by past saints.

Review: “Nominal agreement between initial studies and meta-analyses regarding the presence of a significant effect was not better than chance in psychiatry, whereas it was somewhat better in neurology and somatic diseases.” If I’m understanding this right, it means that an initial study about something in psychiatry conveys literally zero evidence about whether that thing is true or not.

China plans to cut meat consumption by 50%.

Everyone knows that “millennials” are far left, but the truth is more complicated – really into gays, marijuana, and immigration, but not much different than older generations on support for the poor or on racial issues (wait, really?)

Snopes: despite media reports, there is no evidence that the Orlando nightclub shooter was gay. This is so confusing to me that I worry it’s some kind of prank, but how could I even check?

David Chapman on Brexit. This probably has something to offend everybody.

A list of 308 online effective altruism-related resources. Some of the Facebook groups seem kind of Potemkin-y, though.

Relevant to my interests: there was once an unrecognized US state called Scott.

Economists are very pessimistic about (one version of) universal basic income.

Related: an alternative to universal basic income is the universal basic share, where the government says something like “We pledge to forever redistribute 10% of GDP, whatever that may be, among our citizens as a universal basic income”. The hope is that even if this starts out as not enough, as the economy grows it will gradually become more and more until it’s enough for people to live on. But I worry that ignores the effect discussed here, where if the government had tried that in 1900 then by now the income would have grown to the amount the poor needed to support themselves in 1900, yet would still be way below the amount what we consider a minimum standard of living today.

Company that handles tech company interviews makes a feature that changes what gender an interviewee’s voice sounds like, to see if women get more tech jobs when the company thinks that they’re men. To the surprise of nobody who is paying attention, there is no anti-woman bias found and in fact women do slightly better when they are known to be female.

If you miss the predictions of health risks and so on that you used to be able to get from 23andMe, you can get them free from Genotation now – just upload your 23andMe data to their site and it will do the calculations for you. I’m slightly confused that its ancestry panel seems to think I’m East African, but I guess in a sort of cosmic long-term sense it’s not wrong.

Wait List Zero is a group that encourages altruistic kidney donation, eg donation to a person you may not know who really needs a kidney.

Two new studies conclusively determine that the apparent “obesity paradox” – the finding that sometimes overweight people had lower death rates than normal weight people – was an error and that in fact being normal weight is healthier.

Last week: Kentucky legalizes hair braiding without a license. This week: fiery storms scour the land; the living envy the dead.

A few months ago I argued that open-source AI would be a bad thing because it would sabotage safety efforts. Now Nick Bostrom investigates the same question much more rigorously.

D.R. Hagen on why the 11th, the 2nd, and the 3rd of each month are mentioned in books less often than other days.

Psychiatrists often use drugs that modulate norepinephrine, like Effexor and Strattera, on the assumption that this chemical plays an important role in psychiatric disease. But some people have a rare disease that causes them to have literally no norepinephrine at all yet seem to be psychiatrically normal. I have no idea how this can be true.

Intermittent fasting is no better than just dieting the normal way. I hate to gloat, but this concludes an almost ten-year argument I’ve been having with an acquaintance who said that the failure of doctors to immediately endorse intermittent fasting proves that the medical profession are all quacks who don’t care about their patients.

Ross Douthat: The Myth Of Cosmopolitanism. “[We give] the elite side of the debate (the side that does most of the describing) too much credit for being truly cosmopolitan. Genuine cosmopolitanism is a rare thing. It requires comfort with real difference, with forms of life that are truly exotic relative to one’s own….The people who consider themselves “cosmopolitan” in today’s West, by contrast, are part of a meritocratic order that transforms difference into similarity, by plucking the best and brightest from everywhere and homogenizing them into the peculiar species that we call “global citizens”…There is more genuine cosmopolitanism in Rudyard Kipling and T. E. Lawrence and Richard Francis Burton than in a hundred Davos sessions.”

The campaign for rigor in UFO hunting.

Tell me I’m misunderstanding this, or else it’s the most confusing thing I’ve read all month: study shows that sugar only makes you gain weight insofar as it tastes good, and mice who have been genetically engineered not to like the taste of sugar fail to gain much weight on sugar even when they eat exactly as much of it as the mice who like it. Possible implications for artificial sweeteners?

Refugee children who arrive to the US at a very early age like 6 months don’t have substantially better outcomes than those who arrive at a later age like 6 years. This is very strange, because we expect them to be living in a terrible deprived environment before immigration but a much better one afterwards. How do we reconcile this with the “childhood stresses of poverty” theory of poor people’s problems like in that study about the Cherokee reservation?

Also, how do we reconcile behavioral genetics with attachment theory?

All of those studies showing that a picture of eyes watching you would make you behave in a more prosocial way are the latest victims of the replication crisis.

The first fully automated fast food restaurant comes to San Francisco.

Brian Tomasik has a really good article on gains from trade that asks the important question – why is there ever conflict? Why don’t people just Aumann-agree on how the conflict would probably go, and skip the part where they actually waste all of their resources fighting each other? See also this SSC post.

Great moments in Donald Trump tweeting.

This entry was posted in Uncategorized and tagged . Bookmark the permalink.

814 Responses to Links 7/16: Peter Linklage

    • E. Harding says:

      No; this is not a good article. This:

      “In the cold light of morning, I wonder whether dismayed Leave voters will fear that they have cut off their nose to spite their face or whether they were misled by lying Leave politicians.”

      is especially stupid.

      The thing that the Remain side never really bothered to explain to people like me what’s so good about the E.U. I live in the U.S. and I’m a U.S. citizen; I didn’t know whether Britain should Leave or stay until a few weeks before the vote, though I always sympathized with the idea of Leave. In the weeks before the referendum, it became clear that almost every supporter of Remain was a hack or a leftist or some financier. Since when did leftists start to ally with financiers? If they start to do this, this is never good (e.g., the New Deal era), with few exceptions (e.g., Bryan v. McKinley, in which McKinley was in the right). Supporters of Leave, meanwhile, seemed to be champions of British freedom and independence, as well as solid men of the Right and true nationalists. Which side to pick, given these constituencies, was obvious, even if I had no real idea of the costs and benefits. I also looked at Switzerland, Norway, and Canada, the closest comparable countries, and decided they didn’t look too bad. I also remembered Cameron was in favor of Turkey’s ascent into the E.U., which would just have been a disaster in every respect. I recalled how the fairly mild Federal government of the Age of Jackson was turned into a monstrosity by TR, Wilson, and FDR. I didn’t want that to happen to Britain, especially by unelected and unaccountable officials. Though I was never a British citizen, consequently, during the last few days of the campaigning, my thoughts were solidly pro-Leave. Given the absolutely hysterical reaction from the White Democrat press in days after the vote, I’m convinced Leave was the right decision. One does not want Britain to be subject to allied Brahmin-German-Turkish imperialism!

      Moldbug’s old posts seem especially relevant here, especially on the subject of Brahmin thoughts on White (Rhodesia, the Confederate States of America) v. non-White (Zimbabwe, Angola) anti-imperialism.

      • It’s still a good article, though maybe not perfect. Also the author, Alastair Roberts, is not a leftist by nearly any definition of the word, unless you consider the fact that he’s an academic to be ipso facto proof of leftism.

        I mean, I don’t actually agree with his object-level position here, but he does a really good job of laying out the moral and ethical case for nationalism, as the political manifestation of “filial love,” in contrast with the “universal benevolence” which is the moral impulse of progressive internationalism. I’m on the side of filial love, and I appreciated his ability to lay out the vision.

        • E. Harding says:

          “is not a leftist by nearly any definition of the word”

          -I suspect hackery is at work here. I definitely consider David Cameron a hack, even though he’s not left-wing.

          Also, more stupidity:

          “The EU is the epitome of a neoliberal political entity, committed to free markets,”

          -Demonstrably untrue. The E.U. is committed to standardized, managed, controlled, and protected markets. It definitely sounds like a Whig idea in the Henry Clay sense.

          “This is rule by the market, where the interests of the market, and those whose power it extends, triumphs over all else.”

          -If so, why did left-wingers generally oppose Brexit? Left-wingers generally hate the market. Why would they support a pro-market institution? Something’s off here.

          “It is often presumed that persons are essentially blank slates, that old cultures can be shed and new cultures assumed with ease, and that persons and their communities are radically plastic and adaptable to new situations. The reality is not nearly so simple.”

          “For instance, immigrants from any of the countries which are largely constituted by the diaspora of the British peoples—the USA, Canada, Australia, New Zealand—typically have a far more natural affinity with and instinct for our country and culture than persons from most other countries, and not merely on account of shared ethnicity and language. Britain’s less committed relationship to Europe has arisen in no small measure from the strength of its bonds with such countries in the Anglosphere.”

          “The opening up of borders has a profound ethical symbolism for such persons (although it is worth bearing in mind that there are ‘borders’ that the cosmopolitan class often depends upon—house prices, degrees and credentials, school catchment areas or fees, etc.—and that they may be considerably more concerned about the possible removal of these). No human being should be regarded as any more my neighbour than any other.”

          -These parts aren’t stupid, so maybe this is a real part of the reason for Brexit voting patterns?

          • Tatu Ahponen says:

            Again, it helps if you separate the liberal left and the socialist left from each other. It was the liberal left, backed by liberal right, that was the most enthusiastic about the EU: the socialist left was rather lukewarm, and parts of it advocated for a Brexit.

          • The original Mr. X says:

            -If so, why did left-wingers generally oppose Brexit? Left-wingers generally hate the market. Why would they support a pro-market institution? Something’s off here.

            Because those icky right-wing nationalistic proles supported Brexit, and they didn’t want to be like them.

          • I’m bored with that cliche already.

            Another factor is that supporting the EU gives you centre-left policies even when there is a conservative government.

          • E. Harding says:

            @Tatu

            Freddie DeBoer was saying in his Twitter account that no left-winger he knew was favoring Brexit.

          • Luke the CIA stooge says:

            My reading of the lefts support for the EU was that they were pretty naked voting their class interests. Academics, journalists, bureaucrats and politicians all lose access to the massive job market that is the Brussels paperwork factory, and they are now subject to the possibility of genuine market forces in their industries. For the first time a conservative government can now lay off the paper pushers and propagandists at the various state organs without them being able to jump to another institution where they can get revenge and modify the discourse to punish them.

            The Mandirins and inteligensia voted to protect their cartels.

            One cannot understand modern politics without understanding that the same people who call for more regulation and vehemently assert their right to meddle in your business, are the same people who have a constitutional amendment saying THEIR INDUSTRY: the ideas industry can never be regulated. The same industry responsible for wars of the twentieth century and more deaths than all other industry combined.

            Left wing politics at this point is just war waged by the intelligensia against its rival power bases.

            the only solution I can see would be a constitutional amendment granting the same protection from regulation the press/academia enjoys to all industries/people.

            (Note: there where genuine and legitimate arguments for remain just as there were for leave, it’s the way the chattering class all fell into line that I’m thinking can only be explained through class interest. We have the same in Canada when it comes to the CBC: no journalist or academic will bad mouth it(or go hard on a party that supports it) because they all want a job there)

          • LCL says:

            it’s the way the chattering class all fell into line that I’m thinking can only be explained through class interest

            That’s like a group selection theory of media. It doesn’t work because as more chatterers all say the same thing, there start to be huge individual incentives for someone to say the opposite. Being the leading contrarian on a hot topic would draw massively career-enhancing levels of attention, citations, and reaction. The individual interest should dominate the class interest at some point and break the consensus long before everyone has fallen in line.

          • Luke the CIA stooge says:

            LCL

            In my experience, at least with the media, that competitive force doesn’t work out. Since the broad class gets to decide what the Overton window will be, it can coordinate punishment of defection through the informal cultural forces of the Class.

            Continuing with the Canadian example there are numerous talk radio and blogosphere personalities who produce strong content, have unique perspectives and strong followings. None of them will be invited onto the CBC or other National news programs if they are right wing. Even though they could improve their program and attract a broader audience, they can’t invite them on, as their mainstay followers and their peer group has collectively determined that those HIGHLY INFLUENCIAL POLITICAL ACTORS are loons and the broader chattering class would punish them for giving those people hacks a platform.

            There were a lot of public intellectuals, bloggers and politicians who supported Leave. Almost none of these have or will ever hold positions in academia, the public service or traditional media as a result of their support. They have to rough it out in new media, think tanks and elected office.

          • TheAncientGeek says:

            There is a working class left as well.

            And it is quite possible for one class’s interests and everybody’s interests to coincide. Full strength anti-immigration little-Englanderism isn’t a viable economic model, because if you don’t allow other people nationals in, you don;’t get the trade deals.

            And the right seem to do pretty well on media that are required to be balanced. Melanie Phillips is always popping up on the BBC despite her insanity.

            And Overton windows do shift, and shift a lot.

      • Montfort says:

        it became clear that almost every supporter of Remain was a hack or a leftist or some financier… Supporters of Leave, meanwhile, seemed to be champions of British freedom and independence, as well as solid men of the Right and true nationalists

        I suggest you probably knew which side you preferred before you made these observations.

        • E. Harding says:

          I said “I always sympathized with the idea of Leave”. But the original opposition to the Common Market was very different. It was disproportionately left-wing and Scottish. Not sure what’s changed since then, but the general left-wing reaction makes me suspect the E.U. has become a left-wing imperialist project, probably for nefarious purposes. Again, I don’t know all that much about the E.U. and its operations. I watched Brexit: The Movie, which made a good, but by no means rock-solid case for Leave. I’m in favor of free trade and a supporter of neither fully open nor fully closed borders (though my gut says fully open’s better than fully closed, provided the migrants aren’t treated as they are in Germany, but, rather, as they are in Russia or maybe China and perhaps Qatar, but fully closed borders have their advantages, I guess).

          • Montfort says:

            I suppose what I really mean to say is that this is how self-reinforcing opinions feel (to me) from the inside. One starts with an intuitive preference for one side or the other, and that makes the proponents of the favored side look better, which by association makes the favored side look more attractive and more “correct”, etc.

            Which is not to say that you’re wrong, necessarily. It’s just the quoted section resembles a pattern of thought that can make you overconfident in many areas – as it did for me for a long time (and probably still does).

            As for the UK left’s opinion of EU membership, the Telegraph claims it’s mostly about apathy and worker’s rights, but I wouldn’t say I’m an expert in British politics so there may be other things at work.

          • E. Harding says:

            “and worker’s rights”

            -That’s at least a reason and one that’s actually plausible, but I haven’t heard it mentioned before you brought it up, after reading what must be hundreds of Remainer tweets.

          • rel8 says:

            >after reading what must be hundreds of Remainer tweets.

            Reading tweets gave you no insight into the Remain position?

            I’m shocked, absolutely shocked!

          • Tatu Ahponen says:

            The official Remain campaign featured the worker’s rights argument to some degree – you can read still read it from Britain Stronger In Europe campaign facebook site, for example.

          • Aapje says:

            makes me suspect the E.U. has become a left-wing imperialist project

            Ironically, I’m actually against the EU because it is an anti-worker, neoliberal project, which seeks to destroy worker rights for people with weak bargaining positions and which seeks to destroy the uniqueness of cultures. Once upon a time, Stalin sought to destroy the social cohesion of cultures by forcibly moving people in the USSR, today the EU encourages migration to the same effect. The goal is to manufacture a common European identity of people who somehow simultaneously care about their common man (see attempts by the EU to force solidarity with poorer EU countries & refugees) and treats them as economic units to be exploited as much as possible (see EU laws which force organisations to contract with the cheapest bidder and force countries to open markets as much as possible, so people will no longer be protected from the downsides of free markets).

            Technically I’m actually also against the EU because it is ‘a left-wing imperialist project,’ but the modern left-wing elite in the West is mostly elitist, anti-socialist and in favor of laissez faire. So despite being very pro-redistribution, I am very much against most of the modern left elite (especially as the redistribution that they do tends to go from the poor to the rich).

            The problem with ‘imperialist,’ or as I would call it ‘multi-cultural’ governments, is that there is no solid common ground between the governed, so democracy cannot work. ‘The people’ cannot come together and agree on a course, because the subcultures are too different. A French person is not going to be able to find enough common ground with a Brit and a conservative Muslim can’t find enough common ground with a progressive atheist. So there are only two options:
            – an ineffective stalemate
            – hijacking of the government by an elite who push their own agenda, which disagrees with a majority

            As problems still need to be solved and people will get fed up with a stalemate, it is inevitable to get option 2 and that is what we have in the EU. Of course, it could be worse, the Eurocrats are not communists or nazi’s, but they still exhibit extreme group think and fairly overt contempt for even the most well-reasoned dissent. Due to the lack of dissent and a culture of extreme risk taking with the lives of others, they choose solutions that obviously will fail and don’t hedge against that failure. So they cannot help but create huge destruction (and various cases have already happened, like Greece and the ongoing destruction by the Euro). Right now the Euro prevents low-productivity countries from devaluing their currency to keep their products worth buying, while it keeps the (relative) wages artificially low in high-productivity countries. The result is some countries have huge unemployment and stagnation, while in other countries, most of the productivity surplus ends up in the pockets of investors, rather than workers. They then park this offshore, depressing the EU economy, rather than spend it like workers would.

            As a EU citizen, I just hope that this destruction won’t go on too long before people regain their senses and abandon this farce & the elite will allow this to happen in a democratic way. If not, it’s inevitable that oppressed Europeans will resort to violence. A seed of that is already starting to show (sadly enough, mostly aimed at Muslims, who, while a good symbol of the contempt by the elite for the interest of the native poor, are not the architects, but are also pawns).

            Anyway, the irony of the Brexit is that the mainstream in Britain is actually neoliberal/third way and EU outcomes have been closer to the British agenda that to that of any other country. So they’ll just end up with mostly the same politics anyway.

          • Anonymous says:

            The goal is to manufacture a common European identity

            And it would be pretty horrific if they succeeded.

          • erenold says:

            Sp*ndrell is head and shoulders one of the most interesting writers of the New Right for me. I speak Mandarin as a first language and grew up immersed in Chinese culture, and I can attest – he’s not just some poser who majored in Asian Studies or something, he’s the real deal. And he’s got a wonderful and hilarious gift for using Ye Olde Chinese Wisdom to make his points for him. If his claim to read classics of my culture in the original language is true, that would be seriously impressive – consider someone reading Beowulf in Anglo-Saxon, for instance.

            I mean, I then proceed to disagree with just about everything on his website, but I still read it pretty religiously. Just about the only person I can think of for whom that’s true.

            I see he used to hang around here before he stupidly felt the need to wind up our host. Shame.

          • suntzuanime says:

            His Chinese history parables are great, though perhaps more for the Chinese history than the parables. When he talks directly of the present day, he’s nothing special. He has some unique points that are worth making, but his argument structure is only semicoherent and he orbits his points erratically rather than spiraling inexorably towards them. And like most altrighters, he can’t resist throwing the occasional horrifying shibboleth into the middle of an otherwise inoffensive paragraph. He’ll be talking about the origins of the EU and then go off on an aside about how it’s anarchotyranny that women are allowed to nag their husbands but men aren’t allowed to beat their wives.

            Say what you want about Moldbug, but however meandering his writing style was it was still all directed to a definite point, and when he had something to say that would shock my liberal sensibilities, he would use apologetic indirection instead of flaunting his edginess.

          • erenold says:

            Heh – interesting, because that’s almost exactly how I feel about our mutual friend. Didn’t trust myself to be able to objectively evaluate someone with whom I shared so few priors, though.

      • erenold says:

        The thing that the Remain side never really bothered to explain to people like me what’s so good about the E.U. I live in the U.S. and I’m a U.S. citizen

        Uh… why should Remain have to convince you of anything?

        leftists… champions of British freedom and independence, as well as solid men of the Right and true nationalists… unelected and unaccountable officials… absolutely hysterical reaction from the White Democrat press… allied Brahmin-German-Turkish… Brahmin thoughts on White (Rhodesia, the Confederate States of America) v. non-White (Zimbabwe, Angola) anti-imperialism.

        And what could they possibly have said that would have convinced you of anything at all?

        • E. Harding says:

          “Uh… why should Remain have to convince you of anything?”

          -This is why Remain lost. 🙂

          • John Colanduoni says:

            Because they failed to convince US citizens? Should they have paid for commercials in Russia too?

          • erenold says:

            I really can’t tell if you’re trolling me right now. You think Remain lost because they failed to convince the American triple-parenthesis-using far-right constituency? Whose primary motivation was that Brexiteers were his tribal allies and Remainers his tribal enemies?

            Come now, sir.

          • E. Harding says:

            Nobody watches commercials anymore, John (or at least, I don’t). Everybody with sense has NoScript and Adblock Plus installed and Adobe Flash uninstalled (unless working with outdated website features).

            Come on; the UK and the U.S. are both united by a common language. We can both read each other’s writings.

            Notice the smiley after my response to you. And I still think your comment massively misses the point.

            “because they failed to convince the American triple-parenthesis-using far-right constituency? Whose primary motivation was that Brexiteers were his tribal allies and Remainers his tribal enemies?”

            -Well, when you put it that way, that does sound ridiculous. Then again, I think of myself as fairly objective and failed to find Brexit: The Movie decisive in its arguments. I also thought this stuff about cheaper goods and E.U. funding from the Remain camp was a total crock. All I wanted to know was if the E.U.’s benefits outweighed its costs. I wasn’t all that thorough in my research, so my conclusion here was indecisive and had to rely on fallbacks: tribal allegiances, unelected Brussels officials, the plausible Future of the E.U. over the next 40-50 years, and Turkey.

          • hlynkacg says:

            I really can’t tell if you’re trolling me right now. You think Remain lost because they failed to convince the American triple-parenthesis-using far-right constituency?

            No they lost because they failed to convince anyone who was not already predisposed to agree with them, while at the same time insulting those who were “on the fence”.

          • John Colanduoni says:

            @hlynkacg

            I stand with the remain camp as to the decision but I totally agree with that assessment of their methods. This is another case where it was proven that yelling “you so racist” at people who have the gall to just maybe disagree with you is not effective at winning hearts and minds (cf. US presidential election 2016).

          • erenold says:

            @ E. Harding,

            The point is not that you cannot speak English. The point is that Remain’s strongest cards were those of pocketbook economic harm; Nissan plantworkers in Sunderland, EU grant recipients in Cornwall and Wales, these people were the targets of this pocketbook campaign. Others, who work with or in foreign countries. You are, with respect, by definition entirely immune to these concerns by virtue of your passport.

            So, obviously you think they played their cards foolishly and ineffectively. Maybe they even did. But if they had done a better job of forcefully and effectively pointing out the potential economic drawbacks, can you honestly say you would be able to tell?

            @hlynkacg

            Actually, they lost because they lost Old Labour: Wales and the industrial North, which either voted out or stayed at home. All other areas of the UK mapped perfectly to the results of GE 2015 – Scotland and London left-wing, Home Counties and the South East right-wing. And I can think of very little that would have worsened the situation with Old Labour than pandering to the alt-right.

          • hlynkacg says:

            I’m not talking about the “alt right”.

          • erenold says:

            But E. Harding and I are?

            “the Remain side never really bothered to explain to people like me

          • hlynkacg says:

            That the Remain side never really bothered to explain to anyone who didn’t already agree with them is why they lost.

          • The original Mr. X says:

            I’m British, pale green anon, and I think hlynkacg’s analysis of the Remain campaign is more-or-less spot on.

          • Lyyce says:

            I live in Britain and do agree with @The original Mr. X.

            Where do you think WE got it from?

          • Anonymous says:

            The complaint was about faulty epistemology. It doesn’t seem like you two have any answer for that. To quote my fellow anon:

            “pre-existing enemies of the jaywalker” is a good name for SSC’s conservative comment convoy when they perform outrage at stuff they dont care about, and knowingness about things they can’t know.

            These people can’t change their minds and they cant change the subject.

            Everything comes back to blaming liberals while protecting conservatives from any blame whatsoever.

            So, Scott. Whither unbiased rigor?

            If your commentariat was a patient would you put up with the lack of self-circumspection? The obsessive blaming? The grandiosity and paranoia?

            While you self-deprecate and show doubt, the default tone of your fans is comically arrogant. The epistemic hygiene around your blog is a joke, and it appears that works for you somehow. It’s a tell. It really does give away the ghost vis a vis you and the irrational contempt you feel for liberals but have attempted to publicly repress.

          • Aapje says:

            The reason why Remain got a lot of support among the elite, but failed with the lower and lower-middle classes, is because they used arguments that matched the experiences of the elite, but not of the lower classes.

            For example, one key argument was “the EU is an engine for prosperity”

            In the eyes of the elite this is true, because they can now hire cheap Eastern European builders, nannies, etc or British natives whose wages have been driven down by the competition. Their goods and services are cheaper because they are produced or provided by Eastern European workers, farm hands, servers, etc.

            These elites tend to have jobs with high costs of entry, so only a few isolated Eastern Europeans manage to run the gauntlet and compete for these jobs. So few that there is no meaningful downward pressure on salaries or job opportunities, so the elite feel very good about being ‘tolerant’ of these few workers, who are the most integrated and least offensive of their countrymen.

            Meanwhile, the lower and lower-middle classes see a huge influx of workers for their jobs. Most Eastern-Europeans work way below their level of education, so effectively, British people are forced to compete with people that are simply out of their league. There is also widespread cheating, as a low British salary has way more buying power in Eastern Europe. So the Eastern Europeans are willing to work some years or seasons of the year for a illegal salary that you cannot live on reasonably in Britain and/or work in illegal abusive circumstances. Ironically, the willingness by Eastern Europeans to endure all this has resulted in a bias by employers against British natives, so they face discrimination like a job requirement to speak Polish.

            I could go on and on how the upper classes see certain benefits that the lower classes have far less benefit from and downsides that the lower classes experience way more than the upper classes.

            TL;DR: the upper class gets most of the benefits of EU policy and less of the downsides, while the lower classes get less of the benefits and most of the downsides.

            The British lower classes who live in impoverishing places simply can’t accept the narrative that the EU brings wealth, when in their experience, there is a causal link between more EU integration and worse conditions for them.

            The irony is that the anti-EU people get accused of lacking a heart for others, while it is the pro-EU people who actually can’t see beyond their class-interests. Every bonus that the City bankers gave themselves and every self-stroking comment they made how their earned that bonus, rather than achieved prosperity over the backs of others, resulted in more more anger among the lower classes and a feeling that their narratives are lies meant to convince the lower classes to accept oppressive laws & policies.

          • Gbdub says:

            Aapje, I’m curious what your thoughts are on American immigration policy, because it seems like I could swap “Mexican” for “Eastern European” in your post and end up with a halfway decent argument in favor of Trump’s border wall and/or Bernie’s campaign against NAFTA.

            (FWIW I agree with your assessment, insofar as Remain failing to make a case to the working class voters in the North)

          • Aapje says:

            My newer comments got disappeared :/

            Anyway, the EU and the US have very similar policies for work migration in that they try to prevent illegal crossings and only let in outsiders with special skills. So I think that Trump’s wall is just a technocratic attempt to make the current policy work better, not a fundamental change. The ‘other side’ like Clinton don’t seem to want actual open borders, so the posturing seems like virtue signalling to me.

      • John Colanduoni says:

        it became clear that almost every supporter of Remain was a hack or a leftist or some financier. Since when did leftists start to ally with financiers?

        I feel like a complex, nefarious explanation of this is unnecessary. A sufficiently strong part of the agenda of the financiers is simple and hard to contest: they enjoy the single market. The leftists have a variety of non-hidden motivations, such as being pro-immigrant (from other EU countries at least).

        That’s not to say that this means there are no unsavory reasons that these groups supported remaining in the EU. But if the only reasoning behind your position is the constitution of the supporters, Occam’s razor demands more.

        My biggest fault with the Leave side is that I feel they committed the error of doing something because it sounded like it would move towards their terminal values, not because it actually would (a traditionally leftist sin at that). The most outspoken of the Leave side have backtracked on the outcomes they pointed at when campaigning: Nigel Farage noted that the claim that leaving the EU would save the UK £350 that could be put towards the NHS was a ‘mistake’ (which was written on a bus used by the campaign), and Boris Johnson said there will likely be a maintenance of the status quo on immigration.

        So what did Britain get out of this? Freedom from pillowcase regulation? That would be significant (since obviously the EU has regulation for things other than pillowcases) except that a lot of the EU’s regulations for products and services already apply even to US companies that wish to sell to that market. So unless UK companies are producing goods only for UK consumption and the UK takes the time to modify all of these regulations, nothing will change there either. The main difference is now the UK will have virtually no input on the form of these regulations. So much for independence.

        Now that the arguments for leaving that were at the forefront of the debate have been dismantled by those who were yelling them very recently, I am honestly curious what tangible benefits those that are pro-Leave see for the UK.

        • E. Harding says:

          If the E.U.’s so good, should Canada join it? Should the U.S.? Again, Norway, Canada, and Switzerland seem to do fine without the E.U., just like Ireland seems to do OK (though, since 2008, not fantastic) within it. Switzerland withdrew its application to join it less than a month ago (for reasons I’m still not clear on):

          https://www.rt.com/news/346884-switzerland-eu-membership-application-rejected/

          I’ve never understood this line of argument you’ve typed out here.

          Yes, Brexit isn’t magic and won’t turn Britain’s institutions into those the U.S. had c. 1853. But it definitely can be made a step towards that direction, if the British like it to be. The British, despite their clear and evident failures, are not a stupid people. I’m sure they can come up with some great deals with the E.U. and make up for some of the subsidies they’ve lost, and even give out some of the savings they’ve gained to the NHS. It’s all discretionary. I trust British politicians to make their own deals as opposed to unelected Brussels bureaucrats making theirs for them -possibly for the benefit of the French and Italians and other European nations at the expense of Britain and its people.

          “The leftists have a variety of non-hidden motivations, such as being pro-immigrant (from other EU countries at least).”

          -I’ve always thought EU migration was generally vastly superior to Pakistani migration. Look at Rotherham. Only 4% Pakistani, and you’ve got a scandal of this size. It’s the non-European migration I’m worried about, as well as the floating unexploded Turkey bomb.

          I’ve never been a fan of irrational regulatory pile-ons, which Britain’s interference in the regulation-making process could conceivably exacerbate.

          • John Colanduoni says:

            If the E.U.’s so good, should Canada join it? Should the U.S.? Again, Norway, Canada, and Switzerland seem to do fine without the E.U., just like Ireland seems to do OK (though, since 2008, not fantastic) within it.

            We’re not talking about switching world lines to a universe where the UK never joined the EU, we have to compare the UK as it is now as a member of the EU and UK as it will be in the near-ish future as not a member of the EU. Any other comparison is beside the point unless I’m missing something.

            Yes, Brexit isn’t magic and won’t turn Britain’s institutions into those the U.S. had c. 1853. But definitely can be made a step towards that direction, if the British like it to be. The British, despite their clear and evident failures, are not a stupid people.

            The problem is that all of the claims were about having more say about regulating their own country instead of some of that coming down from on high. However, most of the regulations I saw mentioned in the debate were product/commerce related, so they will only really be free if they restructure the very nature of their economic links with the mainland, not just the trade deals. Remember that almost half of their exports go to EU countries.

            I don’t see anyone with a good plan (and few with the desire) to actually make the UK economy less dependent on the EU. I suppose their exit will effectively do that, but not in a good way (i.e. the trade is unlikely to be replaced by something equally profitable).

            I’m sure they can come up with some great deals with the E.U. and make up for some of the subsidies they’ve lost, and even give out some of the savings they’ve gained to the NHS. It’s all discretionary.

            As I mentioned, there seems to be little faith from either side in saving money that can be put into domestic programs by leaving, now that it’s not merely a campaign slogan. So again I have to ask, where are the benefits?

            -I’ve always thought EU migration was generally vastly superior to Pakistani migration. Look at Rotherham. Only 4% Pakistani, and you’ve got a scandal of this size. It’s the non-European migration I’m worried about, as well as the floating unexploded Turkey bomb.

            Note that the UK is not part of the Schengen Area; they still have their own border checkpoints. So although their membership demands they give free passage to EU citizens, they are not vulnerable to migrants sneaking into the Schengen Area and waltzing through the Chunnel. In fact, last year the UK added passport checks for those exiting the UK. So if you’re a fan of immigration of EU citizens, then that should actually be in the con column for leaving.

          • E. Harding says:

            My fear was the E.U. was exactly the sort of institution that could enforce Merkel’s Boner (in the timeless words of Steve Sailer) and establish quotas for member countries for accepting migrants from countries destroyed by NATO. Not sure if they actually did this or not (the UK has accepted very few migrants), but it definitely seems like the sort of thing that could happen 15-20 years from now. Compared to this, Polish and Romanian immigration to Britain is small potatoes.

            “UK as it will be in the near-ish future as not a member of the EU.”

            -I suppose it’ll be, at worst, like today’s Canada.

            “I don’t see anyone with a good plan (and few with the desire) to actually make the UK economy less dependent on the EU.”

            -The E.U. has fairly few trade deals with other countries. Britain outside the E.U. can establish its own trade deals with the New World.

            “So again I have to ask, where are the benefits?”

            -What? The Brexit will save at least 100 million pounds per week. That can go wherever the British Parliament, which at least has some tenuous connection to the British people, would like. Again, it’s all discretionary.

          • John Colanduoni says:

            -I suppose it’ll be, at worst, like today’s Canada.

            Canada’s reliance on the EU’s economy is significant, but still lower than that of the UK. Also, it’s hard to find a way to find a parallels for immigrants in high-demand fields from EU countries spontaneously scrambling to find some alternate living arrangements in Canada’s current state. London’s startup scene is already feeling the effects of this.

            -The E.U. has fairly few trade deals with other countries. Britain outside the E.U. can establish its own trade deals with the New World.

            My point wasn’t the renegotiation of trade, it was that as long as almost half your exports go to EU countries you’re going to bound by a ton of byzantine EU regulations, no matter what trade deal you get (unless they say something like “UK can sell products not up to code in the EU”, which would be original to say the least). This applies to the US and China as well, but both have a lot more bargaining power to push regulations in the direction they want. The UK’s representatives currently get to actually vote on these measures, but will no longer be able to after leaving. Hence me struggling to see how they are liberated in this aspect.

            -What? The Brexit will save at least 100 million pounds per week. That can go wherever the British Parliament, which at least has some tenuous connection to the British people, would like. Again, it’s all discretionary.

            Maybe it’s being misrepresented, but I saw all major Leave officials distancing themselves from that point since the vote ended. Also note that just the NHS has a budget of over 2 billion a week, so it is not clear that this is worth all the economic strife the UK will feel, even if it only takes the form of knee-jerk reactions by markets, investors, and businesses.

          • J Mann says:

            John, I don’t have much real expertise, but I think the regulatory burden of exporting to a country is typically less than the full burden of being in the country.

            Trivially, employers who don’t export to Europe won’t necessarily get all the regulations, but also, I think the US exports to Europe and is subject to less European regulation than Britain. I’m sure there are some duties that fund inspections of incoming goods, so it might ultimately still be a loss, but it’s hard to say for sure.

      • vV_Vv says:

        The thing that the Remain side never really bothered to explain to people like me what’s so good about the E.U. I live in the U.S. and I’m a U.S. citizen; I didn’t know whether Britain should Leave or stay until a few weeks before the vote, though I always sympathized with the idea of Leave.

        Do you think that your state would fare better by remaining part of the US or would it do better if it left and became an independent sovereign country? If you think that it would fare better in the US then you have the answer to why Brexit is a bad idea.

        it became clear that almost every supporter of Remain was a hack or a leftist or some financier. … Supporters of Leave, meanwhile, seemed to be champions of British freedom and independence, as well as solid men of the Right and true nationalists.

        Yes, people like Boris “there is no plan” Johnson and Nigel “I want my life back” Farage are solid men of the Right and totally not complete hacks.

        • E. Harding says:

          “Do you think that your state would fare better by remaining part of the US or would it do better if it left and became an independent sovereign country?”

          -I dunno. I expect the situation not to be all that different, with a few more border controls and trade restrictions.

          Boris seems like a hack (why else wouldn’t he run when everyone wanted him to?), but Farage doesn’t.

          • vV_Vv says:

            -I dunno. I expect the situation not to be all that different, with a few more border controls and trade restrictions.

            Don’t you think that border controls and trade restrictions would be a net loss? What about things like defense?

            Boris seems like a hack (why else wouldn’t he run when everyone wanted him to?), but Farage doesn’t.

            How is Farage not a hack? He campaigned for decades for the UK to leave the EU, now that he nearly got it, instead of leading the country during the transition, he bails out. But of course he doesn’t quit from his seat at the European Parliament, so he will still get this MEP salary while presumably doing nothing.

            It seems to me that all the Leave campaigners were bluffing: they were hoping to lose the vote by a close margin in order to get a political boost without having to do anything concrete afterwards. Instead they won and now it’s clear that they have no idea of what to do.

          • E. Harding says:

            “What about things like defense?”

            -Irrelevant. Canada’s not going to invade. Neither is Mexico or Cuba.

            “Don’t you think that border controls and trade restrictions would be a net loss?”

            -Sure; maybe, but it depends on the quality of immigration.

            “he bails out.”

            -Yeah; of leading UKIP. When your life’s work is complete, you deserve to retire from leading the cause.

            “they were hoping to lose the vote by a close margin in order to get a political boost without having to do anything concrete afterwards.”

            -Some were; others weren’t.

            “and now it’s clear that they have no idea of what to do.”

            -So what? What are they supposed to be doing now? It’s a few months before David Cameron leaves. More than enough time to come up with some plans.

          • TheAncientGeek says:

            One of the advantages of planning in advance is that you can realise that no plan is workable, and what you are trying to do is a bad idea.

      • Quixote says:

        Posts like this judge all points by simplistic education / class combinations (and sometimes mix in race) and have very simple narratives without engaging with consequence make me wish we had a down vote button here. I know it’s probably better we don’t. But I still feel the need to express my view that this is a post of particularly low quality.

    • Alphaceph says:

      On the subject of Brexit, I don’t understand why the EU commissioners insist on completely free immigration from any EU country to any other. Why tie a contentious political choice that has a lot of downsides and few upsides to the much more clearly beneficial idea of free movement of goods and free trade in information services?

      • erenold says:

        Just for absolute general clarity and not suggesting that you personally were confused, there are two things going on here that are very often conflated by non-Europeans. One is the core principle of freedom of movement of labour, expressed in one of their founding treaties, and made explicit by Directive 2004/38/EC. The other is the Schengen Area, the abolishment of passport, visa, and any other type of border control. The latter is to some extent the offshoot of the former, but they are separate things entirely. Most importantly, the first is compulsory for EEA members, and the second is wholly optional (with the sole caveat that, pursuant to the principle of reciprocity, if you want your citizens to enjoy these Schengen travel to other countries – which is obviously very popular – then you must correspondingly grant Schengen travel to your country). Britain, most notably, has never been part of Schengen.

        Contrary to public perception, it is not and has never been the case that Britain, or any other EU state for that matter, is obliged to accept an indefinite number of individuals as opposed to workers. Specifically, art 7 of the Directive allows for the expulsion of foreign immigrants who have become “burden[s] on the social assistance system”, for instance.

        With all that out of the way and as for the why – I strongly disagree that there are “no upsides” to free immigration. Reducing transaction costs in employment decisions is arguably good. Further, for those who consider ever closer union a desirable endgoal, free immigration is obviously inherently good. I would like clarification as to how free movement of goods is “clearly beneficial” when free movement of workers is not – the same if not greater risk of economic dislocation to workers in the host country would seem to apply, for instance.

        • Douglas Knight says:

          (with the sole caveat that, pursuant to the principle of reciprocity, if you want your citizens to enjoy these Schengen travel to other countries – which is obviously very popular – then you must correspondingly grant Schengen travel to your country)

          That’s not wrong, but it seems confused to me. I wonder if you are confusing visa-free travel with passport-free travel (Schengen). I think all EU states allow visa-free entry from each other’s citizens. But if someone from far away has a visa to enter France, that person can enter Germany, but might not be able to enter Britain.

          The passport-free travel of Schengen applies to borders, not people. Yes, there is reciprocity: it applies just as much for traveling from France to Germany as it does from Germany to France. But it applies to everyone crossing that border, even people from outside the EU. Indeed, with passport-free travel, you can’t discriminate because you can’t tell who’s who.

          The UK only has three land borders, so it being out of Schengen mainly means that passports are checked on plane flights, but all planes require ID, so that’s not much of a cost. The border with Ireland (also non-Schengen) is passport-free. The other two are Gibraltar-Spain and the Chunnel train.

          • erenold says:

            Hummm… I beg your pardon, I’m afraid I don’t quite understand the objection?

            (Everything you’ve said is true btw but I don’t quite see the connection to my post, no part of which appears to be contradicted by yours)

            ETA: I think I get it. Is it that citizens enjoying Schengen travel is distinct, coincidental and unrelated from having to provide Schengen travel? If so I’m afraid that’s mistaken, I believe – unless I’m very much mistaken it is a formal and legal requirement to do so.

          • Douglas Knight says:

            Sorry, you did mention visa-free travel as part of the Schengen agreement. That is about people and thus the sentence I quoted makes sense about it. But I think that it predates Schengen.

          • erenold says:

            Ah, I see – apologies for the poor and confusing phrasing!

      • vV_Vv says:

        With freedom of movement for goods and services but not for workers you create an incentive for capital to move production to countries where the labor is cheaper.

        To some extent this already happens within the EU, but workers from poorer countries can move to richer countries, driving wages up in their home countries and down in their host countries, which creates a pressure to balance salaries and allow the workers to capture part of the premium that their employers can get from freedom of movement of capital.

        If workers are stuck in their home countries, this creates a race to the bottom between countries to secure jobs from companies that can move, if workers can move as well as companies, then they have more bargaining power.

        • Snodgrass says:

          The pressure is fairly slow, and the pushback from the host countries rather hard – yes, Bulgarian average wages have doubled since 2008 whilst Britain’s have gone up less than 15%, but there’s still a factor four gap, and it’s not clear the next twenty years can be clearly modelled to be like the last eight.

          It’s not at all clear to me that you can have free movement of people, minimum wages set per-country, and happy unskilled workers in high-minimum-wage countries – as long as a skilled Bulgarian is paid less in Bulgaria than the minimum wage in Britain, there’s a strong incentive for a British employer to hire the skilled Bulgarian over an unskilled Briton at the same wage.

          (and there is a hysteresis here; if you want to be foreman for a group of agricultural labourers, you had better speak their language, and Bulgarian is remarkably little taught in Briton)

          • Winter Shaker says:

            Veering off-topic, but… British person in Britain currently trying to learn Bulgarian here, as it happens – because I’m about to go there on a folk music retreat, not because I want to become an agricultural foreman – and I can report that I’m not aware of any formal teaching being offered with, but there are enough willing native speakers around to practice with that it is hopefully not a lost cause. I am going to have a go at getting something more formal set up on iTalki, though.

      • Edward Scizorhands says:

        Because they think they have that power.

  1. hnau says:

    The Ross Douthat quotation reminded me of a great GK Chesterton essay that attacks Kipling for being too cosmopolitan and insufficiently patriotic.

    • Interestingly enough, the book that contains that essay was published the year before Kipling published _Puck of Pook’s Hill_, which provides strong evidence against Chesterton’s thesis about Kipling. It is possible to be both a cosmpolitan and a patriot, odd though that may seem.

      I don’t see how Chesterton gets his reading of “If England were what England seems.” The poem says nothing at all about the greatness and power of England.

      • Julie K says:

        My thoughts when reading _Puck of Pook’s Hill_: Cute book, same plot as _House of Arden_, even same illustrator as _House of Arden_ (_Puck_ was written first), la-di-da… whoa! Nasty anti-semitism there!

        Have you read Diana Wynne Jones’ _Power of Three_? I think it was partly inspired by the “Dymchurch Flit” section of _Puck of Pook’s Hill_.

        • Julie K says:

          Speaking of DW Jones, she agrees with Chesterton about the grandeur of the name “Smith.” (In _Tale of Time City_ there’s a legendary hero named Faber John. One character says his name means boring old John Smith and another character responds with a rant about smiths.)

  2. Error says:

    …Are those tanks road-legal? That’s almost in my budget.

    • cassander says:

      if they aren’t, who’s going to stop you?

      • Siah Sargus says:

        The Police, who bought their own armor from American Surplus.

      • jaimeastorga2000 says:
        • “As ammunition was kept in another building, none of the vehicle’s weapons could be loaded or used by Nelson”

          I can’t hep but feel that the outcome may have been different if this had not been the case.

          • youzicha says:

            But also “According to live news coverage at the time, officials were seriously considering asking for help from the United States Marine Corps at Camp Pendleton in the form of a Cobra attack helicopter, as the police force lacked the means for disabling a tank”.

            I think, once you steal a tank, you pretty much have to resign yourself to a bad ending, although the details of how you get there might vary.

          • Pku says:

            Hey, it worked for Bart Simpson.

          • Nornagest says:

            And for James Bond.

          • Edward Scizorhands says:

            Can a single person load and fire weapons in a tank?

          • John Schilling says:

            Can a single person load and fire weapons in a tank?

            A modern tank will have several machine guns that are designed for one person to load and operate, but they would have to move from the driver’s position to the turret to do so – I believe that is possible even for non-contortionists in both the M-60 and M-1 series tanks(*), but it won’t be fast and the tank will be both immobile and defenseless in the meantime. Operating the main gun will require shifting between the gunner’s and loader’s positions; both of those are in the turret at least, but expect maybe one round per minute rather than the half-dozen or so a trained crew can manage.

            Against an adversary who assumes Tanks are Invincible, a fully-armed tank with one man who knows how to use it could cause a great deal of damage. Against someone who knows how limited a one-man tank crew would be, and I’d assume any major police department will have veterans who know the basics, my money is on the police.

            * Russian surplus tanks will be more cramped, but if you buy the T-34 that comes with a ball mount for a machine gun that can be fired from the driver’s compartment. And the T-72 has an automatic loader, so one man in the turret can fire the main gun at full rate.

  3. gwern says:

    /r/AccidentalRenaissance: everyday photos which, when you think about it, look kind of like Renaissance paintings.

    All this needs is some deep ‘style transfer’ from actual Renaissance paintings, and it will be perfected.

  4. The caveats at the link about models and eating disorders end up saying that we know practically nothing about models and eating disorders.

    Now that I think about it, we probably don’t have accurate information about the prevalence of eating disorders in the general population.

    • Nasitrap says:

      I think that could be true. I’m a high-functioning anorexic (or something), but won’t show up on anybody’s statistics, as I’ve never been treated or evaluated officially.

      I wonder if there are other high-functioning people who don’t want to deal with the hassle of treatment. I don’t have depression, anxiety, OCD, or any other mental health issue (that I’m aware of). I’m successful in my career and personal life.

      To me it’s sort of like my nearsightedness – something I could probably get treated (LASIK) if I wanted to spend the time and money, but the cost/benefit calculation hasn’t favored treatment yet and may never will.

      • Siah Sargus says:

        If you don’t mind me being a bit probing, what does “functioning anorexic” mean in terms of your day to day eating habits?.

        • Nasitrap says:

          The usual thing I do is allocate a ration of food at mealtimes that won’t make me crazy but also probably isn’t enough for my level of activity. If it’s not a stressful day (which it’s usually not) then I’ll have something more.

          • onyomi says:

            I’m sure I can’t understand what it’s really like for anorexics and that this is somehow an insensitive thing to say, but I sometimes wish I could channel my OCD into something useful like this (though arguably my obsessive fear of failure is responsible for me overcoming laziness long enough to have any professional success).

          • onyomi, there’s a lot of variation in anorexia. If it’s at a level where people think you look too thin and you’re obsessed by food and you’re doing some damage to your bones, that’s one thing.

            If it’s death by starvation in a year or two, that’s another.

            There’s some discussion of whether anorexia should be defined by BMI (which is a lot of the definition now) or by behavior, since it’s possible to be eating dangerously little food while having a high or normal BMI.

            My knowledge of anorexia comes from reading. You could try that.

          • Nasitrap says:

            @onyomi, not sure what you mean about “something useful?” (I don’t take anything you said as insensitive)

            Moving from a non-urban area to a city has made life easier for me; people seem more accepting of dietary preferences without requiring a bunch of explanation.

          • gbdub says:

            Your description to me sounds sort of like the standard description of a “recovering alcoholic” – you’re still tempted to destructive behavior (drinking or not eating), but have developed a workable mitigation strategy to stay mostly healthy. Is that about how you see it?

          • Nasitrap says:

            @gbdub, I’m not opposed to that view, but I’m not sure it fits for me. I think a lot of people would consider a “recovering alcoholic” who drinks moderately (as opposed to not at all) to be not doing so well.

            I like the nearsightedness analogy. It’s annoying, but I manage well enough that I don’t consider it to be a defining characteristic.

          • onyomi says:

            I don’t know if my mental model is correct, but my best way of understanding anorexia is as a kind of OCD as applied to food quantity and a fear of getting fat–that is sort of “watching what you eat” to a pathological degree.

            “Watching what I eat” (not eating too much, eating healthier foods) is not something that comes naturally to me. Rather, my default is to always eat a little too much such that I slowly gain weight if I don’t make a periodic effort not to.

            I’m not fat by US standards, but I’d like to be 10-20 lbs. lighter for health and aesthetic reasons. The main obstacle to that, I believe, is lack of consistency in “watching what I eat,” which feels like a big hassle and something which requires a specific, conscious effort.

            Things I get obsessive about, conversely, require a conscious effort to ignore, but most of them I’d be better off if I naturally ignored them. I am certain I have thrown away no small amount of perfectly good food due to a periodic obsession with food safety. Naturally ignoring “watching what I eat,” however, makes it difficult to achieve my desired weight, though of course it has the upside of not causing me to suffer downsides of anorexia.

            Being skinny is high status in most of the developed world, so if one must have an obsession, anorexia seems like a relatively useful one to have (though I do realize not all anorexics are skinny).

          • gbdub says:

            Sticking with the alcohol analogy, which may be lousy, I guess I think of a “high-functioning alcoholic” as someone who definitely drinks to excess very frequently, but basically gets by despite being constantly drunk. So “high-functioning anorexic” struck me as “definitely eating disordered, underweight and frequently eating too little, but good at hiding it / avoiding treatment”.

            Whereas you seemed to be describing something more like “psychological impulse to anorexia, but usually able to avoid severely unhealthy eating habits”. The latter scenario seems more “under control”.

            Anyway sorry if I’m prying too much / mislabeling. I find first-person input from non-neurotypical people to be one of the more compelling aspects of this commenting community, so thank you for your input.

          • I used to believe anorexia was driven by fear of getting fat, but at least some of it includes desire for high status, and the status is about heroic self-control as well as being thin.

          • Hyzenthlay says:

            I used to believe anorexia was driven by fear of getting fat, but at least some of it includes desire for high status, and the status is about heroic self-control as well as being thin.

            I suspect it also has to do with just wanting a sense of control over one’s body and diet; onyomi likened it to OCD, which is similar to my sense of what it means. Correct me if I’m wrong because I’m not anorexic so I’m just going off of general impressions, but I think the idea of it being all about fear of fat or body image is just one variation that has gotten a lot of media attention, and the reality is more varied.

          • Quixote says:

            At anorexia levels of thinness menstration stops. For people with particularly painful or disruptive periods that’s a big deal. Also many of the hormonal changes assciated with the monthly cycle also go away. For people who have sensitivities that make mental stability a big concern (ptsd among many others) removing that hormonal fluctuation from ther lives is very alluring.

            It’s also about being thin and control and all that, but the other biologically and hormone driven physical and mental factors can be big.

            Source : discussion with multiple people with anorexia issues or recovering from said issues with whom I had been in close long term relationships

          • houseboatonstyxb says:

            @ Nasitrap

            Thank you for posting this. It has got me thinking about some things I ought to be thinking about.

            If there’s a spectrum of anorexia, I’m probably on the milder end of it. My doctor is urging me to gain weight, as have family members since childhood. But I don’t think I’m too fat or fear overweight (always had BMI low 20s, near bottom of Normal range). I just forget to eat, or get too tired to cook, or…

            …my big problem is that after food I get sleepy and useless, sometimes for hours. So I postpone eating and skip meals when possible. Caffeine doesn’t help, low-carb didn’t help.

            ETA: My current BMI is 15.23 and dropping fast, and my doctor is very concerned, says I’m in ‘deconditioning’. I think the decline started around January, after nausea* from Aricept (tried it Nov-Feb) and no kitchen electricity Jan-Feb.

            * nausea with associated lack of appetite, constant feeling of fullness that makes food repellant, etc.

          • Hyzenthlay says:

            At anorexia levels of thinness menstration stops. For people with particularly painful or disruptive periods that’s a big deal.

            That’s interesting…I’d never really heard about that as a motivating factor before.

            I mean, I can definitely understand the allure of skipping painful periods; I did that myself for several years by taking continuous BC (skipping the week of placebo pills every month). I know a lot of people take BC pills specifically for that reason, and nowadays there are brands that are actually designed for that. Trying to stop menstruation by starving oneself seems kind of counterproductive, given that reducing your caloric intake to starvation levels also causes a bunch of painful physical and psychological side-effects.

            I mean, I don’t doubt that some people do it…it just seems like the negative effects would cancel out the benefits.

          • Nasitrap says:

            @gbdub, I see what you mean. I’m not sensitive about labels or any prying; this discussion has been interesting.

            My own experience varies between the two poles you describe – the “good at hiding that there’s a problem” some of the time and “usually able to avoid really bad habits” most of the time.

            @Nany Lebovitz, I think I do have a drive to have “heroic self-control,” so that rings true. But I don’t think I always did, and I’ve had the same eating disorder as long as I can remember.

            @Hyzenthlay, I think you’re right that the media version of anorexia isn’t necessarily typical. I do think media portrayals of really thin people might hurt anorexics more than others, but I’m skeptical about causality.

            @Quixote, that’s really interesting. I’m a guy, so I hadn’t thought of that.

            @houseboatonstyxb, good luck and best wishes.

          • I’ve seen an argument that for (some) people to have a switch which enables them to do a lot of work on little food is an evolutionary advantage when there’s a famine.

            It strikes me as a dubious sort of argument, but this doesn’t mean it’s wrong.

          • Nasitrap says:

            @Nancy Lebovitz, it strikes me as plausible that humans might have an “avoid food” macro, and that various things could trigger it. This goes back to the original “maybe we don’t really know much about prevalence of anorexia in the population” point you made.

            When I was a teenager I wondered if I’d “grow out of” disordered eating. I know/knew some other people who had issues during puberty and then stopped having them after.

            The severity of my own issues did seem to peak in late high school and in college. The times I thought “this is a problem and I need help with it” were all then. The peak levels have been lower since then (I’m in my early thirties now), but the typical level hasn’t changed much.

            Sometimes I wonder if I did mostly grow out of it and just don’t know any other way to behave. I’ve devoted aeons of subjective time to figuring out how to plan and delay meals, developing exercise regimes, etc. – is it just a habit now? Usually deviating from my normal coping routine winds up being unpleasant and I go back to it.

            (I hasten to add to this discussion that I consider myself quite fortunate to have only ever had mild problems with anorexia and I strongly encourage anybody who thinks they might have a problem to get help. Anorexia can be crazy dangerous; don’t dismiss it because some guy on the Internet dubiously compared his version of it to myopia.)

          • Tracy W says:

            I follow anorexia research a bit and the latest treatment for children and adolescents is the Maundsy approach – basically fed them up until their weight’s back comfortably into healthy ranges, then talk about any remaining mental problems.

            The idea is that starvation causes disordered mental processes. It came from research in WWII (by the Allies, not Nazis) into starvation. The developers of the idea are formally agnostic about the causes of anorexia but speculate that often the cause is a drop in weight (eg a diet or an illness) that causes the disordered thinking to start.

            (The reason it’s not for adults is the problems of forcing a legal adult to eat.)

    • Lumifer says:

      “Eating disorders” is a very vague concept. Sensible eating to me is an eating disorder to you and vice versa.

      • DavidS says:

        I think another way to look at it is that for eating disorders (as I think for lots of mental illness etc.) there are multiple questions
        – Do you have some issues around food (or organising things, or sexuality, or whatever) where it takes on some strongly felt special status
        – Are these unusual?
        – Are they harmful in the current social context?

        Loads of people have I think ‘issues with food’ in the way that lots of people I wouldn’t call alcoholics have ‘issues with alcohol’. Food isn’t just another thing which they can give or take and treat as a means to an end (taste or satiation): either they eat for comfort, or they find eating uncomfortable, or both.

        Sensible eating is about how what you eat, and I think two people can eat the same amount of stuff (healthy or unhealthy) without having an eating disorder. If I eat very little and am dangerously underweight because I’ve chosen to be (e.g. it means I earn more money as a model) that doesn’t mean I have an eating disorder. If I eat a lot because I genuinely enjoy it and think it’s worth it to be fat, I don’t have an eating disorder. It becomes disorderly I think when you’re seriously conflicted (I wish I was thin, but can’t stop eating, or I know I’m dangerously underweight but can’t face eating)

  5. gbear605 says:

    http://thefederalist.com/2016/07/01/neil-degrasse-tysons-rationalia-would-be-a-terrible-country/

    On why rationality (actually talking about Neil DeGrasse Tyson, not LessWrong) would make a terrible country.

    I disagree on the general idea, but I don’t know much about Tyson’s specifics that he has not shared with the public, if they exist at all.

    • eniteris says:

      There are so many things wrong with this article.

      I’m sorry, is the example with the Tower of Babel supposed to support his point? The author knows it’s fiction, right?

      Your beliefs are not objective facts of reality. Objective facts of reality are your only beliefs.

      At least LessWrong rationality isn’t blind to its own biases.

      tl;dr, author thinks that rationality is just another belief system.

      • It might be true that “rationality” is not just another belief system. But it’s quite clearly the case that Rationality(tm) is just another belief system. This applies to both the Yudkowskian-Alexandrian branch and whatever it is that Tyson preaches.

        • Chrysophylax says:

          http://lesswrong.com/lw/mm/the_fallacy_of_gray/

          Beliefs are not equal. Calling something a belief system tells you nothing about how accurate the component beliefs are. Pretending that all beliefs are equal is a monkey politics move, not an attempt to work out how reality actually works.

          The other error you seem to me making requires a longer answer. I recommend the Lawful Truth posts, partularly “The Second Law of Thermodynamics, and Engines of Cognition”. In summary: the laws of probability apply always and everywhere. There are exact mathematical laws specifying *precisely* what your beliefs should be at any given moment; and to the extent that you differ from these beliefs, you will (on average) be wrong. (And these laws are backed up by consistency and uniqueness proofs.) The fact that the exact maths is computationally intractable has no more relevance than the fact that you can’t actually build a Carnot heat engine. There is an ideal; all feasible systems are useful to the extent that they mirror the ideal and flawed to the extent that they don’t.

          Saying that something is “just another belief system” tells me nothing about how well it approximates the output of the provably-unique ideal process for forming accurate beliefs. It’s as serious an error as making no distinction between the Wright Flyer and a Su-27.

          • “There are exact mathematical laws specifying *precisely* what your beliefs should be at any given moment”

            How do those laws tell you that you should obtain the priors that are then modified through Bayesian reasoning?

          • FrogOfWar says:

            There are exact mathematical laws specifying *precisely* what your beliefs should be at any given moment

            Math (maybe) specifies these things for logically omniscient reasoners with well defined credences over every proposition (including ones no one has considered) and who set their priors by, as you note, “computationally intractable” methods. It does not specify the path that actual humans can take to approximate it.

            Even accepting Bayesianism as an ideal, that doesn’t tell us anything about whether imitating the few aspects of Bayesianism we can actually implement will get us closer to the ideal than alternatives. There is certainly no general rule that you get the best outcome by trying to imitate ideal actors whose capacities differ greatly from yours.

            The groups that best approximate the ideal apparently don’t try implement Bayesian calculations explicitly. They instead treat Bayesianism as encoding a useful set of qualitative heuristics, which is how I tend to view it.

            It would need to be something more for Rationalia to not be deeply misguided.

          • Chrysophylax says:

            @David Friedman:

            Note that I did say “your beliefs at any given moment”, i.e. the beliefs you should have given all the information available to you if you use it all perfectly.

            For the ideal case, as far as I understand it, you want something like a prior over Turing machines with each assigned a complexity penalty according to its Kolmogorov complexity when implemented using a standard UTM with a prefix-free code. (The choice of UTM should just add a small and irrelevant constant unless you do stupid things.)

          • Chrysophylax says:

            @FrogOfWar:

            I didn’t say that you should pretend to be a Solomonoff inductor. I also didn’t endorse Rationalia; I haven’t read Tyson’s article and expect him to be totally out of his depth.

            I said that it is an error to look only at differences in (ill-defined) kind and ignore differences in degree, and an error to think that you are free to believe whatever you like.

            I think you are making an error about probability theory and superpredictors. Your inability to do the maths with a human brain says nothing about the validity of the theorems. The theorems define the gold standard, the absolute best that could theoretically be achieved. Any deviation from the ideal maths must *necessarily* make you perform worse (in expectation). A mind that *could* do the maths would thrash the superpredictors – in fact, it’s a theorem that it makes bounded error with respect to *any* computable predictor, because it can just consider every computable hypothesis.

            Deciding to trust heuristics because you can’t do the maths is as silly as deciding that petroleum engines are fundamentally better because you can’t build a Carnot engine.

            Thermodynamics doesn’t give you useful hints about engine design; instead, a better engine is one that is more efficient at the thermodynamic processes that define “being an engine”. Bayes doesn’t encode useful heuristics. *Heuristics are useful to the extent that they reflect Bayes.*

          • vV_Vv says:

            @Chrysophylax

            (The choice of UTM should just add a small and irrelevant constant unless you do stupid things.)

            The choice of UTM introduces a bias, not necessarily small, that is eventually asymptotically canceled in the case you are a purely passive predictor (Solomonoff induction). In the case where you want to actually do something based on your belief (AIXI agent), then the bias introduced by the choice of the UTM may be never canceled under realistic assumptions.

            I think you are making an error about probability theory and superpredictors. Your inability to do the maths with a human brain says nothing about the validity of the theorems. The theorems define the gold standard, the absolute best that could theoretically be achieved. Any deviation from the ideal maths must *necessarily* make you perform worse (in expectation).

            This is Platonic idealism.

            Theorems are relevant to practical problems only to the extent that their assumptions reflect the physical world. When they don’t, then the problem is in your mathematical model, not in the physical world. The universe doesn’t care about your math.

          • Chrysophylax says:

            @vV_Vv:

            Define “small”. If your choice of UTM creates a bias (not just an error) large enough that it doesn’t rapidly vanish in the face of inductive hypotheses, I strongly suspect you are trying something fishy. Also, bounded error is still bounded and convergence is astonishingly quick. If you want to create a bias that still matters after X bits of input (for moderately large X, e.g. a million), you need a UTM that can’t encode a strong inductor (or anything that encodes one) in fewer than X bits. This requires really extreme shenanigans.

            I have not at any point said that you should try to perform computations you don’t have the resources to seriously attempt. Bayesian inference is not an anytime algorithm when done by humans. I do assert that if you could do the maths, you ought to do the maths.

            The point of a theoretical ideal is not to be a fully general solution to all real-world problems. The point of a theoretical ideal is to make the reasoning clear, so that you can see *why* it is ideal. You can’t analyse realistic cases until you understand simplified abstractions. If you can’t design a theoretical ideal solution, you don’t really understand the problem and you don’t understand what is and isn’t possible. Until you’ve understood the maths, you’ll keep designing perpetual motion machines and accepting Dutch books.

            You can’t solve the three-body problem, but that doesn’t count against the laws of physics. Reality obeys and embodies the mathematically simple universal laws, not the kludges you can run on a human brain. Any heuristic you propose will be much less simple and universal than the laws of probability theory.

            If you know enough about SI to argue the details with me, but can’t see why it’s an error to have more respect for “useful qualitative heuristics” than for the simple, universal, precisely quantitative laws they approximate, I really don’t know what to say.

          • Cerebral Paul Z. says:

            A just machine to make big decisions,
            Programmed by fellas with compassion and vision!

          • Chrysophylax says:

            Are you trying to make some point or are you just trolling? If the former, you’re making it so badly that you’re effectively a troll. Please don’t do that.

            There’s no point in actually programming SI because it requires a level-1 halting oracle (or an infinitely large / long-running computer). The main point of thinking about these things is that if you can’t solve a problem with inifinite computing power, you’re fundamentally confused about how to solve it and will get nowhere trying to solve it with realistic constraints.

          • FrogOfWar says:

            @Chrysophylax

            At no point in my comment did I argue against the status of the ideal Bayesian as an ideal, so I’m not sure what you take my “errors about probability theory to be”. My point was simply that even if we grant it the status of the ideal that doesn’t tell us the best way to approximate the ideal, and it certainly doesn’t formalize reasoning for actual human beings. You seem to have acknowledged this.

            You also seem to have acknowledged that your claims that math alone establishes the ideal status of the Bayesian reasoner was overly bombastic by admitting that “shenanigans” in the choice of UTM can significantly affect outcomes. To rule out those shenanigans is to solve the Grue paradox, which math cannot do on its own.

          • Earthly Knight says:

            There are exact mathematical laws specifying *precisely* what your beliefs should be at any given moment; and to the extent that you differ from these beliefs, you will (on average) be wrong.

            There are two components to Bayesian epistemology, probabilism (i.e. having credences which obey the Kolmogorov axioms) and conditionalization. You seem to be invoking accuracy-dominance reasoning on behalf of Bayesian epistemology, but you’re confused about the role it plays. Probabilism is a purely synchronic rule– it imposes (very weak) constraints on how your credences should be related to each other at a given time, but tells you nothing about how your beliefs should change over time. An accuracy dominance argument can be given for probabilism: any credence function which violates the Kolmogorov axioms is always less accurate than some credence function which satisfies them, no matter the state of the world.* There are a couple of problems with this argument, though. First, it turns out there are some rather outre cases where probabilism does not accuracy dominate after all.** Second, the arguments is largely circular. Accuracy depends on our choice of scoring rule, and the dominance argument applies only to “strictly proper” scoring rules. But “strictly proper” scoring rules are just those which favor probabilistic credence distributions over all others.

            Conditionalization is the diachronic Bayesian rule which constrains how your credence should be related to each other over time. The chief support for conditionalization comes from diachronic Dutch book arguments*** and expected accuracy arguments.**** But diachronic Dutch book arguments have fallen into disrepute, first, because they are pragmatic rather than epistemic arguments, and second, because there are too many of them and they end up imposing constraints on rationality, like van Fraassen’s reflection principle, that few people actually want to endorse. The expected accuracy arguments fare slightly better: they tell us that only credence functions which update by conditionalization maximize accuracy by their own lights in all cases. But accuracy is again defined in terms of strictly proper scoring rules, which means that these arguments are also effectively circular, for roughly the same reasons as above.

            I should add that conditionalization is not, prima facie, a very plausible constraint on rationality. Why, of all things on heaven and earth, should my credences be constrained by what I believed in the past? If there’s more than one rational set of priors– and, according to subjective Bayesians, there is– it’s hard to see any reason for me to favor my own credence function over any other rational agent’s. Just because it’s mine?

            *From James Joyce (no, not that one, the other one).
            **Michael Caie
            ***David Lewis
            ****Greaves and Wallace

          • Philosophisticat says:

            @EarthlyKnight

            “I should add that conditionalization is not, prima facie, a very plausible constraint on rationality. Why, of all things on heaven and earth, should my credences be constrained by what I believed in the past? If there’s more than one rational set of priors– and, according to subjective Bayesians, there is– it’s hard to see any reason for me to favor my own credence function over any other rational agent’s. Just because it’s mine?”

            I’m not sure what you want to go into “prima facie”, but it’s highly intuitive that having credences that are highly unstable even while no new evidence comes in is irrational. There are highly theoretical arguments appealing to internalist principles or the irrelevance of personal identity that can be raised as a challenge (and other controversial commitments, like uniqueness, one can take on to explain the data), but it seems to me that by any reasonable interpretation of “on the face of it”, on the face of it, the fact that conditionalization prevents agents from switching between credal states for no reason is a point in its favor.

          • Earthly Knight says:

            I share your intuition, but it dissipates once we make clear that we are supposing ourselves to be permissivists and allowing that different agents may respond differently to the same body of evidence without being irrational. Bob is perfectly rational, so too is Mary, they have all of their evidence in common, yet arrive at different conclusions. Why shouldn’t Bob decide to adopt Mary’s credences wholesale as his own, given that they are, by hypothesis, perfectly rational? Why should he be hidebound to the beliefs he formed thirty years ago? The requirements of rationality should be impartial and impersonal, which conditionalization + permissive Bayesianism is not.

            Christensen gives the example of meeting an icthylogist at a party, and using sophisticated technology to download some of her fish-related beliefs directly into your own brain. This procedure violates conditionalization, too, but I can see nothing wrong with it.

            The intuition that shifting credence functions without changes in evidence is illicit arises, I suspect, from subconscious allegiances to reliabilism or the uniqueness thesis. A cognitive process which produces different outputs at different times when fed the same body of evidence is ipso facto unreliable, and, consequently, the beliefs it forms unjustified. Or: there is a uniquely rational doxastic attitude we must take towards any body of evidence, which, given probabilism, guarantees that a rational agent’s credences will change only in response to new evidence in a way that mimics conditionalization. Indeed, if we are all impermissivists at heart, this would explain both why credence-shifting seems objectionable and conditionalization’s superficial appeal as a tool to rule out credence-shifting.

          • Philosophisticat says:

            @EarthlyKnight

            I find that reflecting on a commitment to permissivism doesn’t make the intuition that the agent who waffles moment by moment is irrational go away. That’s just a report, though.

            It’s important to keep in mind that thinking that somebody’s credences are rational for them doesn’t commit one to thinking that they’re as accurate as ones’ own. And I think the latter is what’s relevant to whether one ought to adopt them. Bob thinks that Mary’s credences are inaccurate and her priors are misleading, though she is rational in having them (this is one difference between the regular case and the icthyologist). That seems like pretty good reason for him not to adopt them. Now, I get that there’s a hint of something suspiciously circular going on here, but I think it gets tricky to lay out and defend that worry. I think the defender of conservative principles like conditionalization can and should resist characterizing their view, for instance, as something like “the fact that a belief belongs to me is a reason to think it is true”.

            The idea that the requirements of rationality must be impartial and impersonal sounds well and good, but cashing out the right sense of impartiality and impersonality, defending the claim that rationality must respect that sense, and showing that conditionalization violates it are all, I think, nontrivial.

            I’d worry about drawing too many conclusions off the icthyologist case. For one, it concerns a practical decision rather than an act of epistemic updating.

            I agree that there are other prima facie plausible explanations of the irrationality of waffling. I think there are independent reasons to reject them and I doubt that as a psychological matter our (or at least my) intuitions are explicable as subconscious acceptance of something like reliabilism or uniqueness, to which I have never been very sympathetic.

            Anyway, I’m not really out to defend conditionalization, which I think is false. The main point is just that permissivism is prima facie plausible, the waffling-is-irrational intuition is pretty compelling, and that that grants a lot of initial attraction to the thought that your past mental states can constrain your present ones. It might be that for fancy theoretical reasons that turns out to be wrong (in fact, I’ve argued that it’s wrong in print), but it’s not as ‘out there’ a thought as you seemed to present it.

          • Earthly Knight says:

            Bob thinks that Mary’s credences are inaccurate and her priors are misleading, though she is rational in having them.

            It seems to me that any reason to think that Mary’s beliefs are inaccurate must equally be reason to think that her beliefs are irrational– it was accuracy arguments that led us to adopt probabilism and conditionalization as constraints on rationality in the first place, after all. If rationality and accuracy are tightly wedded enough for us to become Bayesians, I don’t see how they can come apart at this later stage.

            I’d worry about drawing too many conclusions off the icthyologist case. For one, it concerns a practical decision rather than an act of epistemic updating.

            I don’t think this matters. There are two faults the downloading procedure might have: either the act of downloading the beliefs could be practically irrational, or the resulting doxastic states could be epistemically irrational. Given that the whole shebang is intuitively unobjectionable, I think we can conclude that neither fault is present. But this implies that it is at least sometimes acceptable to violate conditionalization.

            The main point is just that permissivism is prima facie plausible,

            Huh? I’ve never met anyone who was naively a permissivist. Sure, people will mouth the platitudes about how reasonable men can differ, but when it comes to probabilities, every layman I’ve ever encountered came into the world believing that the uniquely rational prior credence distribution was fixed by some sort of indifference principle. It’s only after being led through some version of Bertrand’s paradox, often more than once, that you can get them to see that their indifference intuitions are untrustworthy. So permissive Bayesianism is not “prima facie plausible”– it takes, as you say, highly theoretical arguments to make people view it as a live possibility.

          • Chrysophylax says:

            @FrogOfWar:

            I agree that trying to imitate the ideal without thinking about human cognitive architecture is not a practical way to get closer to it.

            I said “I think you are making an error about probability theory and superpredictors”. I’m trying to argue that a useful step towards reasoning well is to keep the ideal in mind. I think that “Here are some useful heuristics” is less powerful than “Here are some useful heuristics for approximating an ideal”, because the latter keeps the target in sight better. It helps with tsuyoku naritai, it helps avoid motivated reasoning, it helps keep you focused on seeking truth instead of winning arguments, and so on. The errors I was accusing Mai La Dreapta of are not possible when you keep the ideal in mind.

            The Grue-Bleen Paradox is bloody stupid. Grue is time-dependent in a way that green isn’t; it’s not about “expecting the future to be like the past”, it’s about hypotheses where an otherwise-invariant thing changes in a specified way at a specified time being strictly more complex than ones where no change occurs.

            The shenanigans in question involve choosing a UTM that not only makes induction ludicrously improbable, but makes anything that could output induction ludicrously improbable; and even this will only slow things down. There will be some finite weight of evidence that makes an inductive hypothesis the best predictor, no matter how stupid the prior is, if the environment admits of induction.

            And for the third time, when I said “your”, I meant “your”! I did not claim to be able to deduce everything with probability 1 from no assumptions whatsoever! If you wouldn’t object to me saying that two plus two is definitely four, you shouldn’t object to me saying that probability is lawful.

          • Philosophisticat says:

            @EarthlyKnight

            Bayesians (often) use accuracy arguments, but they’re a very specific kind of accuracy argument. They don’t want to claim that having credences that are inaccurate is irrational – only an externalist would want to say that. They want to claim that having credences that are guaranteed to be accuracy-dominated by other credences, by your own lights is irrational. Bob’s reasons for thinking that Mary’s credences are inaccurate are not reasons to think her credences are accuracy-dominated by her own lights – so they’re not reasons to think she’s irrational, according to the kind of accuracy reasoning that motivates Bayesians.

            I think there are ways to cash out the kind of epistemic updating conditionalization is trying to give an account of which excludes the downloader case. But in any case, it’s not a general counterexample to diachronic rational constraints, which seemed to be the feature of conditionalization you were focusing on as objectionable.

            We may have different experiences of the man on the street. I find that most of my students are naively permissivist, esp. w/r/t religious belief, and that they are very resistant to thinking that there is always exactly one credence it’s okay to assign to any proposition given the evidence. Of course, I’m sure there are ways to get them to assent to an indifference principle too. People are inconsistent that way. Since the indifference principle is a nonstarter, though, resolving that one seems like an easy call.

          • Earthly Knight says:

            They don’t want to claim that having credences that are inaccurate is irrational – only an externalist would want to say that.

            This isn’t true– the accuracy dominance argument purports to show that probabilism is rationally compulsory because non-probabilistic credences are inaccurate, full stop. And if Bob is justified in thinking that an additivity-violator’s credence are irrational on grounds of inaccuracy, how can he go on to endorse Mary’s credences as permissible when he sees them as inaccurate, too? By Bob’s lights, Mary’s credences may be even further from the truth! It’s a strange sort of doublethink to appeal to accuracy to make the Bayesian constraints binding on all rational agents in the first place, but then, once the constraints are in place, to set people free to be as inaccurate as they like.

            I find that most of my students are naively permissivist, esp. w/r/t religious belief, and that they are very resistant to thinking that there is always exactly one credence it’s okay to assign to any proposition given the evidence.

            You have to be careful with this– you’re not really sounding out whether your students have permissive intuitions unless you make explicit that they are to consider a pair of ideally rational agents who share all their evidence in common yet still hold different beliefs. I’m not so sure their laissez-faire attitudes towards religious disagreements will survive the added stipulations. But I’m quite sure that they will reject permissivism in more straightforward cases. Try telling them that it could be perfectly rational to be 90% confident that a black ball will be drawn from an urn when the urn contains some unknown mixture of red and black balls. They’ll think you’re pulling their legs.

          • Philosophisticat says:

            @EarthlyKnight

            You’re confused about the accuracy point. All non-extreme credences (1s and 0s in everything) are inaccurate. That doesn’t make them irrational. If I’m God, and I know everything, then I’m in a position to judge that everybody else’s credences are less accurate than mine. But that doesn’t mean it’s irrational for them to have those credences. Bob is in the same position with respect to Mary – he judges her credences are less accurate than his (not, in this case, because of lack of information, but because of misleading priors), but that she is not irrational for having them. The point of accuracy dominance arguments is that they show that for someone with credences that violate probabilism, that person is in a position to judge a priori of another credence that it is more accurate than their own no matter how the world turns out. That’s what’s supposed to be irrational about it. Mary is not in a position to know a priori that Bob’s credence is more accurate than hers. So the same kind of reasoning does not go through.

            Regarding the second point, one can be a permissivist without being a wild-west-anything-goes permissivist. Ordinary people aren’t wild-west-anything-goes permissivists. But consider anyone (and there are many) who claims to believe religiously ‘on faith’. They do not typically think that the evidence settles the case in favor of their belief. They do not typically think they are being irrational. And they do not typically think that faith in their particular God is rationally mandatory. This is a permissivist view.

          • Jiro says:

            The Grue-Bleen Paradox is bloody stupid. Grue is time-dependent in a way that green isn’t

            That depends on the way you phrase it. “Green means ‘grue if first observed before this date, and bleen if first observed after this date'”. Then green is time-dependent, but grue isn’t.

            You can try to phrase it in terms of something else (green means this wavelength of light), but you can translate the paradox to numbers instead.

          • vV_Vv says:

            If your choice of UTM creates a bias (not just an error) large enough that it doesn’t rapidly vanish in the face of inductive hypotheses, I strongly suspect you are trying something fishy.

            You are trying to make a theoretical argument for the superiority of Solomonoff induction, but then you attempt to resort to intuitive arguments to fill the gaps.

            There is nothing in the theory of SI that allows to distinguish between “good” and “fishy” UTMs, which is why the only theoretically guarantees that SI provides are asymptotic.

            Note that pretty much all the sane frequentist methods (unbiased estimators is the technical term) provide similar asymptotic guarantees.

            One of the main points for using Bayesian statistics is that since we don’t have access to unlimited amounts of observations, we want methods that can do well in the “small data” regime, and Bayes allows us to use prior probabilities to include our prior knowledge about the world into our estimates.

            But what if you don’t actually have any prior knowledge about the world, other than the assumption that it must be computable and that “simpler” worlds must be more probable, according to some definition of simplicity that you can’t pinpoint to any specific UTM? Then the best you can get are asymptotic guarantees, as in the frequentist case. There really is no free lunch.

            I’m not arguing that the theory of Solomonoff induction is useless, but we should consider it as a theoretical tool in our theoretical toolbox, rather than worshipping it as the One True Platonic Form of Perfect Rationality.

          • Earthly Knight says:

            The point of accuracy dominance arguments is that they show that for someone with credences that violate probabilism, that person is in a position to judge a priori of another credence that it is more accurate than their own no matter how the world turns out. That’s what’s supposed to be irrational about it. Mary is not in a position to know a priori that Bob’s credence is more accurate than hers.

            We are trying to identify why Bob should think that an additivity-violator’s credence distribution is irrational while Mary’s is not, when, by Bob’s lights, the additivity-violator’s credences are more accurate than Mary’s. I don’t think a priority can make the difference. Bob does think that Mary could know (or “know”– conscience compels me to insert the scare quotes) a priori that her credences are inaccurate– if she had adopted the most accurate set of priors (i.e. his) she would have recognized that her own priors are off-base. From Bob’s perspective, the only difference between the additivity-violator and Mary is that the former could know a priori that his credences are accuracy-dominated by probabilistic credences, while the latter can only “know” a priori that her credences are inaccurate. So Bob can reasonably demand an explanation for why he should think it irrational to have credences that are accuracy-dominated but not irrational to have credences which are merely inaccurate. Is it really the domination that’s doing all of the work?

            Regarding the second point, one can be a permissivist without being a wild-west-anything-goes permissivist.

            Sure, but the scenario I chose is the sort where even the moderate sort of permissivist will allow agents to adopt a wide range of credences. Moderate permissivists are moderate because they accept additional constraints on rationality beyond probabilism and conditionalization, typically the principal principle and maybe regularity. But neither of these constraints will preclude an agent from assigning a credence of .9 to the proposition that a black ball will be drawn from the urn.

            They do not typically think that the evidence settles the case in favor of their belief. They do not typically think they are being irrational. And they do not typically think that faith in their particular God is rationally mandatory.

            I think the more common view is that faith is in some way independent of or inoculated against evidence. But I’m not convinced that student’s attitudes towards religious disagreements really represents their naive or pretheoretical intuitions so much as (1) the long protestant tradition of fideism leaking into the students brain’s over years of church attendance or (2) a weird generalization of our civic tradition of religious tolerance to the epistemic realm. We should look for cleaner cases, like the urn.

          • Earthly Knight says:

            @ Jiro

            @ Chrysophylax

            “Green means ‘grue if first observed before this date, and bleen if first observed after this date’”. Then green is time-dependent, but grue isn’t.

            A common misconception about the Goodman paradox is that “grue” means “green if observed before time t and blue thereafter,” as though it were the nature of grue objects to change color. It sounded to me earlier as though Chrysophlyax understood the term this way. The correct definition of “grue” is parallel to the definition of “green” Jiro gives here: “green if first observed before time t and blue if not first observed before time t.” Grue objects do not change color– those observed prior to t are forever green, and those not observed prior to t are always blue.

          • Philosophisticat says:

            @EarthlyKnight

            I’m very perplexed by what you’re trying to say about the accuracy case. Forget about Mary for a second. Bob knows that his own credences are inaccurate. He could have improved his accuracy by adopting the credence function that assigns 1 to all truths and 0 to all falsehoods (God’s credence function). I take it you don’t think this is sufficient for Bob’s own credences to be irrational. That’s because Bob can’t tell which credence function that is. It’s exactly the same for Mary. Mary is not in a position to judge that Bob’s credences (rather than Joe’s, or Steve’s, or Anne’s) are more accurate than hers (and indeed, she knows that none of those credences are the most accurate – God’s credences are). Keeping her credences is the best she can do by her own lights (obviously, it’s not the best thing to do by Bob’s lights, in the same sense that none of us are doing the best thing by God’s lights – but rationality, Bayesians think, is about doing the best we can by our own lights, not those of others). If Mary violates additivity, however, then there is a specific credence function that Mary can calculate is more accurate than hers. The principle at work is something like that it’s irrational to have a credence when you can tell some other (particular) credence is more accurate, not that it’s irrational to have a credence or prior whenever some other is more accurate.

            I think the dialectic over the permissivism point is getting muddled. I’m not claiming that people naively accept bayesianism, of any sort that is accepted by philosophers. I’m suggesting that people are naively permissivist, in that they think that sometimes there are multiple permissible epistemic positions to take towards a proposition. And that’s enough, together with the irrationality of waffling, to grant some initial motivation to the idea of conservative diachronic constraints. We can just grant that the urn case is not such an occasion. I think you see it more in religious belief, moral belief, and more complex cases (the probability that Hillary will win the next election, etc.) Even if some explanation of why people look like naive permissivists of the form you propose is true (and isn’t just an explanation of why people are naively permissivist), that’s an awful lot of extra work to be resting a claim of ‘prima facie’ implausibility on.

          • Earthly Knight says:

            The principle at work is something like that it’s irrational to have a credence when you can tell some other (particular) credence is more accurate, not that it’s irrational to have a credence or prior whenever some other is more accurate.

            The permissive Bayesian is committed, I take it, to the following three claims:

            1. Bob should judge the additivity-violator’s credence distribution to be irrational.
            2. Bob should judge Fred’s credence distribution to be irrational if Fred previously held the same credences as Bob but subsequently shifted to adopt Mary’s, flouting conditionalization.
            3. Bob should judge Mary’s credence distribution to be rational.

            I am inquiring what feature Mary has which the other two lack that should make Bob judge her alone to be rational. It cannot be accuracy, or accuracy-by-Bob’s-lights, because the additivity-violator may be both more accurate and more accurate-according-to-Bob than Mary or Fred. You suggest that the feature is being able to tell a priori that some other credence distribution is more accurate than yours. But Fred cannot tell a priori any more than Mary that some other credence distribution is more accurate than his, so that’s no good, either. If what makes the difference for Fred is that there was some time in the past where he could have “known” that Mary’s credence distribution was less accurate or less-accurate-by-his-own-lights than Bob’s, well, it’s also true that there’s some time in the past when Mary could have “known” the same thing– when she was selecting her priors, she could have picked Bob’s instead.

            I’m suggesting that people are naively permissivist, in that they think that sometimes there are multiple permissible epistemic positions to take towards a proposition. And that’s enough, together with the irrationality of waffling, to grant some initial motivation to the idea of conservative diachronic constraints.

            Not really. To get permissive Bayesianism, you need a case where laymen will be interpersonal but not intrapersonal permissivists (where all agents are stipulated to be ideally rational and have the same evidence). It’s not enough that sometimes they object to waffling while sometimes they allow that reasonable people can differ: you need the cases to coincide. In cases where laymen will clearly find waffling impermissible, like the urn, the explanation is that they have indifference intuitions which make them thoroughgoing impermissivists. In cases where laymen accept permissivism, like religious belief, it’s unclear both that they are imagining ideally rational agents at evidential parity and that they would find waffling objectionable. “Waffling” in the context of religious belief sounds a lot like having a conversion experience, which, I assume, few laymen will take exception to.

          • Philosophisticat says:

            @EarthlyKnight

            I was initially responding to your (rhetorical) question: “Why shouldn’t Bob decide to adopt Mary’s credences wholesale as his own, given that they are, by hypothesis, perfectly rational?” I responded by saying the fact that Bob thinks that Mary’s credences are inaccurate (or, I guess more precisely, Bob’s calculated expectation of their accuracy is low) seems like a perfectly good answer to that question.

            Then you responded by suggesting “It seems to me that any reason to think that Mary’s beliefs are inaccurate must equally be reason to think that her beliefs are irrational”. I pointed out that this was false to anyone but the most radical externalist. In defense, you pointed out that Bayesians like accuracy arguments. So I pointed out that Bayesian accuracy arguments aren’t committed to the kind of principle linking accuracy to rationality that you would need for the kind of parity argument you were going for.

            Now you’re asking something different and stranger – for a single feature that Mary has which rationally separates her from both the additivity violator and the conditionalization violator. But why should we think that the same thing separates her from both? The thing that separates her from the additivity violator is that the additivity violator is accuracy-dominated. The thing that separates her from the conditionalization violator is something else. (The failure to maximize their own expected accuracy, for example). I’m not totally sure what burden you’re trying to place on me here or what it has to do with the dialectic up to this point.

            Fair point about permissivism.

          • Chrysophylax says:

            @Earthly Knight, @Jiro:

            I had only come across the colour-change version. The arbitrary-colours version is also stupid. It requires that the way that an emerald interacts with light depend on when it is observed instead of electron energy levels. There is no specified mechanism for this. It also ignores the fact that you fundamentally cannot make a thing with the chemical composition and structure of emerald that isn’t green. This makes the physical laws vastly more complex and possibly requires acausality. The complexity penalty is suitably enormous.

          • Protagoras says:

            @Chrysophylax, It’s not even a matter of the chemical composition; emeralds are green by definition (a beryl is an emerald iff it’s sufficiently green).

          • Chrysophylax says:

            @Protagoras:

            Arguments about the definitions of categories aren’t relevant for the grue-bleen argument, since one could make a definition in terms of grue and bleen.

            The point I’m trying to make is that beryls with chromium and vanadium impurities will reflect light in the the 495-570 nanometre range as a consequence of quantum electrodynamics, so the proposed counterfactual where emeralds would appear blue if first observed at a different time is stupid because it requires enormously complex physical laws.

            The more general point is that the map is not the territory. “Green” is a high-level concept about electron energies, light wavelengths, opsins in human eyes (and the emission spectrum of Sol) and human neurology. It’s fundamentally stupid to get tied in knots about apparently-possible changes to surface features of the map because surface features of the map are built from deep regularities of the territory. If something is “apparently possible” but never actually happens, your model of what’s possible is wrong and your argument isn’t sound.

          • Chrysophylax says:

            @Earthly Knight, @Philosophisticat:

            I’ve been meaning to get involved in this conversation for a week and haven’t had time. Watch this space, I guess. EK, I think you’re rather confused about a lot of stuff.

          • Earthly Knight says:

            @ philosophisticat

            Let’s add one more case, Suzy, who began with Mary’s credence distribution but thereafter switched to Bob’s (thirty years ago, let’s say). Why should Bob think that Suzy is irrational? From Bob’s perspective, Suzy changing her credences to match his looks like Suzy swapping out an inaccurate (mentally insert the by-Bob’s-lights qualifier as you see fit) set of priors for a more accurate set. Shouldn’t we commend agents when they exchange an inaccurate set of beliefs for a synchronically rational and more accurate set, rather than criticizing them? It’s true, of course, that at some time t in her past, Suzy moved from a credence distribution which, prior to t, she considered more accurate, to a credence distribution which, prior to t, she considered less accurate. But why should Bob care that the (by-his-lights inaccurate) set of priors Suzy had thirty years ago condemned the change at the time?

            @ Chrysophylax says:

            There is no specified mechanism for this.

            This is just a feature of the choice of example. Feel free to replace “green”, “blue”, “grue”, “bleen”, and “emerald” with the predicates and entities of fundamental physics– say, “negative charge”, “positive charge”, “pegative charge”, “novative charge”, and “electron”, respectively– where there are no mechanisms to appeal to. Why do observations of negatively-charged electrons confirm the generalization that all electrons are negatively-charged rather than the generalization that all electrons are pegatively charged (where pegatively charged =df negatively charged if first observed before the year 2100, otherwise positively charged)?

          • Philosophisticat says:

            @EarthlyKnight

            Shouldn’t we commend agents when they exchange an inaccurate set of beliefs for a synchronically rational and more accurate set, rather than criticizing them?

            No, I don’t think so. How accurate someone’s credences are (or how accurate they are by someone else’s lights) is not something internal to them. So internalists, as most Bayesians are, will not commend agents for getting lucky by switching to more accurate credences when the agent’s own beliefs suggest they would be doing worse.

            Brian Hedden actually uses internalism to argue for the kind of pure synchronism you’re defending, (although I don’t think the argument works). If you haven’t read it (and your views are so close to his you may well have), his recently published book on the topic, Reasons Without Persons, is very good.

            There’s a second thing going on which is obscuring the issue, which is the fact that the relevant change in credences is so long ago. Some ways of understanding conditionalization entail that Suzy is irrational now, which, I agree, doesn’t sound right (Give her a break! It was thirty years ago!). For that reason I think we should understand conditionalization as describing the rationality of an agent in particular acts of updating and not the rationality of the agent for all times following those events. (So, on this account, Suzy was irrational when she switched credences thirty years ago – since then, she has not been irrational).

          • Earthly Knight says:

            How accurate someone’s credences are (or how accurate they are by someone else’s lights) is not something internal to them.

            I don’t think this is right– Suzy can know a priori how accurate someone with Bob’s credences will judge her credence distribution to be. But I also don’t think it matters. We are asking why Bob should think Suzy is irrational, and Bob can certainly judge how accurate Suzy’s beliefs are by his own lights. To him, it will look like Suzy is cleaning up an erroneous set of priors. And why should he care that an inaccurate-by-his-lights set of priors advised her not to switch? This may, strictly speaking, be an argument against being a permissivist rather than an argument against the truth of permissivism. But it seems highly counter-intuitive to me that Bob should judge someone to be irrational when her credences are (1) synchronically rational, (2) accurate-by-his-own-lights, and (3) accurate-by-her-own-present-lights, whose only fault is that she had to disavow some mistaken (albeit rational) beliefs to get where she is now.

            For that reason I think we should understand conditionalization as describing the rationality of an agent in particular acts of updating and not the rationality of the agent for all times following those events.

            I’m not sure this will work. Won’t a Bayesian want to say of Suzy that there is some time that she is being irrational? But, conceivably, the change from her initial credence distribution to Bob’s could occur in an instant. Or is it to be the first time-slice after the waffle that is irrational? Only the first time-slice? Or, if you’re thinking of it as a relation, what are the relata?

          • Philosophisticat says:

            @EarthlyKnight

            I’m not sure why you think that Bob judging Mary as inaccurate by his lights should bear on his judgments of her rationality (or on what really matters, which is the facts about her rationality). The idea that there’s something special about being inaccurate by your own lights is kind of the starting point for many Bayesians. I’ve run out of ways of expressing this, so if you still don’t see a plausibly rationally significant distinction between being inaccurate by ones’ own lights and being inaccurate by Bob’s lights, I don’t think I’ll be able to get you there.

            There are different answers you could give to the “when” question. One might say the agent is irrational at the first time slice. That doesn’t seem obviously terrible. But that’s not what I would say (if I were defending conditionalization). I would say that there’s no single time slice at which the agent is irrational – she’s irrational in virtue of the transition, which occurs over multiple time slices. Think Zeno’s paradox of the arrow. On this interpretation, conditionalization doesn’t tell us when an agent’s credences are irrational – it tells us when an agent’s transitions between credences are irrational.

            (See e.g. Abelard Podgorski’s “A Reply to the Synchronist”)

          • Chrysophylax says:

            @Earthly Knight:

            WRT induction, I think you’re confusing priors and updates, and also confusing labels with the things they describe, but I think your biggest problem is that you don’t seem to understand why Occam’s Razor is true.

            The grue hypothesis is *strictly more complex* than the green hypothesis. A hypothesis that says “X until t, Y after t” cannot be specified as easily as a hypothesis that says “X”. Making the second hypothesis “X until t, X after t” is an error because it draws a parallel to the wrong thing. The “X” hypothesis could equally be written “X until t’, X after t'”, or “X until t”, X after t””. It is thus analagous to the entire class of hypotheses “X until T, Y after T” for any T, and so is enormously more probable than any such hypothesis.

            It’s actually more probable even than that, because “X, Y” specifies *two* things; you need to give more information to fully specify the hypothesis. The “X” hypothesis is therefore more probable for the *same* reason that a conjunction is less likely than its component claims: you’re specifying a narrower region in the sample space.

            Observing green emeralds isn’t evidence for “G until t, G after t” over “G until t, B after t” (where t is in the future) because both hypotheses predict G. It *is* evidence against the general class of grue hypotheses because some Ts are in our past; a proportion of the grue hypotheses predict inexplicable violations of electrodynamics. Also, observing that water is wet is evidence against the grue hypotheses! It takes a lot of information to specify a world in which emeralds *and only emeralds* “could have been otherwise”. Observing that the world is extremely regular (regular enough for us to evolve!) is overwhelming evidence that the world runs on simple universal laws.

            Specifying that emeralds are grue before t and bleen after t is either an epiphenomenal complication in the map and thus meaningless, or else a claim that the territory *could* have contained B emeralds, which postulates enormously more complex physical laws.

            We don’t *update against* a particular grue hypothesis. We reason that either grue is a meaningless complication to our map that doesn’t correspond to anything in the territory, and thus should be ignored; or else it is a claim about what laws of phsyics might actually exist, in which case it recieves a ludicrously tiny prior probability. We aren’t doing an experiment with emeralds and finding out that grue is wrong; we’re noticing that grue and green *sound similar to a human*, but are actually very different hypotheses in terms of physical laws, such the only way grue can *describe the same territory* as green is if grue is meaningless. The argument isn’t anything to do with emeralds, it’s about reality being lawful and conjunctions being improbable.

            As for electrons, the environment does not contain any negative charges whatsoever. “Negative” exists in the map, not in the territory. An electron behaves as described by certain mathematical laws. The words we use to describe this are a degree of freedom; only the behaviour is constrained. This is exactly analagous to talking about “green” and “grue” instead of wavelengths and opsins.

            “where there are no mechanisms to appeal to”

            I don’t understand how you can possibly think this. If electrons have different properties depending upon when you look at them, *you have postulated extra laws of physics*. A thing is fundamental when it’s *invariant*.

          • Chrysophylax says:

            An important thing to clarify in this conversation is the *enormous* difference between a prior probability and a prior.

            A prior probability is a degree of credence before seeing (any or some particular) evidence; a prior captures not only this, but also how the probabilties would change in response to different pieces of hypothetical evidence. In other words, a prior probability tells us what we know, a prior tells us what we know and how well we learn.

            https://arbital.com/p/bayesian_prior/

          • Chrysophylax says:

            @ Earthly Knight, @ Philosophisticat:

            I also think we need to define what “rational” means. Rationality isn’t about things, it’s about processes; it’s meta-level, not object level.

            Instrumental rationality isn’t making good decisions, it’s using a process that causes you to *systematically* make good decisions. If I don’t buy a lottery ticket until I’m fifty, that doesn’t make me instrumentally rational if I then go out and buy lottery tickets on a whim, because past good decisions aren’t the same as a process that systematically outputs good decisions.

            Epistemic rationality isn’t having true beliefs, it’s using a process that causes you to *systematically* have true beliefs. If I see a hundred-sided die roll a 6 ten times in a row, I ought to think it’s biased; believing that the rolls are fair is irrational (even if it’s accurate!) without at least 20 decibels of evidence (a likelihood ratio of 10^20) in favour of fairness. Having correct beliefs isn’t the same as using a process that systematically makes your beliefs more accurate as you accumulate evidence.

            Reality is lawful. It has an invariant structure that can be discovered, and this structure prescribes which beliefs are true and which decisions are wise. Rationality is about using processes that reflect the structure of reality and systematically incorporate the structure better over time. This is why induction works: the structure of reality is fundamental and invariant, and so it’s learnable. An error in the map doesn’t change the territory; physics doesn’t care what you think.

        • “For the ideal case, as far as I understand it, you want something like a prior over Turing machines”

          I don’t follow that. Where does your prior come from? Bayes tells you how to modify your prior on the basis of new information, but what’s the basis for the initial prior?

          Your claim was a very strong one and I don’t think it’s true.

          • Chrysophylax says:

            First, I note again that you’re attacking a claim I didn’t actually make, as discussed above. Given *your* priors and the information available to you, the posteriors are determined.

            Second, how well do you understand Solomonoff induction? I recommend reading the Arbital page: it’s a really good intuitive explanation and thorough about the quibbles.

            There is (AFAIK) no perfect ideal prior because there is no perfect ideal UTM. Unless you attempt really obvious shenanigans, this doesn’t matter much because updates on an inductive prior will very rapidly promote good hypotheses, and because very powerful reasoners can be encoded in very short sequences. (Think about the busy Beaver numbers.) You can get arbitrary levels of precision with very little data given any reasonable prior.

            The idea that computational complexity should be penalised can’t be deduced, but we can deduce that some kind of complexity penalty is needed and induce that computational simplicity is the right measure. (Given Cromwell’s rule, finite probability mass, unbounded potential complications and a measure of simplicity such that there are only a finite number of hypotheses that simple, simpler hypotheses must be more probable than complicated ones in the limit.)

          • vV_Vv says:

            Second, how well do you understand Solomonoff induction? I recommend reading the Arbital page: it’s a really good intuitive explanation and thorough about the quibbles.

            Is this the new “Have you read the Sequences”?

            I went there and I found an overly long rambling dialogue with self-congratulatory remarks such as:
            “But it so happens that Yudkowsky did invent or reinvent that argument after pondering Solomonoff induction, and if it predates him (or Solomonoff) then Yudkowsky doesn’t know the source. Concrete inspiration for simplifiable arguments is also a credit to a theory, especially if the simplifiable argument didn’t exist before that.”
            when discussing something amounting to the very well known fact that probability mass functions with infinite support must be decreasing w.r.t. some function (with finite preimage for each value) of their support, as it was something specific to Solomonoff induction or its discovery was such a great insight by Yudkowsky.

            As a rule of thumb, I suggest not to rely on Yudkowsky for your technical knowledge on this topic (and others). Try Scholarpedia, for instance.

      • The Nybbler says:

        There’s a lot RIGHT about the article too. It’s not so much that the author thinks rationality is just another belief system; it’s that he points out that those who would make it the basis of governance would make it another belief system. The Tower of Babel is fictional; the French Revolution, however, is not. The best bit, I think, is this (with partisan attacks elided)

        It takes a healthy dose of self-awareness to admit that rational, sane people could come to different conclusions about the world than you have. […] But it gets scary when […] in power try to weaponize their lack of empathy by outlawing dissent. Science itself becomes an inquisitorial instrument, meaning it ceases to be science and becomes a political cudgel.

        That’s the most likely outcome of trying to run a country based on “science”; “science” becomes whatever those in power want it to be.

        • vV_Vv says:

          Indeed communism was initially marketed as “scientific socialism”. It ended up with geneticists sent to gulags for disagreeing with Lysenko.

      • Samedi says:

        I agree. The one interesting point in the article is buried by a religiously motivated diatribe. I saw the tl;dr as, rationalism and those associated with it are anti-religious and therefore bad.

        The author also apparently has no sense of irony. A religious person concerned about science being “weaponized” for policy reasons, as if religion had never been used for that purpose. A more reflective and intelligent article might have considered policies that allow neither to be used in that way.

    • MichaelM says:

      The problem with guys like Tyson isn’t that they’re into rationality, it’s that they’re not. A commitment to Reason, the need to give reasons for beliefs, brings with it a deeply embedded understanding of the limitations of knowledge. This includes both the soft limitation of things we just don’t know right now and the hard limitation of things we really can’t ever know.

      Men like Neil DeGrasse Tyson, regardless of how they might feel in private, do a really good job in public of looking like they’re really, really sure about everything they believe. That puts an awful face on the science they popularize, like it’s a method of discovering truth (and a surprisingly easy method! For all the valorization of the labors of geniuses in modern popular science, they still ultimately make it all seem so easy!), rather than one for laboriously and incompletely stripping away falsehood.

      And that comes back to the critique of their ignorance of philosophy. I’m not so sure how familiar Mr Morris is with philosophy, but the place of philosophy in a rational life really is central, because philosophy is really the practice of reasoning. Guys like Mr Tyson just don’t seem to be using reason very often in a public manner and it shows when they denigrate philosophy. By denigrating philosophy, they’re denigrating rationality, simple as that. Tyson doesn’t want to live in Rationalia, he wants to live in Tysonia.

      • Mongeese says:

        Another offender in that category would be Lawrence Krauss. And it’s starting to become clear to me that the whole Philosophy is Great vs. Philosophy is Useless debate is a really tired game. It’s as if
        1) Analytic philosophy is somewhere on a continuous spectrum of rigorous thought in general.
        2) Philosophy as it’s mostly practiced in academia is almost completely useless.
        3) Point (1) is painfully obvious to Massimo Pigliucci, philosophers generally, and most rationalist-minded people.
        4) Point (2) is painfully obvious to people like Tyson, Krauss, Weinberg, Yudkowsky, many rationalists, and most normal people.

        Then the popular scientists say philosophy is useless, which most of it is, and the philosophers interpret that as saying that the only kind of rigorous thinking is quantitative or directly empirical, words words words, etc.

        Also Tyson seems to be in the habit of making confident statements well outside his expertise. Alas.

        • Samedi says:

          I don’t think the topic is tired but I do wish it were argued better. It could provide a context for discussing the way our understanding of the world has changed. I would argue, for example, that our view of the world has changed to be fundamentally probabilistic. We no longer seek deductive, true or false statements, instead we use tools like statistics to identify probabilities. A world of contingent probabilities, discoverable only after lots of trial and error by many people over time, is not compatible with the classical philosophical program at least as I understand it.

          I think the discussion of intellectual dead-ends is important. It isn’t a question of whether philosophy is “useless” or not but how has our understanding has changed in such a way as to render previous assumptions untenable. The same applies to the social sciences which appear to be in something of a crisis themselves.

      • Gbdub says:

        At this point I think Tyson is actively damaging science. Most of his schtick these days is spouting off on policy – he’s basically a political pundit – and asserting with authority facts well outside his area of expertise. As if an out of context single number in a tweet constitutes the sort of “evidence” a real rationalist would run a country on.

        That’s what people want to believe science is – hard facts easily digestible into unchallengeable truths with obvious implications. Precisely because this fantasy is so tempting, our popular “science communicators” ought to be pushing back against it. Instead Tyson, Nye, etc. just seem to be basking in the hero worship.

        • Hyzenthlay says:

          That’s what people want to believe science is – hard facts easily digestible into unchallengeable truths with obvious implications. Precisely because this fantasy is so tempting, our popular “science communicators” ought to be pushing back against it. Instead Tyson, Nye, etc. just seem to be basking in the hero worship.

          I don’t have much to add but I agree wholeheartedly with this statement, and I find this state of affairs very frustrating.

          Someone who’s very committed to being rational and examining their own biases will spend a lot of time doubting and questioning. Whereas Tyson and Co. project a smug self-assurance that they have all the answers and that anyone who questions their stance is obviously stupid.

          It’s easy to give lip service to “reason” and “science” and “evidence” and then act as if all your positions are backed by those things. It’s a lot harder to walk the walk and provide actual reasoning and evidence.

    • Ryan Beren says:

      He tweeted “virtual country”, not a real one.

    • Scott Alexander says:

      Insofar as I have any ability to speak for the rationalist movement, let it be known that we don’t support Neil deGrasse Tyson in this and that like the first rule of actually starting to be rational is not to put too much trust in your own inside-view rationality.

    • J Mann says:

      IMHO, it says a lot about someone whether their science hero from the following list is:

      1) Richard Feynman

      2) Neil DeGrasse Tyson, or

      3) Bill Nye.

      If (2), it’s possible that they just suffer from presentism and education would help. If (3), I don’t know what to say.

      • Ninmesara says:

        What about (1)?

        • null says:

          It means you’re part of the cargo cult, obviously. The last real scientist is the guy who drank H. pylori to prove that it is a cause of ulcers.

          Less flippantly, (1) is the ‘right’ answer, and there are very compelling reasons why this is so, such as the way Feynman would talk about science. There are all sorts of famous anecdotes about this.

          One of the more compelling ones, although this one is more about science education:

          After a lot of investigation, I finally figured out that the students had memorized everything, but they didn’t know what anything meant. When they heard “light that is reflected from a medium with an index,” they didn’t know that it meant a material such as water. They didn’t know that the “direction of the light” is the direction in which you see something when you’re looking at it, and so on. Everything was entirely memorized, yet nothing had been translated into meaningful words. So if I asked, “What is Brewster’s Angle?” I’m going into the computer with the right keywords. But if I say, “Look at the water,” nothing happens – they don’t have anything under “Look at the water”!

          • null says:

            I was not aware of this, thank you.

          • Ninmesara says:

            I don’t understand… Choosing (1) is being part of the cargo cult? Is his some kind of joke I don’t get or is it just a way of saying that having any idols at all instead pf running the experiments ourselves is being part of the cargo cult?

          • null says:

            It’s a joke, given some people on this site’s attitudes towards the process of academic science. Sorry if that was unclear.

          • J Mann says:

            “Cargo cult science” is one of Feynman’s most famous ideas, and well worth reading.

          • J Mann says:

            Exactly – people who have Feynman as a hero are IMHO much more likely to have a healthy attitude about science.

            People who have DeGrasse Tyson as a hero might like looking at space pictures and enjoy being smarter than other people, which is OK but not as good as the first option.

            People who have Bill Nye as a hero are marking tribal status points, and probably in a cargo cult to boot.

          • gbdub says:

            It kind of depends on which Neil deGrasse Tyson you claim as your hero. The astrophysicist who shows cool pictures of space? Totally fine. The basically-retired-from-science pundit who tweets out of context statistics about gun control, with the authority of a Scientist(TM)? Not so much.

          • Desertopa says:

            I’ve heard the quote a bunch of times before, so the basic concept is old hat to me, but what jumps out at me on this rereading is his use of the phrase “I’m going into the computer with the right keywords.” It’s kind of fascinating to me to think how he’d absorbed the practical elements of computing into his conceptual lexicon, when he was born in 1918.

  6. Mazirian says:

    I was expecting you to link to this study. It’s right in your bailiwick. It shows that grit, intelligence mindset and similar character traits share a common core which is a mixture of Big Five conscientiousness and openness and is essentially genetic. Moreover, only that shared genetic component in them predicts academic achievement.

    • Nicholas says:

      How can Openness be essentially genetic, when we’re pretty sure that good strong brain scrubbings with hallucinogens can increase Openness pretty much indefinitely?

      • Pete says:

        That’s the surprising thing about “good strong brain scrubbings with hallucinogens”, since they manage to affect a mental change that, as far as we know, is not possible otherwise.

        We can think of Openness as essentially genetic because (a) it’s hereditary and (b) it’s stable in adults – AFAIK, it’s been shown not to change at all in absence of trauma or mind-altering chemicals (including many hallucinogens but not most of the popular drugs), so most adults have the same measure of expressed Openness as their genetics prescribe.

        • Peter says:

          Key point – good strong brain scrubbings with hallucinogens are rare – therefore, the amount of variation in Openness out there in the wild due to brain scrubbings is pretty low.

          If something is highly heritable (i.e. “genetic”), then what that tells you is that there are no environmental sources of variation that are both common[1] and strong in your sample. It tells you nothing about other times and other places, next to nothing about super-rare things that could conceivably be made common, and little about things you could do to change the heritable thing, beyond “don’t bother looking at environmental variations that are common in your sample”.

          [1] as in, 50% common, rather than 100% common; if something happens to 99% of people then the interesting source of variation is the absence of it that affect 1% of people.

        • wintermute92 says:

          “Good strong brain scrubbings with hallucinogens” genuinely seem to be one of the most interesting medical events out there. Not because they’re so useful (though they may be) but because they keep turning out to be bizarrely potent. Like, as-influential-as-a-railroad-spike-in-the-brain potent.

          It’s the sort of issue I’d love to see more psychopharmacology people poke at, because damn does it bury the effect sizes of just about anything else known.

      • How can two leggedness be essentially genetic when a good chain sawing can remove a leg.

      • Randy M says:

        How can something depend on the chemicals our brains are built out of, when putting other chemicals into those brains can change them? Is that a serious question?

    • LPSP says:

      >Big Five
      Well, it tried. 😐

      At least it didn’t use Myers-Brigg.

      • Mazirian says:

        What’s wrong with the Big Five? One of the points of the study was to relate these new-fangled character traits to the good old Big Five to demonstrate that there’s little that’s new in them.

        • LPSP says:

          Openness as an axis (I used to be a fan of it, but I’ve come to see its weakness as a measure), the labelling of Intro vs. Extroversion (the axis is measuring anxiety, and anxiety negativey correlates with socialisation, but you can be unanxious and unsocial; the entire dichotomy is misleading, given that the axis is actually measuring something, and it leads to confusion regarding Neuroticism) and Conscientiousness (some people are picky about what they’ll work hard for and others aren’t, this again does correlate with overall level of hard-work/contribution but the terminology is misleading).

          Agreeability is sound, and Neuroticism is only limited by confusion generated from the Intro/Extro axis. Big 5 is also probably better than any test that tries to use “grit” as a measure. It’s still broken in easily-solved ways, and only remains in its current state out of inertia.

          • Mazirian says:

            Any references, especially to those easy solutions?

          • LPSP says:

            I outlined most of them. Logic is the reference. In summary:

            Replace Introversion vs Extraversion with Anxiety, stress the difference between this and Neuroticism; this resolves the issue of people who happily socialise but don’t go out of their way to do so, who would be classed as introverted by habit but extroverted by nature, by removing the distracting “how often” part with a relevant “how does it make you feel” question.

            Replace Conscientiousness with Community, defined as “willingness to work hard/contribute because of expectation to work hard/contribute”, which resolves the issue of people who work diligently or intensely at certain tasks but don’t feel a sense of duty, who could be classified as high and low Conscientiousness from different perspectives.

            Replace “Openness vs. Closedness” with “Experience-Seeking”, stress the importance of going out of one’s way to partake in different as opposed to abstract acceptance (this I admittedly didn’t outline).

            Preserve Agreeability, which is perfectly fine.

          • Jill says:

            LPSP, that makes a ton of sense. Thanks for your ideas there.

          • LPSP says:

            You’re welcome Jill.

          • Creutzer says:

            It makes intuitive sense that introversion measures something like anxiety, but I’m a bit skeptical. You can make an argument that the measure conflates two things, because introvert behaviour can stem either from inclination or from anxiety. But if you just relabel it anxiety, you’ve still got the very same problem.

          • LPSP says:

            I’d qualify that with “it makes intuitive sense that, *for tests that concern emotional factors*, introversion measures anxiety”, because that is the case. Some B5 tests do this, while others to varying or total extents instead measure end actions – they ask “do you go out frequently” in addition to or instead of feeling queries. The latter totally confounds anxiety and inclination as you so aptly put it. By making the question just about the emotional state and not about the final product, the issue is resolved – an anxious person may still socialise frequently, as a matter of necessity or because of a high Communalism score (they have a sense of duty to act).

          • Creutzer says:

            I see. But don’t you think the inclination is worth measuring in itself? Wouldn’t it then be better to try to tease out anxiety from introversion and put it with neuroticism to have the intro-/extraversion axis measure inclination alone?

            I wonder how all of this relates to the PCA results pertaining to the Big Five…

          • I’m wondering whether anxiety correlates with introversion in a society where extroversion is considered very normal. Perhaps there’s also some anxiety associated with extroversion (discomfort with being alone) but it’s not as visible.

          • Mazirian says:

            Few things are easier than making convincing-sounding arguments in the absence of data.

          • LPSP says:

            Creutzer: As I’m seeing it Neuroticism and Anxiety are different things. Neuroticism is a compulsive need to address or act on issues one percieves as pertinent, and with that perhaps a tendency towards high pertinence or “getting worked up”. Anxiety is a restraining fear, that one’s actions will be unacceptable or result in ostracisation.

            The inclination to socialise itself, besides the factors in the revised model, is basically circumstantial. If you live close to your friends, you will socialise frequently; far from any friends, not at all.

            Nancy: I wouldn’t be able to say on the society part, but I’d definitely agree that anxiety can lead to, or intensify, extroverted behaviour. To an extent this may reach the conceptualisation of Community as an axis; similar yet inverted to Anxiety, Community measures a feeling of a need to act in order to play a needed part or fulfill some duty. Someone high in both Anxiety and Community would be constantly socially active, and always within an acceptable norm – they must act, and they mustn’t act wrong.

            Mazirian: The data’s the same for B5. It’s just interpreted differently.

          • Creutzer says:

            The inclination to socialise itself, besides the factors in the revised model, is basically circumstantial.

            On the basis of my life experience, I am very disinclined to believe this; in particular, the idea that variation in socialisation is caused only by circumstances and anxiety does not explain how people seem to differ quite starkly in their need and desire for socialisation.

            So: What’s your evidence?

          • Mazirian says:

            LPSP, then show me how your model fits the data better than the Big 5.

          • LPSP says:

            Creutzer: Of course, circumstances and anxiety do not explain the difference entirely. Circumstances, anxiety andthe need to perform expected social roles – Community – explain the difference entirely. You seem to have missed the other factors of the revised model when their relevant was indicated.

            Mazirian: I’ve given examples, and I repeat those plus a few more:
            – instances of individuals who do not socialise frequently, but have no issues with socialisation. Introvertion in the Big 5 states a 1:1 relationship between inclination towards/comfort with socialisation, and actually socialising. Differences are explained away with the concept of Neuroticism, but that complicates things further because:
            – Neuroticism bundles passive fears and worries with a propensity to get agitated or hooked on certain issues. There are individuals who do one but not the other and vice versa.
            – the fact that individual’s level of socialisation is influenced by ease of availability to friend circles. Consider the situation of someone who lives far from friends and family, but who flies out once a year to one or more large meet-ups and has a perfectly relaxed experience, and who would happily experience more if they didn’t need to return. Now consider someone who is constantly active socially, but only because they share a room/hall of residence/neighbourhood with work peers, something beyond their control, who genuinely likes their peers but wouldn’t socialise so frequently if they felt they had a choice. They socialise less than their peers, but still socialise far more than the flight-out-once-a-year case, yet they constantly panic over how they must socialise and what not to do while interact with their closest friends, a trait Big 5 labels “introverted”. This is probably the single biggest hole in the Big 5 after Openness to Experience.
            – And of course, all of Openness to Experience, for which my criticisms are not unique or special. I would just copy-paste them from elsewhere.

            Ultimately, Big 5 makes large assumptions that assume 1:1 correlations between certain behaviours and personality types, and assume absolute causations. These can be simply corrected.

          • Mazirian says:

            The Big Five does not assume perfect correlations between any items and their associated personality traits. The B5 is operationalized factor-analytically and the basic assumption of factor analysis is that all measurement is error-prone and that single items that perfectly capture an underlying trait cannot exist. All items contain variance that is unrelated to the trait. In factor analysis, traits are operationalized by equating them with the shared variance between multiple items.

            The Big Five doesn’t fit all individuals. This is a general property of all individual differences constructs. The Big Five is not meant to describe individuals but to give a general picture of differences between individuals.

  7. Thursday says:

    I said the Cherokee study was anomalous when it first came up. Best bets are that it doesn’t really mean anything, though more study is always good.

  8. expjpi says:

    I got paywalled at the “fired for doing a study” link.

      • Jiro says:

        Other articles easily found with Google indicate that there were concerns over ethics violations. And it’s not clear whether those are being used as an excuse ot the university would actually treat similar violations about other subjects this way; and the latter is not totally implausible considering that ethics boards can be strict.

        • linker says:

          Scott’s article also mentions the accusations of ethics violations, but only your article mentions that the procedure was approved ahead of time.

          This is really a story about academic independence.

    • Siah Sargus says:

      The past was almost always wrong about the future. Surely we will be almost always right? Personally, I think there will be a bit more automation, before we have finished picking all of the low hanging fruit, and will require more sophistication. I’m pretty confident that diverless cars will become a luxury item within the next ten years, anything beyond that is a pure guess. (To me at least.)

      • Matthias says:

        Driverless cars will either be a mass product or won’t happen. The economics just don’t support a luxury-market-only.

        The big expense is in developing the software and fighting the legal battles. Those are all fixed costs. The variable costs per car, sensors plus some computers, are miniscule by comparison. Especially if you think about Uber replacing costly human labour in a that’s used all day, and not just a commuter using the car perhaps an hour every day.

  9. Charlie says:

    On the survey about economists and basic income – the basic income plan in the survey was pretty dumb. Giving money to everyone over 21 but nobody under, without changing the tax structure. So, naturally, a lot of people marked ‘Disagree.’ So I’m doubtful that this is an in-depth exploration of their feelings about basic income guarantee plans in general.

    • Anonymous says:

      What’s dumb about that?

      • Virbie says:

        I can’t speak for Charlie and don’t know enough about the theoretical nitty-gritty of basic income to say why or whether that particular policy would be dumb, but he’s right in that the survey doesn’t support Scott’s claim that “economists are pessimistic about basic income”.

        The polled economists disagree on average with the specific plan described in the question, namely:

        Granting every American citizen over 21-years old a universal basic income of $13,000 a year — financed by eliminating all transfer programs (including Social Security, Medicare, Medicaid, housing subsidies, household welfare payments, and farm and corporate subsidies) — would be a better policy than the status quo.

        If you read the (optional) comment sections on many of the responses, many of them say stuff like “$13k isn’t enough given that you’re removing all these other programs”, “X proposes a variant which is better”, “21 is a silly cut-off”, “I support UBI but Medicare should remain”, etc. There’s even a guy who mentions that it couldn’t work dropped in to the current system: universal healthcare and tax reform would be necessary (I have literally never talked to a serious proponent of UBI that thinks we could just drop it in wholesale to the current system without at the very least, tax changes). Those people quite clearly aren’t disagreeing with the concept of basic income per se. Those who are less sanguine about UBI in general are of course represented in the sample as well.

        I’m actually kind of surprised that Scott chose to phrase it simply as pessimism about basic income when it seems to imply a decidedly mixed response.

        • Wrong Species says:

          Remember the days when basic income proponents tried to convince us that it would actually save money by replacing these other programs?

        • Yes basic income instead of medical assistance is a bad idea. No it’s not the version of BI most BIwrs have in !mind.

        • It’s true that the response was to a particular proposal. But that proposal was presumably designed to make the numbers balance, to cost the same amount it saved.

          If you extend the basic income to younger people or keep some parts of the existing system it no longer balances, and you have to reduce the amount of the demogrant or increase taxes or change something else–which will create new reasons for people to oppose it. The fact that lots of people would be in favor of a basic income if only it were funded from a large pot of free money isn’t very relevant.

          • Virbie says:

            > If you extend the basic income to younger people or keep some parts of the existing system it no longer balances, and you have to reduce the amount of the demogrant or increase taxes or change something else–which will create new reasons for people to oppose it.

            …. And create new reasons for people to support it, as is made evident by the fact that multiple economists in the poll disagreed because of the necessity of tax changes, disagreement with the cutoff age, or the amount being too small. The fact that different proposals create new reasons to both support and oppose it is…. Pretty much basically the definition of “different proposal”. which is precisely why it’s silly to treat negative reactions to this specific proposal as economists disagreeing with UBI. In particular when many of those disagreeing are doing so in explicit support of UBI as a concept.

            This is hardly a novel concept I’m describing: it’s like looking at all the single-payer proponents and their opposition to the ACA as not doing enough, and saying “Behold, all these people are against more government action in healthcare”.

            > The fact that lots of people would be in favor of a basic income if only it were funded from a large pot of free money isn’t very relevant.

            This is a whopper of a strawman. There are more alternatives to a given specific, fully-determined UBI proposal than 1) funding it in a way that’s so vile that it definitely isn’t feasible and 2) free money pots.

            Here’s a blindingly obvious issue in the program: the lack of tax changes means that households with an annual income from (say) $100k to $infinity are getting a net benefit of $13k. I’m not aware or any UBI proponent who thinks that makes any sense. Logistically, they’d be getting the UBI payment but it seems like common sense to change the tax structure so that their net benefit is (at most) $0. Even with this simplistic change, we’re in a situation where costs have dropped by 20% with no real change to what proponents want out of a UBI.

            I’m obviously not suggesting that particular change (especially not with a cutoff cliff), but the general point stands: treating a proposal as if it’s representative when no proponent would take it seriously is just dishonest.

          • Cadie says:

            Simplifying the tax code to remove a lot of loopholes and deductions that primarily go to wealthier households would definitely offset some of their UBI benefit. I’m not sure if that can be done without increasing the tax percentages themselves – I don’t know enough about economics and taxes to figure that out, to be honest – but if there are fewer deductions they can take and they end up paying a larger percentage of their tax rate than they would have before the change, obviously they’re paying more tax. The trick is to balance it out so that most households that are doing okay end up paying extra tax somewhere around the UBI payout, and wealthier households pay more than that in extra tax, so it’s only the poorer households that actually get more under the system. There’s still a benefit for everyone richer than that, though; since they never lose the UBI, they can take risks like opening their own business or working less for awhile to advance their education without fear that they’ll lose everything and end up penniless. The floor is being fairly poor, not being homeless and not having enough to eat.

            Figuring out the details so it could work is the very hard part.

        • Loquat says:

          13k a year per person to replace both Social Security AND Medicare? The AARP would have your head for that alone.

      • Charlie says:

        Think about the change relative to the status quo. There’s no changes to taxes, you just replace a bunch of government safety nets with direct payments to everyone over 21.

        There are three groups of people who will be impacted by this plan: Those who are currently using government safety nets and are over 21, those who are currently using government safety nets and are under 21, and people over 21 who aren’t currently using a government safety net.

        The largest group is the third – people who will just gain an extra $13k relative to now. Is this a good use of government money? On average, no – it’s not a particularly good use of money to give it to people like me, no questions asked. That money should probably be used to pay people to repair roads or lay fiber-optic cable or something.

        The people who are currently using government safety nets and will get $13k might be better off or might not be, it’s a tough question. I think it’s pretty plausible they’ll be worse off on average, both because less total resources will be spent on them, and because I subscribe to a sort of vague paternalism that says that there is a policy-relevant fraction of people who suck at managing their own money and need a safety net managed by others.

        And, of course, the people who are currently using government safety nets and are under 21 will get shafted.

        • Julie K says:

          And, of course, the people who are currently using government safety nets and are under 21 will get shafted.

          Yup. Currently, a lot of means-tested benefits go to single-parent families. I can’t see us saying “Sorry, from now on your family will only get $13,000, full stop, because we’re going to start giving money to people who aren’t needy.”

        • Winter Shaker says:

          there is a policy-relevant fraction of people

          That is an excellent turn of phrase which I hope to be able to remember.

    • suntzuanime says:

      Yeah I’m a huge National Income fan but I would mark Disagree there because cutting off Medicaid/Medicare to pay for it will leave poor people who can’t get cheap health insurance to die.

      • Anonymous says:

        It turns out you don’t actually drop dead the second you aren’t covered by health insurance.

        • Nonetheless., Suntzu’s comment makes sense under a charitable reading.

        • sweeneyrod says:

          It turns out that not having access to medical care causes people to die. I welcome you to attempt to disprove that.

          • Ptoliporthos says:

            In the U.S., insurance (Medicare is a government run insurance program) is not medical care. If you walk into an ER dying of something you’ll get care, but you may incur bills you cannot pay without insurance.

          • Lumifer says:

            @ Ptoliporthos

            One of the consequences is that if you’re really poor (“judgment-proof” in legalese), you can walk into ER and demand care anytime you feel like it. Nothing bad will happen to you.

          • keranih says:

            I don’t think that the examples of the ER, etc disprove sweeneyrod’s point – without population based interventions like vaccines and sanitation, a lot of people would die sooner than they would if they had those things.

            Likewise, without trauma wards and antibiotics and insulin and radiation and chemo and the like, a smaller (but non-trival) number of people would die sooner rather than later.

            Sweeneyrod’s (and suntz’s) error is in thinking that not having insurance (or Medicare/Medicaid, etc) = not having health care.

            The Oregon medicaid experiment showed that the morbidity/mortality difference between getting insurance and not is extremely subtle.

          • suntzuanime says:

            You’re right, I was careless. We could have the poor and elderly skip out on their emergency room bills. This is still not a great way to organize society, if we can avoid it, but better than having them dying in the streets.

          • John Schilling says:

            It turns out that not having access to medical care causes people to die.

            To a first approximation, heart disease, cancer, and old age cause people to die. Medical care causes them to die a few years later, but be careful what you ask for. And, as others have noted, medical insurance and medical care are two different things; even the uninsured can die by very slow decay.

            The second-order effects and the fringe cases are where all the interesting and sometimes useful stuff happens. But if you’re going to reduce the discussion to a mindless one-line slogan, then get it right: life causes death, medical care doesn’t help with that.

          • Edward Scizorhands says:

            1. It’s really hard to show that American medical care, taking out a few specific examples, extends lives. If we were to do QALY measurements we would have good evidence it does, but whoever proposes this gets attacked by the other party.

            2. ER care is a really small part of the nation’s health care bill. It would be nice if it were a big part, because we could attack it, but it’s just not where a lot of costs add up.

      • Is the version you are in favor of one with an even smaller demogrant, one funded by a large increase in taxes, or one combined with a reduction in some other programs the size of Medicaid/Medicare? The money has to come from somewhere.

        • suntzuanime says:

          One funded by a large increase in taxes. If the middle+ incomes don’t end up with at least $13,000 in extra taxes the plan is actually redistribution-negative. I favor a redistribution-positive version, but at the very least we should try to be redistribution-neutral.

          Of course I’d love to fund a National Income by reducing some other programs. If we could give the money we spend bombing foreigners to our citizens instead, I’d be all for it. But there’s a limit to how wildly unrealistic I’m willing to make my dreams, so taxes it is.

    • Lysenko says:

      I am a huge proponent of Guaranteed Income in the form of a NIT on general principles, and somewhat in favor of a good UBI scheme if I ever see one proposed (one of the reasons I don’t make a very good libertarian). That said, I’ve long since filed it under “Will never be implemented without a revolution or similar disastrous upheaval of the status quo”.

      As already noted, any attempt to implement it in the US without massive tax and welfare reform (simply “axe all the programs in five years” is as bad as “don’t change any of them”) is going to be a shitshow. At best it adds another layer of complexity with modest benefits, like the last time NIT was proposed and we got the EITC instead. At worst, I see all sorts of new perverse incentives created and bold new opportunities in creating entire social classes utterly dependent upon careerist public sector employees for their survival.

      • Samedi says:

        The problem with UBI that is that its proponents ignore, as even intelligent people often do, the second-order effects. It’s naive to think that UBI wouldn’t be subject to massive political manipulation. Its proponents see only the benefits and not the impact of turning a large portion of the population into UBI serfs. What a near perfect instrument of control.

        • John Schilling says:

          You will, if you care to look, find no shortage of UBI proponents addressing those issues. But this,

          …turning a large portion of the population into UBI serfs. What a near perfect instrument of control.

          makes no sense at all. A UBI, by virtue of its universality, is the least serfdom-like means possible of providing for the indigent, and affords the least control over them. A recipient of classical welfare very nearly is a serf, bound to the land of a particular state or county and barred from taking any other employment, on penalty of having their aid cut off. Almost every alternative that has been proposed, has the explicit “feature” of demanding that its beneficiaries live in the particular manner their benefactors deem most virtuous.

          Advocacy of a UBI is driven largely by a desire to avoid this perversity. And while you are correct that political forces will seek to corrupt it, a system that is designed to afford maximum individual liberty but might be corrupted in the general direction of serfdom, seems preferable to alternatives that essentially are serfdom from the outset.

          Really, what else have you got and what makes that alternative less serf-like or less subject to political corruption?

          • Samedi says:

            To respond to your comment that calling UBI a form of serfdom “makes no sense at all”, I would say simply that no matter how clever the system you might try to put in place there will inevitability be strings attached. The fear of having their UBI cut will be enough to keep people in line.

            Even if you state in the UBI law that it cannot be cut for any reason it will still happen. There is plenty of precedent in the US for this (e.g. the 1st Amendment, “Congress shall make no law” followed soon by the Alien and Sedition act). Exceptions will emerge over time, complex rules will grow up around it, a large bureaucracy will form, and a simple idea intended to help people will turn into something else.

            As for alternatives, I don’t know that one is needed. If it were, I would look beyond the simplistic “pass a law = problem solved” mentality. I would favor an approach that made people more independent and self-sufficient rather than highly dependant.

          • John Schilling says:

            I would say simply that no matter how clever the system you might try to put in place there will inevitability be strings attached.

            I agree. But this will also happen in every other possible welfare system or social safety net, and it seems pretty clear to me that there is much more latitude for it happening in non-UBI systems than would be the case with a UBI.

            As for alternatives, I don’t know that one is needed.

            Here’s one useful alternative: don’t talk about “turning a large portion of the population into UBI serfs”. That is only a meaningful concept if there is a section of the population that would not be serfs under other welfare systems but would be made uniquely serfs under a UBI.

            The concept of a UBI, while unlikely to be implemented in practice, is a legitimate and sincere attempt to reduce the quasi-serfdom inherent in the concept of social welfare. It is wrong of you to describe it as a cause of serfdom, simply because it would be imperfect at eliminating serfdom. And you haven’t offered anything beyond that.

  10. I have been doing intermittent fasting for around five years where, ideally, for six days a week I restrict my consumption of carbs and protein to a six hour window and often drink coffee with butter in it outside of this window. The most remarkable effect is that I now rarely get sick. My wife and I are both professors and we have an elementary school child so we get exposed to lots of sick people, and before I did intermittent fasting I would usually get a cold if either my wife or son did, but now this rarely happens. And when I do get a cold I get over it much quicker than I did before.

    • gwern says:

      Intermittent fasting is no better than just dieting the normal way. I hate to gloat, but this concludes an almost ten-year argument I’ve been having with an acquaintance who said that the failure of doctors to immediately endorse intermittent fasting proves that the medical profession are all quacks who don’t care about their patients.

      I can’t endorse such inflammatory language, but I’m not sure that this is really dispositive. Isn’t it really encouraging that intermittent fasting so far looks at least as good as regular dieting? It certainly sounds like an easier lifestyle change.

      If anything is negative about this meta-analysis, it’s their note that the dropout rates don’t look very different between dieting & IF. This might reflect that IF is easier for some people and harder for other people, so someone who can’t diet normally may find IF doable instead, or it might be that standard caloric restriction is not as hardcore or difficult to do as people assume – I have been interested in how well the CALERIE CR trial has been going and how low their dropout rate has been:

      “A 2-Year Randomized Controlled Trial of Human Caloric Restriction: Feasibility and Effects on Predictors of Health Span and Longevity” http://biomedgerontology.oxfordjournals.org/content/70/9/1097.full Ravussin et al 2015
      “Effect of Calorie Restriction on Mood, Quality of Life, Sleep, and Sexual Function in Healthy Nonobese Adults: The CALERIE 2 Randomized Clinical Trial” http://www.gwern.net/docs/2016-martin.pdf Martin et al 2016

      • Fluttershy says:

        Very good points. If intermittent fasting is is about as good as regular dieting, you’re absolutely right that what we care about is dropout rates, and/or how much willpower is required to stick with one’s diet.

        I suspect that for some people, successfully doing either regular dieting or intermittent fasting will require spending willpower in different ways. Specifically, I’ve found intermittent fasting to be challenging to get used to initially, but effortless do after getting used to it. On the other hand, I’ve found regular dieting to be easier to start, but harder to maintain. Of course, YMMV, and all that.

        I’m tempted to mention that spending willpower on establishing habits is better than spending it on maintaining them, but you’d expect my appreciation for intermittent fasting to nudge me to say something like that in any case.

    • linker says:

      Did you try modifying your macronutrient ratios separately from intermittant fasting? How do you assign blame?

      • Yes when I drastically cut my sugar consumption. I’m not sure what you mean by the second sentence.

        • linker says:

          You imply that your lack of illness is due to IF. How do you know it is IF and not reduced carbs? If you reduced carbs SEPARATELY from when you switched to IF, you should know.

          • TPC says:

            I’m a nursing mother who can’t IF, but I do have to keep carbs pretty low for health reasons. I find the lower carbs are what reduce the illness tendency.

            The rebuttal is my weaned children who are carb fans due to being little kids who are in the top 5% for physical activeness.

            I get sick less when they do bring something home, but they get sick less than the kids they are surrounded by in social settings. I think it’s probably a third option– food quality. We eat extremely high quality food, usually very fresh and local. The kids and adults in our social circles who get sick all the time eat non-local food that is more shelf-stable and highly processed with lots of corn starch and other binding agents.

          • Interesting, you might be right. When I first tried intermittent fasting I tried to be very low carb but it didn’t work for me (I had low energy), but I still probably have far fewer carbs than I did before because I do consume little sugar.

    • Neanderthal From Mordor says:

      I never heard anyone saying that intermittent fasting leads to faster weight loss than other types of dieting, so this meta analysis refutes a statement no one made.
      The biggest IF claim is that fasting may improve health and prolong life, but there is no much evidence either way.
      I like IF because I would rather skip some meals and then eat a big meal rather than eating small portions (that I have to count calories for) that leave me feeling hungry. Because I also eat out a lot it’s also more convenient this way.
      So, for me IF is the best because it’s the only diet that I sticked with for long periods and compliance is the most important part of a diet.

      • Gbdub says:

        Same here. I did a version of IF (basically, eat only 1 modest meal ~500 calories every other day, eat normally other days) for about 2 months and dropped 20 lbs I’ve kept off pretty well since then.

        I’ve since stopped because I started a workout program and need more daytime calories to do that (and recover from it) but the nice thing was that the program was relatively easy to follow. Yeah, you got hungry during the day, but the fact that the deprivation was short term rather than a constant lack of satiety was very helpful. After a couple weeks I was fully adjusted and the fast days weren’t hard at all (I did add a light ~100 calorie snack at lunch time to really take the edge off).

        Perhaps the nicest long term effect was a better understanding of what “hungry” and “full” really feel like (and don’t feel like). Since then I’ve had an easier time not snacking when I’m just a little peckish, and stopping eating when I’m full.

  11. Watercressed says:

    But I worry that ignores the effect discussed here, where if the government had tried that in 1900 then by now the income would have grown to the amount the poor needed to support themselves in 1900, yet would still be way below the amount what we consider a minimum standard of living today.

    I think if this is implemented, there would soon be companies that provided room and board in exchange for 80% of the basic income, and one of the things these companies would do is lobby the government to ensure that they aren’t regulated out of existence in the name of increasing living standards.

    Which, y’know, isn’t great, but such is how the system works.

    • Scott Alexander says:

      Couldn’t you say the same about ordinary slums? Shouldn’t slums have a lobby to prevent being regulated out of existence in favor of increasing living standards?

      • Nicholas says:

        According to the National Housing Institute, they do.

      • Watercressed says:

        Two differences come to mind:
        -The specific level of a basic income is a much more defensible schelling point than general low-income housing.
        -The politicians in favor of basic income would have an incentive to make sure the system works well, giving the housing providers a sympathetic ear.

    • Pete says:

      Well, that scenario sounds just fine to me, why do you think that they should be regulated out of existence?

      I mean, unlike various current and historical housing programs and subsidies, in *this* one people aren’t tied to a specific provider and are able to vote with their wallets and feet; and this market doesn’t seem to show any strong advantages to monopolies above some minor economies of scale, so the market would likely be competitive and if another provider would provide better food and board for the same price, then they can just go there.

      • Watercressed says:

        I don’t think they should be regulated out of existence, but there is some potential for rent-seeking.

    • caryatis says:

      > there would soon be companies that provided room and board in exchange for 80% of the basic income

      What’s wrong with that? Lots of people out there can’t be trusted to provide for their own basic needs, even given sufficient money. Letting them contract up front to have a house and food all year without having to make pesky decisions about money seems like a great idea.

  12. Simon Penner says:

    Related: an alternative to universal basic income is the universal basic share, where the government says something like “We pledge to forever redistribute 10% of GDP, whatever that may be, among our citizens as a universal basic income”. The hope is that even if this starts out as not enough, as the economy grows it will gradually become more and more until it’s enough for people to live on. But I worry that ignores the effect discussed here, where if the government had tried that in 1900 then by now the income would have grown to the amount the poor needed to support themselves in 1900, yet would still be way below the amount what we consider a minimum standard of living today.

    I completely agree with your concern, but would like to use it to step up on an obnoxious soap box. Take a moment and think through the full implications of what you just said. There’s two interesting ones.

    The first: regardless of how we decide to redistribute our economic wealth, you will always be able to describe it in terms of fraction of the whole. If we said today that we are going to earmark $167bn for welfare spending, this is equivalent to saying “we will spend 1% of GDP on poverty”. If you were to say that we’ll redistribute 10% of GDP as an anti-poverty measure in a universal basic anything, and if that was still “way below the amount we consider a minimum standard of living today”, then there are exactly three options:

    1) Redistribute more than 10%, until you redistributed enough to achieve your goals
    2) Change the balance of the redistribution. Instead of making it universal, scope it to the needy in some way so the money is more effectively used
    3) Accept that the problem as you’ve defined it is insoluble.

    In a nutshell: If redistributing 10% of GDP, whatever it is, would play out like you say, this means that a basic income of that amount will never, ever, ever work.

    For my two cents, I think a basic income or negative income tax is probably a good idea. But, as you just established above, it will be substantially more modest than people think it is. It’s to a silver bullet

    —-

    The second is less important, and is more politically polarizing. Consider your standard socialist activist rhetoric. Corporations make profits immorally on the backs of the working man, and if the workers just controlled the means of production, we could make this world into a utopia.

    If you could make a political program redistribute 10% of US GDP in that fashion every year, you would have, definitionally, given workers control of 10% of the means of production. And yet, what does this get you? It gets you a reasonable standard of living by 1900 standards.

    Turns out ownership isn’t all it’s cracked up to be. All those profits aren’t as big as people think they are. But it gets worse.

    When you are an employee, you have a reasonable assurance of security/consistency/stability. Or, to go with the UBI example: when you get a UBI denominated in dollars, it’s like a paycheque. You know that you will consistently get $X per month. You can plan around that. X might be modest but you can rest easy, planning on that consistent income.

    When you’re an owner, you sacrifice stability/security for agency and ownership. The owner gets to call the shots. But the owner is also ultimately responsible for things. The owner holds all the risk. Just as the owner will make millions in a record year, the owner will also make _zero_ before the employees’ paycheques don’t get signed.

    Comparing this to a universal basic share: If we thought that, on the basis of common socialist reasoning, a UBS is better than a UBI, this has one massive problem: Everything is going to be really, really, really bad in a recession. All of the sudden, individual poor peoples’ welfare payments will fluctuate wildly depending on whether we’re in a boom or a bust. Most people find this much less enjoyable than the UBI alternative. Turns out, giving ownership of capital to workers, usually doesn’t help out workers, because it’s just not worth enough money

    • gwern says:

      Comparing this to a universal basic share: If we thought that, on the basis of common socialist reasoning, a UBS is better than a UBI, this has one massive problem: Everything is going to be really, really, really bad in a recession. All of the sudden, individual poor peoples’ welfare payments will fluctuate wildly depending on whether we’re in a boom or a bust.

      How’s that? Consider the recent Great Recession: “From peak to trough, US gross domestic product fell by 4.3 percent” http://www.federalreservehistory.org/Period/Essay/15 So the maximum fall in that UBS is also going to be 4.3%. That doesn’t strike me as ‘fluctuate wildly’. People weather much greater volatility in their incomes and welfare as it stands.

      • Rafe Saltman says:

        A share is the most residual claim on an organisation’s earnings and assets. Normally, the suppliers, employees, and other short-term creditors have to be paid first, then the indenture holders if there is any left over, then the debenture holders if there is any left over, then the taxman if there is any left over, then the preferred shareholders if there is any left over, and finally the common shareholders if there is any left over. Shares don’t get you a proportional slice of the pie. They get you the leftovers. If the pie shrinks, there won’t be leftovers.

        • suntzuanime says:

          You could just… not have it work like that. “Share” was being used in an informal sense, not by contrast to “debenture”.

          • Aapje says:

            You could just… not have it work like that.

            Taxing companies on profit that they don’t make results in bankruptcies.

          • gwern says:

            Taxing companies on profit that they don’t make results in bankruptcies.

            You should think about how taxation and GDP currently works.

          • Aapje says:

            > You should think about how taxation and GDP currently works.

            Businesses currently don’t have to pay a lot of taxation if they don’t make a profit and can actually use a loss in one year to reduce their taxes in another year.

        • Trevor Walker says:

          Not as defined here. 10% of GDP is 10% of GDP, period.

    • Jesse says:

      You could have it be 10% of the GDP from 3 years ago (roughly half of a recession to recession cycle) so the UBI would likely be near a peak in the downturn, and at the minimum during the economic peak…

    • brad says:

      It would make more sense if it was some (perhaps higher) fraction of GDP excluding government expenditures.

  13. Tsnom Eroc says:

    Well, as to the antidepressant study, it may interest you to know the same thing was repeated almost 15 years ago with the same results

    http://jannel.se/prozac.childrenEU.pdf

    “In December 2003, the British MHRA banned the use of all antidepressants for children and adolescents under 18 years…. only antidepressants that MHRA specifically excluded from the ban was Prozac”

    Interesting phrase as to why prozac was allowed

    “The profession know they are going to
    have to prescribe something …..”

    Now a quote from The Last Psychiatrist, as to what happens when all antidepressants are banned

    “All of this is true, but this isn’t psychiatry finally coming to its senses; this is psychiatry entering the manic phase. Sure, it’s less SSRIs for kids; but it’s more antipsychotics.”

    • Scott Alexander says:

      But I think that case wasn’t a study showing Prozac was better. It was just that Prozac was the only one the companies had gone through the effort of getting approved.

      In fact, this is part of why I’m skeptical – by coincidence, the one drug that was approved happens to also be the one that actually works?

      • Tsnom Eroc says:

        Hmm. Can you expound on how that makes you skeptical? On its surface layer, the general form of that statement “Only product X was approved, was shown to be the only one effective” seems reasonable…but that is simply the surface layer.

  14. God Damn John Jay says:

    Snopes: despite media reports, there is no evidence that the Orlando nightclub shooter was gay. This is so confusing to me that I worry it’s some kind of prank, but how could I even check?

    The longstanding jokes about homophobes being closet gays reminds me of a quote (by Breitbart I believe) that a liberal is someone who will call you a homophobe and a f*g in the same sentence.

    I was never able to find find the quote again or even find out what he actually meant, I just find it weirdly accurate in these situations.

    • The original Mr. X says:

      a liberal is someone who will call you a homophobe and a f*g in the same sentence.

      In a similar vein, whenever I’ve seen somebody use the phrase “butthurt” it’s always been a liberal insulting someone less liberal than them.

      • LPSP says:

        Go to 4chan, you’ll find a lot of vaguely- to intensely-rightleaning comments calling people butthurt.

        I think it’s more slightly common among left-leaners that rights, but massively more common among the current 12-28 year-old generation than not.

        • The original Mr. X says:

          Given that 12-28-year-olds are more likely to be liberal, I think that age might be the root cause of the insult’s relative prevalence on the left.

      • The Nybbler says:

        I’m not particularly liberal, but I’ve used “butthurt” to insult those more liberal than me. (Usually in something akin to the “Twitter butthurt police”, when people want Twitter-the-company to stop other people from tweeting disagreement to them)

        Not being particularly liberal I also don’t much worry about the term’s alleged connection to anal sex. I just use the definition of “An inappropriately strong negative emotional response from a perceived personal insult.” KnowYourMeme suggests it’s actually derived from non-sexual childhood spanking.

      • Quixote says:

        For a contrary annecdote set, I’ll note that I’ve only ever wheats it used from the (relatively) more traditionalist set and it is usually directed at a more liberal individual who pointed out something that they felt was offensive.

  15. Miranda says:

    Interestingly, I lost about 10lbs once I started intermittent fasting, and it seems to have become my new weight set-point. (Confounder: it was really IF + much lower carb than I’d been eating previously. Also, I was within normal weight range before and still am. Also, I wasn’t actually dieting or trying to lose weight; I was mainly trying to fix the thing where I needed to eat every two hours and if I couldn’t I wouldn’t be able to think. This is incredibly inconvenient as an ICU nurse, and being able to work a 12h shift without eating if I need to is *really convenient.*)

    • svalbardcaretaker says:

      Wait what? I do have the need to eat every 3 hours thing as well! How did you get adapted to new order?

      • Miranda says:

        I had already tried and failed to adapt to IF a couple of times. This time, I first started skipping breakfast and not eating until 10 or 11 am. Then I started rigorously tracking my diet in the MyFitnessPal calories section, and calculation macronutrient ratios, aiming for >50% of calories from fat. (And about 25% each from protein and carbs, although it varies by day; I don’t worry much about eating enough protein, I figure my body will tell me if I’m not.) Then, once I’d figured out what mix of foods I needed to eat to manage this, I started pushing the time of my first meal later and later, until I settled around 3:30-5:00 pm (and I would stop eating at 10 pm.)

      • Loquat says:

        You may find this site helpful as well. Their basic theory is that your body should be able to dip into your fat reserves for energy whenever your blood sugar gets low, but if you’re in the habit of eating lots of carbs all the time your body can stop doing that and start just yelling at you to eat more carbs every few hours instead. Their solution is to eat a low-carb, high-fat diet, and after a short adjustment period your body should remember that fat can be used for daily energy needs and stop demanding such frequent meals.

        • Edward Scizorhands says:

          > after a short adjustment period

          This is the, er, meat of it. You can break your body’s habit.

          What helped for me was traveling for work. I could only eat at certain points, and I had no food in my hotel room. It enforced the no-snacking concept, and I’ve been pretty good about sticking to it.

          One thing I worry about now is making sure I’m eating enough. I don’t know the answer here. Do I need to count calories? That would suck.

      • dreamt says:

        Same way people get used to polyphasic sleeping schedules; deal with the temporary significant discomfort until one adapts and it is no longer so uncomfortable.

  16. Anon says:

    Sam Hyde and the Eternal Terrorist Victim, locked in combat for aeons untold. We see but a brief glimpse of their story in our lives.

  17. Ryan Beren says:

    While of course we should be skeptical of the study on mice and sugar palatability, it’s so tempting to rush ahead and apply it to humans. The mystery of the obesity epidemic and it’s confusing twists and turns would be solved: commercially produced food tastes too damn good. First it was because it was wonderfully fattening. Then we got all anti-dietary fat. So the food got loaded with sugar. Then we got Atkins diets. Now they sell protein donuts!

    So the obvious test to try in overweight humans: eat mostly food that you dislike.

    • I think I would rather just be fat.

    • Scott Alexander says:

      I’m worried this is the actual best policy, but food that I dislike tastes terrible

      • LPSP says:

        What about merely neutral food? I hate mushrooms for instance, but am just unexcited by courgettes. Quiches and omelettes are nice but not that nice, same with plain tap water. If you tried dieting on foods on that level or lower only, you could get benefits without making yourself retch.

        • Agronomous says:

          I never saw this word “courgettes” until this past week, when my wife sprang it on me. Now I’m seeing it here in the comments. TINACBNIEAC.

          Incidentally, Americans call them “zucchini”, not some weird foreign word.

          • Lumifer says:

            Incidentally, Americans call them “zucchini”, not some weird foreign word.

            Depends on your ethnic background : -D but both are foreign words, it’s just that courgettes come from French and zucchinis come from Italian. Means “little squash” in both languages.

          • onyomi says:

            I was recently surprised to read the phrase “vegetable marrow” in a book. It’s British for “gourd,” I believe? Do people still say that?

          • sweeneyrod says:

            @onyomi

            Yes, “marrow” is the name of mature courgettes in the UK.

          • AlphaGamma says:

            @onyomi: Yes and no. “Marrow”, as sweeneyrod says, is still used- they’re an ingredient in Branston pickle among other things. However, people tend not to specify “vegetable marrow” any more as bone marrow has become less popular as food.

      • Tadrinth says:

        The study seems to line up pretty well with the theory behind the Shangri-la diet. The handwaved evo psych explanation, IIRC, is that tasty foods make your body think it’s summer and time to stockpile calories, while untasty foods make your body thinks its winter and it should conserve calories and food.

        I think I’m going to eat more mealsquares and oatmeal based on this, and avoid everything I can with artificial sweeteners.

        • Justin says:

          Other-way-around explanation: The food tastes good because the body is able to derive energy/nutrients from it easily. The mice may have been engineered to not process sugar as easily, so they don’t get the incentive to eat more of it.

      • Edward Scizorhands says:

        But what if I change to foods I dislike, and then learn to like them? Will I get fat again?

      • Michael Terry says:

        Another solution is to eat whatever you want, but get very strong for your size. Strength training is fat loss training. When you strength train, your body trades fat for muscle. Most people, if they eat habitually, only very gradually gain weight. You can easily outrun this effect by getting stronger so that you improve your muscle to fat ratio.

        Focus on weighted dips, pull-ups, and squats for a couple years at first. Be serious about progressively overloading them.

    • Outis says:

      I think I’ve mentioned this before, but the world was created by an evil demiurge. That is why food that tastes good is bad for you, and vice versa. I am pleased to see new evidence for my theory.

      • Ninmesara says:

        It’s probably not the demiurge’s fault. It’s the fault of our ancestors, who created a society in which it is easy to overeat. If you want your instincts to line up with healthy behaviors, you can always go back to subsistence hunting 🙂

      • Cadie says:

        One thing I’ve done that works for me is to fill up most of the way on healthy and non-fattening foods that aren’t terribly exciting but taste at least okay, and then finish off the meal with a smaller serving of something richer and yummier. Salad with a little dressing or a big portion of lightly seasoned veggies first, and whatever decadent high-calorie goodness is on the menu last, when I’m already partly full and thus don’t have room for more than one piece/scoop.

        Boring food is much more palatable when you’re very hungry, too, so eating the healthy but meh stuff first makes it seem to taste better.

      • Nornagest says:

        It’s too bad this comment wasn’t written by Ialdabaoth.

    • TPC says:

      Stephan Guyenet has written extensively about this with his discussions of satiety and palatability.

      Google Whole Heath Source. Tons of information on low vs high satiety foods and what it means if you want to lose fat.

    • The Nybbler says:

      Seems logical that it would work; who would eat more food they dislike than necessary? But it seems like a very difficult diet to stick to. Especially if you qualify it with “mostly”.

    • Nornagest says:

      Protein donuts taste like chalk, though.

  18. Eli Rose says:

    It looks like a lot of the economists are responding to a specific proposal ($13,000 per person per year, in exchange for all transfer programs) rather than the general idea of UBI.

    e.g. Eric Maskin says “A minimum income makes sense, but not at the cost of eliminating Social Security and Meidcare” and Wiliam Nordhaus says “And the children get nothing? The basic idea is sound but too simplistic as stated.”

    So I don’t think it’s fair to say “Economists are generally pessimistic about it.”

  19. TrivialGravitas says:

    This appears to be the origin of what Snopes is saying. Note that A) It’s not actually an FBI statement, its a source inside the FBI, and B) What the AP is saying is that the FBI has not found concrete evidence. Which is rebutting something nobody actually said, what’s been said is there have been various people claiming to have interacted with him in the gay community. That doesn’t amount to concrete evidence.

    But neither is it particularly unbelievable that the press got a juicy story from some sources and never bothered to examine wether or not the sources’ stories were full of holes.

    • linker says:

      People claim to have interacted with him (1) in specific apps and (2) at the particular club. Here here are several gay dating sites denying the specific claims that he used them, on the record. More interesting is the claim that he was a regular at the club, which could have been for many reasons, such as scouting. Here FBI sources (neither named nor clearly identified as a leak, but maybe I don’t know the code) claim that they looked at cell tower data, which should address this. Although they don’t quite say that, only that he didn’t have a secret life.

      • Edward Scizorhands says:

        The evidence he was gay was specific people who saying it, right?

        The simplest explanation is that they are glory hounds who want the spotlight. We had a guy go on TV at least twice to talk about holding the emergency exit shut. That’s a pretty stupid thing to do, until you realize he wanted to be on TV.

  20. E. Harding says:

    “Pseudoerasmus reviews Empire of Cotton. Even though he’s not a fan of the book, just his hostile summary helped me understand some of what people mean when they say that “free trade” has set back the developing world.”

    -You weren’t aware of this line of reasoning before? It’s fairly common in certain circles.

    “The first fully automated fast food restaurant comes to San Francisco.”

    -Didn’t stuff like this exist back in the 1950s?

    “Everyone knows that “millennials” are far left, but the truth is more complicated – really into gays, marijuana, and immigration, but not much different than older generations on support for the poor or on racial issues (wait, really?)”

    -Just the fact a few provocateurs in the media have been doing what they tend to do doesn’t mean it’s representative of a whole generation. Remember those North Vietnam flags in the 1960s? I wasn’t there, of course, but they did exist.

    “Refugee children who arrive to the US at a very early age like 6 months don’t have substantially better outcomes than those who arrive at a later age like 6 years.”

    -I suspect it’s because the refugees studied didn’t have it all that bad while they were young. Maybe there are some other refugees that did.

    “Countries, rather than going to war, could just assess the relative likelihood of each side winning”

    -That’s what everyone already does before they go to war.

    “Since the 1960s, “values issues” have defined stable left and right political coalitions. Most people dutifully lined up with one side or the other, and most political questions were forced to align with a fixed left vs. right opposition.”

    -Uh, not in the United States, at least. Political coalitions only stabilized as of 2000. I wrote a post on this recently; you may have read it.

    “The 2016 American Presidential campaign, and the UK Brexit vote, have split “left” and “right” internally, each into roughly equal halves. A new basic division of political opinion has emerged—in these countries, at least. But what is it?”

    -Something tells me that the present-day left-right spectrum isn’t changing anytime soon.

    “Some readers may be old enough to remember Occupy and the Tea Party.”

    -LOL.

    “These movements were strikingly similar”

    -No, they weren’t.

    “, although one was on “the far left” and the other on “the far right.””

    -Whatevs.

    If smashing the system is something only the non-college-educated like to do, then I wonder why Barry Goldwater won the White college-educated vote to a greater degree than GWB?

    “Both were highly upset about something, and wanted immediate change, but—it was much noted at the time—neither could say what they wanted, why they should get it, or how anyone could give it to them.”

    -Only true for Occupy. For the Tea Party, the goals were clear: #1. take back the House, #2 take back the Senate, #3 take back the Presidency.

    • Nicholas says:

      I would have pegged the left-right split at Clinton instead of Bush II. What part of the nineties makes you feel things were still in flux?
      Also, while the plan to put friendly faces in power was the strategy the Tea Party’s leadership united under, I don’t know if that’s really what the Tea Party’s members could be said to want. I think they’re being accused of a more general disconnect along the lines of “wanted things that politicians couldn’t give them, decided to elect politicians anyway.” and “knowing what they didn’t like, but not having a clearer goal that ‘anything but that'”.
      Also Also, my prediction is that the left-right split isn’t particularly stable, and in the future historians will just look back on the whole thing (in partial error) as having been roughly homogeneously unstable from the Reagan Counter-Revolution on forward. The seeming strength of the split is just an artifact of having to live with it seven days a week.

      • E. Harding says:

        Technically, 2000 was the Clinton era.

        The 1996 election was very interesting, as it showed what was still in flux and what wasn’t. Greater Appalachia, the Southwest, and Louisiana was still very much in flux at the time. Most of the coastal South wasn’t.

        The Tea Party wanted lower spending. Control of the House could, but didn’t, give them that.

        “Also, my prediction is that the left-right split isn’t particularly stable, and in the future historians will just look back on the whole thing (in partial error) as having been roughly homogeneously unstable from the Reagan Counter-Revolution on forward.

        -What do you mean by this? I don’t understand it.

      • cassander says:

        >I would have pegged the left-right split at Clinton instead of Bush II. What part of the nineties makes you feel things were still in flux?

        The 90s were when the south really transitioned from democrat to republican. Bush II was the first time a republican won the whole south without winning almost every non-southern state as well.

    • fubarobfusco says:

      -Didn’t stuff like this exist back in the 1950s?

      You’re thinking of the automats, such as Horn & Hardart. Automats featured large vending-machines supplied with food made by human cooks in an ordinary kitchen.

      https://en.wikipedia.org/wiki/Automat

    • Psmith says:

      -Didn’t stuff like this exist back in the 1950s?

      Yep.

      The format was threatened by the arrival of fast food, served over the counter and with more payment flexibility than traditional automats; in the 1970s, the automats’ remaining appeal in their core urban markets was strictly nostalgic. Another contributing factor to their demise was inflation of the 1970s, making the food too expensive to be bought conveniently with coins, in a time before bill acceptors commonly appeared on vending equipment.[citation needed]

      At one time there were 40 Horn & Hardart automats in New York City alone. The last one closed in 1991. Horn and Hardart converted most of its New York City locations to Burger Kings. At the time, the quality of the food was described by some customers as on the decline.[5][6]

      Those seem like pretty weak, historically contingent reasons for automats to have failed. But if they are in fact the actual reasons that automats failed, this time may be different.

      (ETA: fubar beat me to it, and also notes that automats had human-staffed kitchens. Well, there you go.).

  21. Cheese says:

    RE: Intermittant fasting.

    There’s a variety of things that come under the category ‘intermittent fasting’ as everyone probably knows and the review points out. 5 & 2, single fast days, etc.

    There’s another form of intermittent fasting which the jury is currently out on, which is 2 weeks restriction/2 weeks normal intake (or 4/2). There’s some evidence from mouse models (a lot yet unpublished I think) that this might actually address the root cause of ‘rebound’ in diets – the theory is that a 2 week period at maintenance intake might be enough to trick the various neuro-endocrine factors involved into thinking everything’s fine and they don’t need to ramp up hunger or ramp down resting metabolic rate. And you might not get that from diets where restriction is more regular or consistent. Maybe. Hopefully.

    Anyway, I know a couple of people involved in clinical trials on this, this one being one of them. It will be interesting to see the results.

    I also agree with another poster who said that it’s interesting 5&2 does not appear worse than continuous engery restriction, mind you I think the evidence so far is that continuous is pretty useless over the long term anyway.

    • anon says:

      I realize this is anecdata from an anonymous internet stranger–ironclad in other words, but I tried this approach in a homebrew form of strictly 1,800 – 2,000 kcal/day for Monday-Friday, then anything I wanted (within reason) on weekends, so 5 days ‘fast’ then 2 days ‘non-fast’. Combined with moderate exercise (about 20 minutes of brisk walking per day). I lost nearly 20% of my body weight over a 1-year period (270lbs to 220lbs) and have maintained that weight for 14 years at this point. Granted I don’t formally eat that way currently and my exercise regimen has changed, but that initial approach was sufficient to lose the weight gradually without wrecking my metabolism–at least in my case.

      The big advantage to me of monitoring calories is how eye-opening it is regarding portion size and the relative energy density of foods. What you learn after doing it for a while is a solid foundation for moderate eating going forward.

  22. Anonymous says:

    How does Genotation compare to Promethease for analyzing 23andMe data? If you’ve already used Promethease is there any reason to use this as well?

  23. Matthias says:

    From the Empire of Cotton review:

    > “War capitalism” was the violent exploitation of the non-West through piracy, enslavement, theft of natural resources, and the physical seizure of markets. It was not caused by superior technology or organisation. Nor did it rest on “offering superior goodsempireofcotton at good prices”, such as you find in the la-la-land of economics textbooks, but on the “military subjugation of competitors and a coercive European mercantile presence in many regions of the world”.

    A bunch of history books I am reading now (The Gunpowder Age and Why did Europe cconquer the World? argue that it was exactly the accelerating European lead in military technology that enabled their world domination.

    But, they didn’t actually profit too much from it. Industrialization made Britain and then Europe rich. Before they were still backwaters, even though they had already conquered swaths of the globe, and been preying on global trade in eg South and South-East Asia for ages.

    The productivity improvements in the military were crazy, even if you just look at simple things like gun shots per soldier per minute. They put even the fastest general economic growth during the height of the industrial revolution to shame. (And those simple things will underestimate improvements, since the guns also got more powerful and cheaper to make.) European drill and logistics also got better and better, just to cover the improvements to organisation.

    • Michael Watts says:

      A bunch of history books I am reading now (The Gunpowder Age and Why did Europe Conquer the World? argue that it was exactly the accelerating European lead in military technology that enabled their world domination.

      Some of that new, world-crushing technology can sound surprisingly simple. Power and Plenty credits Holland’s time as a major naval power to a single innovation: instead of sending cargo ships that need to defend themselves against pirates, send a defenseless cargo ship (more room for cargo!) escorted by a dedicated military vessel that can effortlessly smash a pirate ship.

    • Montfort says:

      There is a rather subtle distinction here. In the summary, at least, I haven’t read the book in question.

      Specifically, the claim is that “war capitalism” was not caused by superior technology, i.e. it did not naturally arise from innovations like the sextant or artillery, but instead was a deliberate(?) series of policy decisions. This is made confusing because the way they are said to have implemented “war capitalism” was by taking over all sorts of things and killing people very effectively, which were enabled by the superior technology, organization, financing, etc. I think the author would acknowledge the advantages of european militaries in the period.

      • cassander says:

        You are correct that this cart before the horse argument is absurd. It’s also depressingly common.

    • Precisely. Even if everything Beckert says is correct (and it is not, there are many errors of fact and interpretation), his framework cannot explain how the West was the one to inflict “war capitalism” on the Rest rather than the reverse. I think Hoffman’s model — which is very narrowly focused on gunpowder technology and related (such as sailing) — supplies a pretty good answer.

    • cassander says:

      >It was not caused by superior technology or organisation. Nor did it rest on “offering superior goodsempireofcotton at good prices”, such as you find in the la-la-land of economics textbooks, but on the “military subjugation of competitors and a coercive European mercantile presence in many regions of the world”.

      The idea that you can have superior military power without superior technology or organization is laughable, at least in the context of the age of exploration. It is very well established in military science that the farther away you operate from your home bases, the weaker your forces are. the Europeans didn’t just conquer their colonies, they conquered colonies thousands of miles away with soldiers that were shipped there in wooden sailboats. And Europeans achieved that level of military dominance BEFORE they profited from their colonies, because without it they couldn’t have gotten the colonies in the first place.

  24. Sniffnoy says:

    If you’re going to link to /r/AccidentalRennaissance, it’s probably worth mentioning that this is the post that originally inspired it.

  25. Sir Gawain says:

    Refugee children who arrive to the US at a very early age like 6 months don’t have substantially better outcomes than those who arrive at a later age like 6 years. This is very strange, because we expect them to be living in a terrible deprived environment before immigration but a much better one afterwards. How do we reconcile this with the “childhood stresses of poverty” theory of poor people’s problems like in that study about the Cherokee reservation?

    Maybe race and/or culture play a role in determining group outcomes…[crimestop]. It’s probably just that there’s a level of 100% random arbitrary racism specifically calibrated for each immigrant group that completely determines their socioeconomic outcomes. Because both cohorts of immigrants face the same subtle, unconscious, hard to detect, yet all powerful, racism, obviously their outcomes don’t differ substantially.

  26. On the Brexit article: Another way to put Trump vs. NeverTrump is that it’s about prowess vs. diligence. As for the solution to the problem of rent-seeking, the only thing that can stop a bad guy with a system is a good guy with a system.

  27. Siah Sargus says:

    Speaking of subreddits, I would like to hightlight a particularly awful one that seems to be missed in the usual corral of “morally repugnant subreddits”: r/PublicHealthWatch. Public health watch is a subreddit I found as I was looking for a subreddit focused on epidemiology, the prevention of contagious disease and the public safety surrounding disease; but instead I found a subreddit focused on hating LGBT people under the guise of concern for HIV/AIDS and sexually transmitted diseases. Seriously. Fortunately the sub is quite small, under 1K subscribed, but as a result of its size, its been able to fly under the radar, and is neither banned nor quarantined by reddit admins. What that means is I have discovered, briefy, a petri dish of shit, a little hate group, that I can lurk and observe in on. I don’t plan on posting or voting, just readed and rolling my eyes.

    Having been caught up in the “vaguely against the excesses of social justice” bubble recently, its a weird relief to know that there are still people who are genuinely homophobic, people that warrant an aggressive social justice movement.

    But still, I hope the admins ban this sick filth.

    • memeticengineer says:

      > But still, I hope the admins ban this sick filth.

      Why? Are they causing actual harm to anyone? It sounds like there’s barely anyone there, and hardly anyone notices their tiny forum for posting mean things. Banning it would just given them attention and let them play the martyr.

      I’m tempted to think your post a Straussian attempt to get people to go read the subreddit in question to make it more popular.

    • birdboy2000 says:

      I hope they don’t. The people who use it will go somewhere and I’d rather have them on reddit than on sites I actually like.

      • Winfried says:

        They used to be at another subreddit that got banned for some flimsy reason other than hate speech.

        I want to say it was something like antiPoz but I only saw it mentioned in some drama subreddits.

        • Siah Sargus says:

          Yeah, honestly, there were a lot of unjustified bannings and some actual hate subreddits closed down; it was more of a pr purge to make the rest of reddit look nice and clean, than it was a coordinated silencing of political speech. The reasons were always flimsy, but the admins made clear that the site belonged to them, and was not an open forum. FatPeopleHate was the most controversial banning, probably just on account of its size (heh), but I honestly feel like nothing of value was lost there, because 4chan always did fat hate threads better than reddit.

    • suntzuanime says:

      I’m sure they agree with you on the desirability of banning sick filth. For my part, I prefer an attitude of pluralistic tolerance, unless they’re causing clear and direct harm to innocents.

      • Siah Sargus says:

        To be honest I think there is more reaction in this comment section of use of “sick filth” than anything else I said. I admitted to lurking on the subreddit – maybe if I just struck that last line out, you wouldn’t see this post as a call to censorship.

    • TPC says:

      I don’t think this was very kind.

    • TPC says:

      Ok, I went and looked. Under the raving hysteria are legitimate public health concerns about specific kinds of behaviors. That is worthy of discussion.

      • Siah Sargus says:

        I don’t think you’ll find much meaningful discussion there.

      • Anon says:

        The legitimate public health concerns are valid but the reason they are brought up seems to be for the posters to gain righteous indignation rather than actual attempts at policy or social coordination to deal with those concerns…

        Or I’m totally wrong but post titles like: “”Herpes infected whore potentially spreads it to multiple other partners and contemplates not telling future partners so she can continue to whore around”” give me that impression.

  28. Steven says:

    Hmm. I went through several antidepressants as a teen with no effect — with the exception of Prozac, which both worked and caused me to break out in hives. (I found the relief from depression so great that I tried to convince the doctor to keep me on it despite the hives; she was worried it would progress to anaphylactic shock.)

    • Scott Alexander says:

      I have one child patient who had a very clear dose-dependent response to Prozac. But she’s 16, so maybe she’s biologically an adult in whatever sense that matters.

  29. Nornagest says:

    Economists are very pessimistic about universal basic income.

    I was hoping for more economics in the responses, but I suppose I can’t expect much from a Twitter-sized response field.

  30. Bram Cohen says:

    The study on voice masking found that women had a much higher dropoff rate after getting rejected from an interview. Maybe the much higher rate of rejection geek guys have to put up with in dating prepares them for job hunting.

    • Pku says:

      Alternatively, possibly women are more likely to give up because they’re likely to see individual rejections as part of that structural sexism they keep hearing about.

      • lunatic says:

        My money’s on neither of the above.

        • Matt says:

          My money’s somewhat on the former. As a rule, women get coddled more. I don’t think it’s anything necessarily specific to geek guys though.

    • Anonymous says:

      I find it quite hypocritical when western media pushes so overwhelmingly for women in tech, while tech is really in the middle on the job-gender ratio distribution spectrum. See the statistics: http://i.imgur.com/qIeobkE.jpg
      Why doesn’t the media push for more women in construction, oil drilling and cement work?
      To an unbiased viewer it looks as if women-dominated jobs tend to be less physically hard and way less dangerous than jobs that are male-dominatied. This looks like a plausible cause for males having 90% higher occupational mortality.
      If we start to talk about social justice and fairness, I should note that this looks very unfair & skewed: if you are born male then comfortable social jobs (e.g. schoolteacher) are de-facto forbidden for you and you are being pushed into dangerous industries.

      On the second note it is also pretty hypocriticial that the blogpost author and commenters tend to conjure elaborate hypothesis about second-order female/male speech features while ignoring the so-called “WAW effect”: https://en.wikipedia.org/wiki/%22Women_are_wonderful%22_effect which describes a prevalent bias in favor of women. It seems plausible that males (which tend to be tech interviewers) just like females, add to this the media push for more women in programming and you get a widespread bias in favor of women.

      I wonder if an unbiased observer would look at all this and conclude that our society tends to be quite gynocentric…

  31. Mongeese says:

    Scott, in the blogroll you seem to have linked to Ribbonfarm twice for some reason?

  32. linker says:

    The Wikipedia article on the State of Scott claims that it was a Union enclave, but Scott County borders Kentucky, thus was part of the contiguous Union.

    • AlphaGamma says:

      Weren’t the relevant parts of Kentucky under Confederate control for at least some of the war?

      • linker says:

        Are you thinking of something specific, or just that there was some Confederate control in Kentucky? I think it was generally western, but it was all pretty amorphous, in both Kentucky and Tennessee. I’m not sure it makes sense to talk about Union control in Scott, either.

  33. I find it somewhat amusing (and possibly also encouraging?) that the complaints of an anti-capitalist match up so well with the exact same things that libertarians love to complain about:

    – protectionism

    – the military industrial complex

    – forced labor

    – eminent domain

    Surely this means we can finally have a partisan realignment in which libertarians and leftists work together, right?

    • E. Harding says:

      DW-NOMINATE is discouraging as to those prospects.

    • Nicholas says:

      The essential sticking point is whether the word capitalism is used as the token for “the system that only exists when all of these things happen” and “a system that can be defined in part to the degree that these things aren’t happening.”
      To the Leftists, Libertarians look very strange. To imagine how strange, try to pretend how surprised you would be to discover a group dedicated to fighting anti-Semitic discrimination called the National Socialists Group.
      Also I’m pretty sure this alliance was tried once before in the Seventies, and that’s what got us here. So while I’m not skeptical of it happening, I’m skeptical of it working.

    • Lysenko says:

      There are a lot more leftists than libertarians, so I for one would be skeptical of the benefits to libertarians relative to the costs.

    • birdboy2000 says:

      We’ll accept it the moment Libertarians reject private property.

      • Urstoff says:

        The alternative to private property is some sort of state/collective enforced property rules, right? The obvious rejoinder “if you don’t believe in private property, I’m going to come in to your house and eat your food” I imagine is a straw man. It’s not that there’s no such category as property, it’s that the state/collective determines who gets what and what they can do with it. Thus, at no stage can a person be said to own something or have legitimate property. That sounds awful, but is that what communists/anarcho-syndicalists/etc. mean when they say they want to get rid of private property?

        • Jill says:

          Do we actually have a single communist or anarcho-syndicalist on this board. I imagine there may be plenty of them on the Internet. Has anyone actually met one in person? Or even more than one? I haven’t.

          Even on the Internet, I don’t see them being there. What I see is them being used as straw men to accuse someone who disagrees with you of being way far over on some very extreme part of the political continuum.

          We’re so politically polarized that we continually polarize each other by putting each other on political extremes that we are not actually on.

          And God forbid anyone should try to be pragmatic about politics. They would be placed by others at some extreme– maybe at one extreme by some folks and at the other extreme by others.

          Because polarizing and categorizing each other politically is one of the 2 primary political activities of our time– the other primary activity being bashing every official and every citizen who is of a different political tribe than our own.

          • John Schilling says:

            Multiheaded is a straight-up communist, though only intermittently active here. And then there’s this “Jill” character, who I read as de facto communist while avioding the label, but probably she can explain her views better than I can. Not sure about anarcho-syndicalists.

          • James Picone says:

            @John Schilling:
            Oh come on. Jill is pretty garden-variety left wing. Describing her as a ‘de-facto communist’ is a ridiculous exaggeration

          • “Because polarizing and categorizing each other politically is one of the 2 primary political activities of our time– the other primary activity being bashing every official and every citizen who is of a different political tribe than our own.”

            For example, by writing:

            “Libertarianism’s function in our society– as opposed to its high ideals– is as one sector of the Republican party, and as excuse givers for the destructive actions of crony capitalist welfare queen companies.”

            ?

          • Publius Varinius says:

            > Even on the Internet, I don’t see them being there. What I see is them being used as straw men to accuse someone who disagrees with you of being way far over on some very extreme part of the political continuum.

            Academia is your best source of communists. For example, theoretical CS and constructive mathematics is a great breeding ground for communists: a great entry point is
            Sterling, the inventor of the RedPRL programming language, and cop-murder advocate, or really look up any of the signatories of the LambdaJam petition (linked in the appropriate thread).

          • Lysenko says:

            I’ve met various Trotskyists and Socialists, and exactly one person who was proud to be an outspoken Stalinist. That last one was the friend of a friend met via online gaming. He was in his late teens, and rather strident about what great visionaries and leaders of men Lenin, Stalin, and Mao were. He would regularly post links to some official party newspaper or other (Don’t remember if it was CPUSA or some splinter faction thereof), and I got the strong impression that he had latched onto “Communist” as a label entirely on the basis that his environment growing up routinely used the term as a pejorative and the ideological commitment came later. He would regularly try to break into semi-literate Russian to demonstrate his cred.

            Speaking with him was…interesting, as he went beyond claiming that communism couldn’t be blamed for the famine deaths under Chinese and Soviet control and asserted that there were no such famines, pure products of Western Capitalist propaganda. He first denied that dekulakization and the decossackization existed, and within thirty minutes of being presented with enough sources he defended them as entirely necessary, logical, moral, and ethical exercises of policy in light of the goals of the time.

            So…interesting. Provoked high blood pressure more than once, but interesting.

            As far as pragmatism goes, if by that you mean coalition-building, compromising, and settling for what I can get, sorry, no interest in it. My personal policy preferences are so idiosyncratic and unpopular as to be a punchline for conservatives, liberals, or in many cases both. What prospect do I have of advancing my values by allying with EITHER camp? Alliances and compromises only work if you already have enough power to make violating the terms of the alliance or compromise costly. To my mind, libertarians and libertarian-ish flavors of conservative lack that power at this point. In America at least I think that true Socialist, Communist, and the various Left-Anarchist perspectives are similarly screwed.

            The only alliance I can see a point in would be some sort of alliance OF the extremes AGAINST the status quo. Let the alt-right who shall not be named and the anarcho-everythings and the greens and the libertarians and the communists all team up to systematically dismantle the structural supports of the political status quo and shatter the Overton Window entirely in the hopes that whatever comes next makes some progress possible…but that’s not going to happen.

            Doing so peaceably requires a level of long-term coordination that such disparate groups can’t hope to manage (as for means, ballot access reform was going to be my hobby horse before I death-spiraled out of college). The costs of doing so violently are so horrific that no one’s ready to pay them yet, myself included. So that leaves the status quo as it is.

            David Freidman beat me to the point I was going to make about polarization, categorization, and self-demonstration. The hazards of sneaking personal use of work internet and posting when you get off shift. So I’m deleting the first part of it, but leaving the rest. I’m leaving the rest because I actually think there is some meat to what Jill’s saying about the function of some fiscally conservative, socially liberal voters. She’s wrong in generalizing that to mean libertarians in general, but here’s the problem as I see it:

            Polarization and simplification is baked into our political structure. Most people aren’t full-time political activists. Fewer still have the platform and talent to persuade large numbers of people to change their behavior. That means that most have exactly two vectors for meaningful input into our political process. Their vote, and their money. Most people don’t care enough about politics one way or the other to donate their often very limited financial resources to specific causes, and as giving goes, I think there are some pretty strong arguments to be made that there are better and more effective places to spend your money for improving quality of life for fellow men and women. That leaves voting.

            We have a structure that strongly favors a two party equilibria, and we’ve further strengthened that with laws that entrench the current equilibrium. It would be much harder for a third party to unseat one of the major two now than it was in the 1850s.

            This means that all you have as quantifiable external data on the vast majority of citizens is their voting record, and each point in that data set has two states that dominate over all the others. How can we NOT have polarization? The problem isn’t cultural. It isn’t personal. It’s structural.

            The average person’s options here are pretty damn constrained, and once someone falls into one of those two big-ass statistical buckets, the ones labeled “Member of [Redacted] Bloc, P< 0.2”, it doesn’t matter how or why they got there. From an outside view, they are functionally identical in their agency to the lowest common denominator, most extreme, least rational person in that same bucket. As for the label? Red Tribe, Blue Tribe, Liberal, Conservative, Republican, Democrat, Right Wing, Left Wing? I don’t think it matters much.

            Pick your poison. If you want to fix the problem, figure out how to correct the structural problems that created it. The cultural issues are effect, not cause.

          • Anonymous says:

            And then there’s this “Jill” character, who I read as de facto communist while avioding the label, but probably she can explain her views better than I can.

            Well no, she can’t.

            She knows she isn’t a communist because no one knows any communists and calling someone a communist is an exaggeration and an attack because everyone knows that communism doesn’t work.

            It’s just that she can’t articulate any actual differences she has with communists nor see any reasons why communism didn’t work.

          • Chalid says:

            People calling anyone left of Joe Manchin a communist or socialist is one of the most annoying things about internet political discourse. I would really appreciate it if that habit did not spread to SSC.

          • Urstoff says:

            Well this was not the reply I expected to my comment.

          • onyomi says:

            “Has anyone actually met one in person? Or even more than one? I haven’t.”

            I personally know at least two anarcho-syndicalists. And they are not considered weird in my academic circles. They are basically just a somewhat extreme version of what most of the academics I know believe.

          • John Schilling says:

            Oh come on. Jill is pretty garden-variety left wing.

            Jill’s positions on economics are both unusually strident and unusually simplistic for a “garden-variety leftist”. I genuinely cannot recall her offering anything in the area of economic policy or philosophy that differs substantially from what a moderate communist would likely have to say about the same matter.

            And I was sincere when I suggested I would like to hear her own views on the subject, which is likely to be far more productive than you and I arguing about what she believes.

          • Anonymous says:

            What the heck is a moderate communist? Throwing around communist in 2016 as an insult is bad enough, doubling down and actually defending it just makes you seem like you got lost on the way to a 1950s themed reenactment event.

            Actual communists are even rarer and less relevant than actual open borders advocates. But they sure do make fun weakmen amirite?

          • Whatever Happened To Anonymous says:

            He’s making fun of the way she conflates and strawmans the views of people she doesn’t agree with, you dingleberries.

          • E. Harding says:

            “People calling anyone left of Joe Manchin a communist or socialist”

            -Have you ever heard of the Junior Senator from Vermont?

          • hlynkacg says:

            @ WHTA
            Not called for.

          • Whatever Happened To Anonymous says:

            The point is that he’s not being uncharitable or using a weakman, he’s just being an asshole. Whether that’s justified or not, well, I’ll just await the judgement of the Rightful Caliph.

          • Jill says:

            Yes, John, people who are only slightly Left of center, like me, are constantly assumed when on the Internet to be Communist. Part of the process of polarizing and categorizing one another, that is our society’s political process.

            We have only black and white. If you are not Right Wing, then you are assumed to be a Communist. But is there anyone who really is a Communist?

            James, yes it is a generalization– and one that is made constantly on the Internet about people even very slightly Left of Center.

          • There’s still a lot of sentimentality on the left about communist governments— I don’t know whether the sentimentality has policy implications, but it does make me nervous.

          • Jill says:

            No one I know on the Left– and I know plenty of people there– is the least bit sentimental about Communist style governments. I expect I could find some Right Wingers on the Net who are sympathetic to Naziism, but I wouldn’t necessarily say that all Right Wingers here are Naziis. Although I would be tempted to say that the ones who just called me a Communist are.

          • CatCube says:

            @Jill

            There’s a pretty brisk business in Che Guevera t-shirts for a lack of sentimentality for Communist governments.

            And, by the way, people wearing those t-shirts don’t get run out of their college campus, despite the ideology that ol’ Che represents being almost as grotesque as Hitler’s.

          • Snodgrass says:

            “There’s a pretty brisk business in Che Guevera t-shirts for a lack of sentimentality for Communist governments.”

            Che Guevara T-shirts are pretty clearly promoting the Che of the Motorcycle Diaries rather than the Che who spent eight years in Fidel’s government; they’re promoting rebellion against the oppressive right-wing regimes in Latin America of the early Fifties, and rebellion against those regimes was mostly the right thing to do.

          • cassander says:

            @Snodgrass

            And my Hitler T-shirt is just protesting the unfairness of the Versailles treaties, my objections to the danzig corridor, and my desire for peaceful anchluss with austria, not any of the nasty stuff that came after.

          • Hector_St_Clare says:

            I don’t comment here much and only read this blog occasionally, but I guess I would self-define as communist, broadly speaking. Albeit culturally conservative in some regards.

          • @Hector:

            Would you be willing to discuss your view of how a communist society would work, economically speaking, in particular how it solves the coordination problem? Central planning, with authorities at the top figuring out what everyone should be doing how, or something else?

          • TheAncientGeek says:

            A strong point against Jill being a communist is that she hasn’t specified any particular tendency thereof.

          • Vorkon says:

            Do we actually have a single communist or anarcho-syndicalist on this board. I imagine there may be plenty of them on the Internet. Has anyone actually met one in person? Or even more than one? I haven’t.

            How about the person who said, “We’ll accept it the moment Libertarians reject private property,” to whom the comment you are responding to was directly replying?

          • Anonymous says:

            Nah, he’s just a revolutionary socialist.

          • Hector_St_Clare says:

            @David Friedman,

            Hi, thanks for this response! I’ve got to say, I find this blog pretty interesting.

            I go back and forth on how optimistic I am about central planning versus markets. Around the late 2000s after having read Alec Nove on the Soviet economy, I favoured a market-based communism on the Yugoslav model. (I.e. an economy with near-complete ownership by either public or worker-owned enterprises, with markets setting wages and prices). I’m more optimistic about planning and less optimistic about markets since then (partly as a result of having advanced in my professional career and seen the extent to which we can model and plan complex biological systems) although as I said I generally do go back and forth. I guess at this point I’m generally in favour of the Hungarian model: planning with some degree of freedom for enterprises to make their own decisions, a large role for workers’ cooperatives (especially in agriculture) and a limited role for prices, including prices that fluctuate within a fixed range. I think such an economy would combine some of the benefits of planning and the market.

            Of course, I’d also favour a relatively gradual and piecemeal approach to communism- along the lines of Bukharin rather than Stalin- with a gradual accumulation of productive property and a gradual increase in regulatory control by the state. (Not least because the way Stalin did things ended up with massive famines). So presumably we’d be able to see exactly the point at which the balance between planning and market was optimal.

            I’m well aware of the limitations that centrally planned advanced-industrial economies experienced in Europe, relative to market economies, by the 1980s. (In developing countries the story is more mixed: African communist states didn’t underperform African capitalist ones, IIRC). I still identify with communism in spite of that, for several reasons.

            1) I’m hopeful that advances in information technology and statistical methods since 1989 would allow an advanced industrial communist economy to work better than the European communist states did in the 1980s.
            2) I think the communist economies performed better than many people give them credit for. For example, the exact East German GDP in 1989 is disputed, because different people have different methods of accounting for quality differences (since the GDR wasn’t a market economy, you can’t just account for quality based on market price). If Gerhard Heske’s numbers are correct, the GDR went from 40% of the west german economy in 1950 to 55% in 1989, which means it actually grew faster than the west. (And without depending on Middle Eastern immigration either, which is turning out to be its own trainwreck for Germany). And if that’s true, the GDR’s lower economic performance by 1989, which was supposed to prove the inferiority of communism, actually proves just that they started from a much lower base (largely due to extremely harsh treatment by the Red Army between 1945-1950).
            3) Advanced capitalist economies have started experiencing their own kind of stagnation, which in some regards is even worse than the communist stagnation era. Median income in the United States since 1975, for example, has grown at around 0.55%, which is lower than median income growth during the ‘stagnation era’ of the Soviet Union, 1975-1985. (The figures are the same if you just compare 1975-1985 in the Soviet Union as well).
            4) The communist economies had some signal advantages over the west, that (at a particular level of development) might cancel their disadvantages with respect to GDP growth. Economic inequality was much lower in the advanced communist states relative to advanced capitalist ones, for example. (The GDR had a Gini index of 17, IIRC, which is lower than any country in the world then or now). And for some people (e.g. industrial workers) communist economies allowed better opportunities than modern capitalist economies where their jobs had been automated away. Soviet coal miners, for example, made 2-3 times as much as computer engineers.
            5) Most generally, I’d say that people who actually experienced communism (in eastern Europe and the former Soviet Union) and have now experienced capitalism, generally seem to think their countries were better off under communism. This is most pronounced in the former Soviet Union where 58% of people say they wish they had communism back, but it’s also true of countries like Romania, Hungary, and arguably even Lithuania. That seems, to me, like a fairly good piece of evidence that late stage communism might not have been as bad as most people in the west seem to think.

        • sweeneyrod says:

          One possibility is that under a successful anarcho-syndicalist society, it would be fine for you to come to my house and eat my food, but no-one would do so (and it wouldn’t matter if they did), as everyone would live in small communes with enough food for all.

        • I’ve met Maoists, but that was long ago in college. Multiheaded is the only communist I can recall encountering online in recent years.

          There are lots of self-defined socialists around, including one who did pretty well in the Democratic primaries. The problem is that the label means different things to different people. I’m pretty sure that Sanders isn’t in favor of government ownership and control of all of the means of production.

        • “No one I know on the Left– and I know plenty of people there– is the least bit sentimental about Communist style governments.”

          Do you count Chomsky as on the left? He coauthored a chapter defending the Khmer Rouge, not merely communists but the most murderous government of the 20th century.

          Back during the Vietnam war there were quite a few people whose basic objection was that we were on the wrong side.

          • sweeneyrod says:

            I think support for Communist governments has probably decreased a lot since the fall of the USSR. In my experience, there are very few left-wingers who would outright defend Stalin or Pol Pot, but there is vastly more tolerance for joking praise of Stalin than of Hitler; more appreciation for the positive aspects of Communist regimes than right-wing ones (plenty of people point out how fast the USSR industrialised, very few remark on how the Nazis reduced German unemployment); and still lots of admiration for Cuba (Che hats etc.). E.g. see this picture.

        • CT says:

          Communists and Marxist generally make the distinction between personal and private property.

          https://en.wikipedia.org/wiki/Personal_property#Personal_versus_private_property

          https://www.marxists.org/subject/economy/authors/pe/pe-ch28.htm

          Social ownership under socialism extends to the means of production and the finished products. A part of these products subsequently becomes means of production and remains social property. Another part, consisting of objects of consumption, is distributed among the workers in accordance with the quantity and quality of the labour of each, and becomes the personal property of the working people.

          • Urstoff says:

            Interesting, thanks. So once the goods are distributed, what’s yours is your and what’s mine is mine?

          • Viliam says:

            Where exactly is the boundary between “objects of consumption” and “means of production”?

            For example, my computer seems to be the former while I am browsing web, but becomes the latter when I try to program something useful. Is perhaps that the moment when the Party should come and take it away? Or maybe just install a blocker against downloading certain apps, such as programming languages?

            What about the jobs where the important “means of production” is what’s inside one’s head; for example therapists?

            I guess there would only be some rule of thumb, such as: if someone notices you making too much extra money using X, the Party will take X away and throw you into prison. Which would be a disincentive against inventing useful things. But maybe that’s a feature, not a bug, because the world without smart inventions is preferable to a world where smart inventors enjoy benefits that others don’t have.

          • Julie K says:

            Where exactly is the boundary between “objects of consumption” and “means of production”?

            Perhaps the boundary would be items expensive enough that we can’t just give one to each person.

          • CT says:

            @urstoff: Precisely. From the Communist perspective, if there’s ever a point it makes sense to seize personal goods generally, rather than any which are criminally anti-social, it means that tragedy has struck industrial society and that the infrastructure of wealth production has been disrupted. There are likely to be some edge cases during the reappropriation of collective wealth from the bourgeoisie.

            @Viliam: I think @Julie K’s suggestion broadly applies, though I’m sure it’s up for some heated debate. In your specific example, the networked personal computer is so tightly integrated into and informed by the capitalist spectacle that I find it unlikely to be able to justify its social and environmental costs within a communist society. Under such conditions, it’s likely that they would only remain personal property among aficionados of computer history.

          • Jiro says:

            I think he’s very clear about it: your personal computer counts as means of production and in an ideal Communist society it would be confiscated.

          • LHN says:

            Orwell certainly noticed– he thought literature was a fundamentally liberal art form, and didn’t believe it could survive totalitarianism of any variety. And while he was a committed socialist, he seemed uncertain and ambivalent what that would mean in practice for writing as a profession. (E.g. in “Writers and Leviathan” (1948))

        • Hector_St_Clare says:

          See the distinction below between personal property and private property.

          There are lots of different communists who have lots of different visions of the exact end-goal, so I can only speak for myself, but I believe in (ultimately) socializing the *means of production*, not taking away small scale personal property.

          For what it’s worth, communism is a fairly rare political preference in America, but it’s not at all uncommon in other parts of the world- Russia, for example. Almost 60% of Russians currently say they would like to return to a communist political and economic system.

          • Nornagest says:

            I don’t see quite the strong distinction between personal and private property, and what the “means of production” is and is not.

            Marx was writing in the middle of the Industrial Revolution, when there was a sharp dichotomy between cottage-scale production (capital-light but uneconomical) and industrial-scale production (capital-heavy, economical, huge clanking steam engines everywhere). Even then you could make stuff at home, and a lot of people did. But Marx wasn’t content with a medieval smallholder’s level of prosperity for everyone; his whole schtick was about technological utopianism. In his world you couldn’t scale up to that level without a lot of capital, capital that couldn’t easily be raised selling nails or hats or whatever out of the back of a cart: so the means of production are the large capital investments that allow economical production of goods, and which would be totally unaffordable to their operators.

            It doesn’t make as much sense when you’re dealing with knowledge work, or even when you can buy machine tools with personal savings or a small loan. Economies of scale still exist, but more as a gradient than a binary, and the loss of contrast there undermines a lot of Marx’s original arguments.

          • Hector_St_Clare says:

            @Mark Atwood and @Nornagest,

            Of course it’s impossible to draw a strict distinction between personal and productive property, as there are things that fall on a continuum. Household gardens are a good example- there’s no strict line between gardening as a hobby and selling fruits and vegetables as a means of income, so any line you draw is going to be somewhat arbitrary. Partly for this reason, communist regimes varied somewhat in how much tolerance they had for small scale agricultural ‘household’ production. Nevertheless, the fact that any line we draw is arbitrary, doesn’t mean we shouldn’t draw a line at all. Cf. the line from Burke about ‘the existence of dawn and twilight doesn’t mean we can’t distinguish between light and dark’.

            There are two separate issues conflated in your response. An enterprise consisting of an individual and their household, on their own, owning land/tools and producing things for sale isn’t a capitalist (they would qualify under what some 20th century Marxists called the ‘household mode of production’). Capitalism is defined by the existence of a capitalist class, and if you don’t employ workers you aren’t a capitalist. Their land and/or tools might still qualify as productive rather than personal property, though. The things you would want to look at are how expensive and involved the equipment they own is, how much capital is tied up in it, how much they are actually producing (in terms of food, clothes, houses, or whatever else), and those things (at least for me) would help determine whether that equipment or property should qualify as personal or productive (and thus whether it should be socialized or not).

            I assure you that I know something about, at least, the process of agricultural production (having been a Peace Corps agriculture volunteer, having a graduate degree from a crop science department and currently employed by another, and knowing quite a few farmers both in this country and overseas). In agriculture, at least, I wouldn’t have a particular problem drawing a line between personal and productive property, and between farms that I think should be socialized or cooperativized and ones which shouldn’t.

          • Hector_St_Clare says:

            @Nornagest,

            Actually, I think technological advancement since Marx’s time has strengthened the contrast between personal and productive property. It takes a lot more capital to grow food today than it does in the mid-19th century. You need tractors, fertilizers, insecticides, and you also need much more land in order for farming to be economical. This is part of why ownership of agricultural land is much more concentrated than it was in Marx’s time. Same with other enterprises: you can plausibly have a household business making oxcarts. You can’t really have a small business making cars, much less airplanes.

            I work in a ‘knowledge field’ (I’m a research biologist) and, no, I could not do what I do out of my basement. It’s extremely capital intensive in terms of the equipment, reagents, etc., involved. Which is partly why agricultural research, pharmaceutical research, etc., is dominated by a few really large companies, or else by government-funded agencies which also have big research budgets. You could do biology in the early 20th century in your basement, possibly, because there was a lot more low hanging fruit. Not so much anymore.

          • Agronomous says:

            @Hector_St_Clare:

            …and those things (at least for me) would help determine whether that equipment or property should qualify as personal or productive (and thus whether it should be socialized or not).

            In agriculture, at least, I wouldn’t have a particular problem drawing a line between personal and productive property, and between farms that I think should be socialized or cooperativized and ones which shouldn’t.

            Just to be 100% crystal clear here:

            When you say equipment, property, or land should be “socialized”, you mean it should be taken away from the current owners using government force, arresting and imprisoning those current owners if they resist, right?

          • Hector_St_Clare says:

            @Agronomous, yes, in general.

            Again, I’d favour a relatively gradual transition, so not all of a sudden. And in cases where it was financially possible I’d try to give partial compensation to previous owners, or in some cases perhaps keep them on as owners (both strategies were used in some instances in Cuba). Having said that, in principle, yes, socializing the means of production is going to involve government force.

    • Anonymous Comment says:

      Libertarians are a small minority, they have no future in electoral politics. If a person wants to live under a genuinely libertarian government then that person needs to look for solutions beyond democracy. Libertarians need to somehow gain a sovereign peace of land. This won’t be easy but its going to be impossible until libertarians wake up and stop putting their hopes in democracy.

      I think the libertarian movement’s best shot is to ally with the other secessionist groups. In particular this means the White Nationalists who want to create a white ethno-state. But libertarians should be willing to help anyone who agrees the libertarians get their own state.

      • Jiro says:

        You seem to be confusing “libertarians think X should be permitted” and “libertarians like X”. Libertarians think that white nationalists should be permitted to exist and guaranteed the same freedoms as everyone else, but libertarians don’t like them or their goals. Why would you ally with someone you don’t like?

        • The Nybbler says:

          An alliance of convenience, presumably. But IMO an alliance with the white nationalists would be a scorpion-and-frog sort of thing.

        • Anonymous Comment says:

          Its not worth forming a coalition unless the coalition winning lets you achieve your main goals. The White Nationalists and libertarians however can still form a coalition. If their coalition wins they would both be willing to agree to “ok we both take our pieces of land that we run accoridng to our values.” No other group I can think of would actually be willing to let the libertarians go their own way.

          The White Nationalists are the only group, that I can think of, whoa re actually willing to genuinely ally with the libertarians. The libertarian project is deeply unpopular. Libertarians cannot be too choose-y about allies.

      • onyomi says:

        This is actually the plan of a certain subset of libertarians, though of the two major youtube proponents I know, one is more of a white nationalist and the other more of a left-wing anarchist in the Noam Chomsky vein.

        I certainly find it both more likely to succeed and more ethically defensible than “elect a libertarian dictator to impose freedom on all.” The more truly libertarian position is “freedom for those who want it,” “restriction for those (and only those) who want it.”

      • Tibor says:

        The Free State Project* seems to me like a good way to get reasonably close to libertarian ideals with democratic means. Recently, they got the 20 000 pledges they required to start the project. We can see the outcome.

        Obviously, this works on a state level, which means they cannot influence the federal politics (at least not directly) this way. Still, it seems like a nice relatively low-risk and low return venture, the high risk and high return one is (although it is not for libertarians only, it could be used as a platform by anybody) seasteading.

        There is a European version of the Free State Project, although they only started recently and so far they’re pretty small compared to their US counterpart. Their advantage is that they want to move to Ticino, Switzerland, which is smaller in population than New Hampshire (and in a country which has 50 times fewer people than the US and which is probably more decentralized than the current US). Also, possibly, Switzerland is probably closer to the libertarian ideal today than the United States are and given their system of government it has a good chance to stay that way.

        Also, Ticino and actually all of Switzerland is absolutely beautiful, so that’s another advantage 😛 At least if you like mountains 🙂

        An alliance with people like Le Pen (the closest you get to white supremacists in Europe, except for literal neonazis who have zero political significance) sounds like a terrible idea. Le Pen in particular is protectionist and rather socialist AND socially authoritarian (I think that is a better description than socially conservative, at least from the libertarian point of view – radical left may not be socially conservative but they are also socially authoritarian and for a libertarian it is more or less the same as Le Pen in this respect). There is virtually no common ground between people like her and libertarians.

        *The website seems to be malfunctioning at the moment. It works, but the design is not there.

    • cassander says:

      It’s called liberaltarianism, and the trouble with it is that the leftwing hostility to capitalism leads them to throw pretty much any point of agreement under the bus whenever push comes to shove. Want to be anti-protectionist? the left say “sure, except for mandating a “fair playing field” on labor standards. eminent domain? Sure, unless we want to build a train system or economically re-develop a disadvantaged community. And so on.

      • Urstoff says:

        Likewise, conservative/libertarian alliances hit the rocks when conservatives think the states (and not the federal government) should be able to make all sorts of crazy laws restricting various liberties.

        • cassander says:

          Since the conservative libertarian alliance spends most of its time opposing the left’s latest ideas rather than enacting its own, that’s less of a problem.

    • Jill says:

      Libertarianism’s function in our society– as opposed to its high ideals– is as one sector of the Republican party, and as excuse givers for the destructive actions of crony capitalist welfare queen companies.

      So there is no way for such a group to join with Left Wing folks, as they are defenders of the most Right Wing aspects of the current system.

      • keranih says:

        Libertarianism’s function in our society– as opposed to its high ideals– is as one sector of the Republican party, and as excuse givers for the destructive actions of crony capitalist welfare queen companies.

        Jill, what would it take to falsify this statement to the point where you saw Libertarians as a separate thing, neither Republican nor Democrat?

        (Also – crony capitalist welfare queen companies – come one, you can do better than that.)

      • onyomi says:

        Pretty much all prominent libertarians complain about crony capitalism.

        • For current examples of crony capitalism, consider subsidies to firms pushing renewables or electric vehicles, or the biolfuels mandate.

          Which libertarians oppose. Lots of other people don’t. I don’t know what Jill’s view is on any of those–perhaps she can tell us.

          • Jill says:

            I am for these. The reason being environmental. These are a tiny drop in the bucket of the large number of programs that a person could call crony capitalism. And they are for what environmentalists would term a good cause– to save the earth. Of course Libertarians would be against them, because they are environmentally focused.

            Environmentally oriented subsidies lack the Ayn Randian virtue of being based on individual greed and selfishness, so it does make sense, from a theoretical veiwpoint.

            Libertarians are not environmentalists. They defend to the death people like the Kochs and their “right” to pollute the earth to death and not to clean up their own messes they make– just as basic Republicans do. Which is why the effect of Libertarians is essentially the same as the effect of basic Republicans on our society.

          • Nornagest says:

            @Jill — You do realize that David there wrote The Machinery of Freedom, right? You don’t need to assume anything about what libertarians believe. You’ve got a serious libertarian theorist right there. You can just ask him.

            That being said, I’m not too unhappy with most clean-energy subsidies. They’re a market distortion, sure, and you generally want to avoid those, but carbon — if you buy the basic climate change argument, and I know David is more skeptical of that than I am — is one of the better examples of a negative externality that’s not being well captured by the market. A carbon tax would be a better approach, but subsidizing clean energy isn’t a terrible one.

            As long as it’s actually having a positive effect. You can label a lot of things as green.

          • “Of course Libertarians would be against them, because they are environmentally focused.”

            And not because they are against crony capitalists? Do only those crony capitalists you are opposed to count?

            So far you are the only one who has offered an excuse for a particular form of crony capitalism. I thought that was our job.

            The biofuels mandate isn’t a “tiny drop in the bucket.” It’s a huge cost to consumers and, as a side effect, promotes world hunger by driving up food prices.

            What are the subsidies to crony capitalists, ones you don’t approve of, compared to which biofuels is tiny?

          • James Picone says:

            @David Friedman:

            …as a side effect, promotes world hunger by driving up food prices.

            How many starving third-world children normally buy corn from the US? I’d be surprised if the biofuel thing was particularly relevant for world hunger.

            My understanding was that most starving-third-world-countries are more a product of not having the infrastructure, more than not having enough money.

          • Agronomous says:

            It’s like that scene in Annie Hall, where Alvie pulls out the actual Marshall McLuhan….

          • “How many starving third-world children normally buy corn from the US? ”

            I have no idea, but corn from the U.S. is a close substitute for corn from other places and corn is a somewhat less close substitute for other foods.

            If the Saudis increase output, fuel prices go down in the U.S., and would even if the U.S. didn’t import any Saudi oil. Isn’t that obvious?

            Similarly, if you turn a third of the maize produced by the largest producing country in the world into alcohol, that increases the world price of maize.

          • Tibor says:

            @David,@James: A point made even by an old-time communist like Fidel Castro when he was condemning the evil imperialists who use maize and soya for biofuel and thus raise the food prices. That was also roughly at the same time as the Arab Spring and in fact some people argued that the raising prices of food due to biofuel were one of the factors that started these uprisings in the Arab world.

            I know it might sneering (although it is mostly a feeling of a frustration and not meant in an insulting way), but so often I see people on the left who seem to be really nice and decent people, who are well-meaning but who have a catastrophic understanding of economics which leads them into supporting things that hurt people a lot. I mean, I am not economist myself (and I am annoyed by libertarians who pretend that “it’s all obviously this or that way” and who believe that they magically learned everything about economics by becoming libertarians), but the most obvious mistakes seem to be done by either people on the left or protectionist people on the conservative right. What makes it worse is that a lot of those people have an instinctive distrust of economics and don’t really want to learn it.

          • HeelBearCub says:

            I think the ethanol program needs to be scrapped, but that’s because it’s a cruddy biofuel. We are spending a great deal of money to make some mediocre fuel that doesn’t even give us the benefits we wanted.

            That said, I’m not sure we really know what would happen to corn production if the ethanol program was canceled. It seems unlikely to me that we are currently producing as much corn as we possibly could in the world, so it strikes me that the reason so much corn is grown is that ethanol makes it lucrative.

            Given that US ethanol production is driven by a quota system, once that quota is exhausted, there is no more incentive to plant corn for ethanol and acreage will then be planted in the next most lucrative thing (which may not be corn). Corn for food production is then a separate market from corn for ethanol production.

            We can look to the South to see that when the the tobacco quotas were ended, acreage went fallow, rather than being planted. It’s not clear to me that the same would not happen with the least productive acreage if we end ethanol programs.

          • keranih says:

            RE: Impact of US ethanol program on world food supply –

            Prior to 2004-ish, it was a long standing complaint of those who did not support globalization to charge that free trade agreements impoverished the developing world because those agreements forced small inefficient farmers to compete with American farmers. To add insult to injury, some of the American export surplus was directly supported by agriculture subsidies,(*) and a lot of the rest (specifically livestock) was indirectly supported.

            The US got a lot of heat that others, specifically the EU, did not, because the EU worked on a quota system that essentially paid its farmers to farm, but only so many as were centrally authorized to produce that crop. This minimized EU shortfalls/imports (**) but didn’t allow for much export. So even though developing farmers were more shut out of the lucrative EU market than they were out of the US market, they were competing with the US more than the EU.

            For over a decade, globally, people were calling on the USA to reduce its farmer support, stop producing so much excess, and let the poor farmers of the world sell their crops fairly. The theory was that as farmers were paid more, they would buy more stuff and raise the economy around them. Whereas in the USA, agriculture was such a niche market that it didn’t matter much to the general economy if farmers lost business and some had to go out of business.

            Then the ethanol subsidies went into effect, US farmers responded to incentives (by delaying rotations with soybean and planting as much corn as they could) and the world supply of corn(***) dropped through the floor. However, instead of making developing countries richer by giving a better price to the farmers in developing countries, the situation only added to instability by making global supply swings more extreme – and likewise the price.

            (*) The history of US agriculture price supports is an interesting one, and could take up acres and acres of pulpwood. The short version is that aside from the notorious mohair, and honey, and sugar, nearly all subsidies are limited to major grains, with the aim of smoothing out supply highs and lows. As complicated as it can be to apply for subsidy support for corn, wheat, etc – these are long term well understood easily stored and graded commodity crops. Trying to get subsidies for, oh, tomatoes would be a nightmare.

            (**) A lot of EU shortfalls are a result of requiring farmers to stick to inefficient farming methods, so that production is offshored to locations that allow eggs from battery hens and pen-raised veal.

            (***) People don’t eat a whole lot of corn. But they do use a lot of corn starch, corn oil, and corn syrup – and lots of corn feeds livestock. When corn is not available to feed livestock, then wheat, barley, soybean, milo, cottonseed, and other crops are substituted. And that drives up the price of those other crops.

          • “It seems unlikely to me that we are currently producing as much corn as we possibly could in the world, so it strikes me that the reason so much corn is grown is that ethanol makes it lucrative.”

            Which it does only because it raises its price. Isn’t that obvious?

            Of course we are not producing as much maize as we could. If we divert X bushels of maize into biofuels, maize production goes up by something less than X because if it went up by X the supply of maize for food would be unchanged, the price of maize would be unchanged, and there would have been no reason to increase production.

            Since production is up by less than X, there is less maize then before on the food market.

          • James Picone says:

            @David Friedman:
            Yes yes reduces supply etc., but oil is substantially more fungible than food. Also, as I said in the previous comment, I was under the impression that there are not-directly-monetary reasons why starving Africans don’t import corn off the world market; specifically that they don’t have the relevant infrastructure set up and/or they can’t afford the cost of shipping the food over in the first place (so the actual price of the corn doesn’t really matter).

        • Chalid says:

          One issue is that some tax cuts can be crony capitalism, and many people who call themselves libertarians seem to reflexively support any tax cut they see.

          • Jill says:

            Yes, one of numerous examples about how Libertarians are theoretically against crony capitalism, but the effect of having Libertarians in American society is pretty consistently to enhance the position and power of crony capitalists.

        • Jill says:

          Yes, they do complain about crony capitalism. And then they almost always vote for the party that most supports crony capitalists. That’s the difference I was referring to in ideals vs. how a party functions in our society and politics.

          • onyomi says:

            “they almost always vote for the party that most supports crony capitalists”

            By that I assume you mean Republicans.

            Libertarians don’t “almost always” vote for Republicans by any means. Most of them vote libertarian or don’t vote.

            Perhaps you are confusing people like Glenn Beck with libertarians. The fact that more Republicans now claim to have libertarian sympathies doesn’t mean they are libertarians. It means libertarianism has had some measure of effect on politics, and not just the right. We were the first people talking about ending the drug war, for example, and are also more consistently anti-war than almost all democrats.

            Second, I don’t take it as at all given that Republican policies support crony capitalism more than Democrat policies. Remember Solyndra?

            I guess you’ll point to regulation, but remember the whole regulatory capture debate? There’s a strong case to be made that regulation (not “deregulation,” which I put in scare quotes because that word is used as a blue tribe bogeyman to refer to all different kind of things) is crony capitalists’ strongest weapon.

          • Chalid says:

            Libertarians don’t “almost always” vote for Republicans by any means. Most of them vote libertarian or don’t vote.

            I can’t seem to find any really data on this. This says that “14% of independents and 12% of Republicans said they are libertarian, compared with 6% of Democrats.” And according to this, around 80% of libertarians voted Romney in 2012 (see table 3) which suggests a strong Republican lean. Do you have anything better?

            I also feel like media and political campaigns *act* as if libertarians mainly draw from Republicans – see reaction to strong third-party runs – which is pretty suggestive.

          • onyomi says:

            I would agree that, of the two major parties, Republicans are more broadly sympathetic to (most of) libertarianism (not the anti-war, free immigration, sometimes anti-police aspects), at least rhetorically (it’s not cool in Blue tribe to go on and on about free markets and the Constitution).

            But this doesn’t mean libertarians, through the Republican party, make America worse. I think a theoretical Republican party without libertarian influence would be worse. Less concerned about their pro-business side becoming cronyish, even more hawkish, even more willing to be socially conservative…

            The fact that would-be Republican voters might be more likely to vote for Gary Johnson than would-be Democrats (though this is not at all clear; he is, after all, pro choice, pro gay marriage…) actually weakens Republicans electorally–the opposite of Jill’s claim–except insofar as Republicans modify their positions to attract more Gary Johnsonish voters. And, again, I think the Republican party (and the Democrat party) being more like Gary Johnson is a very good thing.

          • onyomi says:

            By the way, of the three presidential elections I’ve been eligible to vote in, I’ve sat one out, voted for one Democrat, and voted for one Libertarian. 2016 I will vote for Gary Johnson.

          • Chalid says:

            @onyomi I do agree that libertarian influence improves the Republican party. But it seems like Jill’s factual claim that libertarians “almost always” vote Republican is at least plausible – I find 80% in 2012 close enough to round up to “almost always”.

            I’m not sure about the idea that swing demographics are more influential due to being actively courted by the parties. One alternative story would be an “energize the base” story, and another story might be that when in office politicians reward their core supporters. I’d imagine it depends on the details of the situation – anyone know anything about this?

          • onyomi says:

            I personally don’t find the “80% of libertarians voted for Romney” statistic plausible, though some might say it’s because I have an overly doctrinaire definition of libertarian. I don’t think 80% of libertarians voted for anybody, given the significant proportion of libertarians who are against voting altogether.

            A problem with the increased popularity of libertarianism (maybe a problem inherent to any movement gaining traction), is the number of people who might claim some sympathy with libertarianism who yet vote for and support many extremely un-libertarian candidates and policies.

            Bringing up the example of Glenn Beck and his Tea Party followers, for example, it became popular for many of these “let’s respect the Constitution and the founding fathers more” types to self-describe as libertarians, but there is a huge gulf between them and people like Rand and Rothbard.

          • On the question of how libertarians vote, it depends on how you define “libertarian.” Hard core libertarians, judging by casual observation, either don’t vote or vote for the LP.

            Libertarians broadly defined, meaning people who tend to support less government involvement in both economic and social matters, are more likely to vote for Republicans than for Democrats, but it depends on the candidate and the election. The first time Bush ran, a large majority of libertarians (in that sense) voted for him. The second time it was still a majority, but a much smaller one.

          • Urstoff says:

            Why do you think Republicans support crony capitalism more than Democrats?

          • Douglas Knight says:

            Chalid, it’s worth noting that your second link defined libertarians by their views on political issues, in contrast to your first link, which defined it by self-identification.

          • Jill says:

            If you think that Libertarianism improves the Republican party by being a part of it, do you think this is true of the most powerful self-described Libertarians– the Kochs? Do they improve the GOP?

          • onyomi says:

            I don’t really consider the Kochs to be libertarians, and I don’t pay much attention to what they do, because it’s difficult to evaluate, given the degree to which they’ve become a liberal bogeyman.* I’ve definitely seen them supporting some ideas which are distinctly unlibertarian, but I’d still guess the world (also because they donate a lot to unrelated charity) and GOP (because they seem to pull them in a more a free marketish direction) are better off with them than without them.

          • Civilis says:

            Wikipedia defines Crony Capitalism as an economy in which success in business depends on close relationships between business people and government officials. It may be exhibited by favoritism in the distribution of legal permits, government grants, special tax breaks, or other forms of state interventionism.

            How do you determine which party most supports crony capitalists? Is there a way we can settle the debate one way or the other?

            As far as the Kochs specifically, read the following letter:
            https://www.washingtonpost.com/opinions/charles-koch-this-is-the-one-issue-where-bernie-sanders-is-right/2016/02/18/cdd2c228-d5c1-11e5-be55-2cc3c1e4b76b_story.html
            I understand that proves nothing, but nobody in this thread has offered any evidence at all.

          • Jill says:

            Wow the Kochs can talk the talk. I’ve read a lot about their doings though and have never seen any record of any instance of them walking the walk regarding that supposed viewpoint of theirs.

          • ” I’ve read a lot about their doings though and have never seen any record of any instance of them walking the walk regarding that supposed viewpoint of theirs.”

            Articles written by people who agreed or disagreed with that viewpoint?

            The Kochs have been active in libertarian circles for decades and spent quite a lot of money pushing libertarian ideas. David Koch even ran for VP on the LP ticket.

          • onyomi says:

            I appreciate the linked article, but mostly my brain just tunes out anything related to the Kochs in the context of political debate. The very name has become a signal, like “multinational corporation” or “neoliberalism” that the person has a major ideological ax to grind.

          • HeelBearCub says:

            @onyomi:
            Have you watched the videos of the last two shootings? They are horrific.

            The governor of MN, Newt Gingrich and Redstate are now basically on board with BLM. Maybe your Facebook feed is taken up not by “lefties” but by people who see that we need to make improvements in the nation’s police force? Or, even if they are lefties, maybe they are simply correct?

            Ironically (in the worst way), in choosing to target the Dallas PD those asshole(s) killed officers who had been doing the hard work of reform successfully, as Dallas PD has greatly and rapidly improved on inappropriate use of force in recent years.

          • Civilis says:

            I’ve read a lot about their doings though and have never seen any record of any instance of them walking the walk regarding that supposed viewpoint of theirs.

            The Kochs are bad guys. We know, because they support people we don’t like. Therefore, they’re obviously doing bad stuff.

            (Further quotes taken from the following article: http://www.theatlantic.com/politics/archive/2015/03/do-the-koch-brothers-really-care-about-criminal-justice-reform/386615/ which says a lot about just how the Koch’s walk the walk when it comes to criminal justice reform, which is one area that both liberals and libertarians agree on.)

            Holden, the Koch counsel, is a spike-haired Massachusetts native who once worked as a jail guard—seeing youths from his blue-collar neighborhood on the other side of the bars, he says, made a deep impression. He takes issue with the notion that the Kochs only want to pad their own pockets, pointing out that they take many positions “contrary to our short-term economic self-interest”: opposing corporate tax breaks; opposing ethanol subsidies, even though Koch is in the ethanol business; opposing the Cash for Clunkers program, even though the company makes auto parts; supporting the Keystone Pipeline even though it will not touch the company’s refineries.

            Those are specific claims. You can look and specifically see whether or not those claims have been met. I would suspect that if those claims were false, the article would have mentioned it, given how much play it gives to accusations against the Kochs by environmentalists. I would argue that support for the Keystone Pipeline is not libertarian, though one could make the case that the right of way for a project like that is a valid use for the government power of eminent domain. However, the rest of it is heavily anti-Crony Capitalism.

          • Civilis says:

            A problem with the increased popularity of libertarianism (maybe a problem inherent to any movement gaining traction), is the number of people who might claim some sympathy with libertarianism who yet vote for and support many extremely un-libertarian candidates and policies.

            The problem is that we’re presented with a small group of candidates for any particular office, and all we know about those candidates is written by two groups of propagandists, one for the candidate and one against. In this environment, it’s hard to know for sure what any candidate believes or will do when in office.

            While I have many strong agreements with libertarian philosophy, I am usually careful to make it clear that I am not a true libertarian due to my disagreements on foreign policy. Many people aren’t so careful, either in how they describe themselves or how they understand the various political outlooks. While they are stereotypes, the ‘left-libertarians are all stoners’ and ‘right-libertarians are all Randians’ stereotypes have evolved for a reason.

          • keranih says:

            @HBC –

            I’ve seen those videos. They show a person dying. For that alone, because each life is precious, they are horrific.

            I do not find them horrific because they are black men who are dying, or because they were shot by police, or because there is racial bias in this country.

            I question the narrative which has been built around these two shootings in particular – mostly because we don’t know the details yet, which may yet prove that the immediate popular narrative is entirely incorrect -, and I *deeply* question the narrative built around police shootings in general which insists that African American people are brutalized at an unequal rate by police across the country.

          • HeelBearCub says:

            @kerinah:

            The key question is around abuse/misuse of force, which you failed to even mention.

            Would you allow that the shooting of Walter Scott was horrific? Horrific not merely in a “someone was killed” way?

          • keranih says:

            HBC –

            No, I did not *explicitly* say “abuse/misuse of force” – because it is not at all clear from the information at hand that force has been misused. The narrative I question is the assumption that force was misused, when we do not have that information.

            It is not clear to me why you are bringing up Walter Scott – this was not a recent event (ie, was not one of the “two recent videos” you described as horrific) and has not been widely promoted as a cause for public concern.

            (Yes, I have an opinion(*), but I am more curious as to why you think I should have one related to Walter Scott when we were not talking about that particular case.)

            (*) I always have an opinion. Someone might be in need of a spare, and I would feel bad if they asked me to help them out and I didn’t have one I could give them.

          • Civilis says:

            If anything bugs me right now about the recent shootings, it’s the use of a police robot with a bomb to take out the shooter in Dallas. The other two police shootings are still at a stage where we don’t know all the details, and they seem to fit into the murk mistake fog-of-war that a lot of police shootings fall into. The use of a police robot with a bomb is on a whole different level.

            Why does a police department have ready access to a lethal explosive device? As far as I know, police, even SWAT teams, aren’t issued fragmentation grenades. Police have enough issues with non-lethal weaponry being capable of inflicting lethal wounds, and I can understand this, as anything likely to incapacitate a healthy athletic individual is going to have enough force to kill some people that aren’t in perfect condition.

          • Lumifer says:

            @ Civilis

            I don’t think you’ve been paying attention. Police militarization has been a thing for quite a while. This, for example is police. There is even a book on the subject.

          • Snodgrass says:

            “Why does a police department have ready access to a lethal explosive device?”

            Because a bomb-disposal robot often deals with a bomb by carrying out a controlled explosion – that is, putting a small bomb near the bomb and setting it off.

          • HeelBearCub says:

            @kerinah:
            Because I’m trying to find some clear example of something you wouldn’t object to being called horrific with the connotation I used. If that example is not agreeable, then we have to have a very different conversation than if it is.

          • keranih says:

            HBC –

            Please, I’m trying to understand. What connotation did you use? That “abuse of force” was the deciding factor in horrific/not horrific?

          • Civilis says:

            Police militarization, I know about. The local SWAT team accidentally killed someone a while back when carrying out an arrest when a gun went off. I’m well aware that cops now carry more sophisticated weapons, it’s that in this case of one guy, they didn’t use them. If they’re buying body armor and submachineguns, and they’re not good enough for this, why not?

            Explosive weaponry is a whole different order of magnitude due to the risk to bystanders. Yes, guns place bystanders at risk as well, as the NYPD has proved on at least one occasion. Since the MOVE fiasco, I’m not aware of any police in the US using bombs. Even the siege of the Branch Davidian compound didn’t use explosives, although the results in that case were still horrific.

            As far as civilian bomb-disposal robots go, from what I’m aware they tend to use very specially designed shaped charges if they use explosives, and most use water blasts to wreck potential devices. News reports indicate that the robot in Dallas was jury-rigged with a bomb made of C-4. I suppose a SWAT team might have had ready C-4 available for breaching doors which could be fashioned into a bomb.

            We have two cases. One in which a fight with a suspect with a pistol who was not stopped by a taser resulted in the suspect being shot while being dogpiled by police when he might have gone for his gun. The other, a case where we have a suspect with a rifle, they send in a robot with an improvised bomb.

            http://www.nytimes.com/2016/07/09/science/dallas-bomb-robot.html

          • HeelBearCub says:

            @kerinah:
            Is the Walter Scott killing a horrific example of police abuse of force? That’w what I am trying to get at.

            Edit: In particular, is it the nature of the abuse of force that causes shock, fear and disgust, not merely the fact that a death occurred or that the death itself was particularly gruesome.

          • Lumifer says:

            @ Civilis

            Explosive weaponry is a whole different order of magnitude due to the risk to bystanders.

            I don’t know about that. I think the important issue is the the overall escalation of the levels of force applied plus the willingness to use overwhelming amounts of force (which is a hallmark of the military).

            In the siege situations where the surrounding areas have been evacuated I don’t think grenade-level explosives are a big deal.

          • “and ‘right-libertarians are all Randians’ stereotypes have evolved for a reason.”

            I’m curious. Are there any statistics on what percentage of the people who self-identify as libertarians also self-identify as Objectivists? As ex-Objectivists?

            Rand certainly had a large influence on the movement, but I’m not sure how large, percentagewise.

          • keranih says:

            @ HBC –

            Okay, thanks, I think I see better what you’re getting at.

            is it the nature of the abuse of force that causes shock, fear and disgust

            In my mind, Walter Scott being killed did not rise to the level of horrific due to his death being a (very likely) misuse/abuse of force. To my thinking, the running/grappling/running nature of the interaction left some room for considering the shooting itself to be at the edge/just over the edge of appropriate escalation of force.

            HOWEVER, the (apparent) lying and attempted coverup of what actually happened – that I *do* find horrific and deeply troubling.

            In other words, I am less troubled by even a blatant error of judgement like shooting at an escaping non-violent suspect than I am by the attempt to cover it up. Making mistakes is one thing. Covering them up is another.

            Does this help?

          • Orphan Wilde says:

            I’m a centrist-libertarian who is Randian; I vaguely approve of universal incomes, but largely as an explicit pay-off for unconverted property rights.

            That is, if you own land and do nothing with it or to it, you’re paying property taxes to the rest of us as a sort of social agreement that we’ll leave “your” land alone in spite of there being no basis in the original ownership of that property. (The land was there before people were.)

            My ideal tax system taxes, therefore, land (but not property, critically; your land can be taxed, but not for the value of your house sitting on it) and intellectual property (which in information-theoretic terms, is also “found”, and whose basis of ownership, like land, is purely in excluding other people the use of it).

            *Shrug* I’m not right-libertarian; I don’t think your land is, in a meaningful sense, yours. I’m not a left-libertarian; your factory sure as shit can be yours. And I consider myself Objectivist/Randian.

            I think right-libertarians are less wrong than left-libertarians, though, or at least somewhat better-rooted in decently solid philosophical principles. Left-libertarianism is, as far as I’m concerned, mostly wishing for a universe with better incentive structures.

          • Civilis says:

            Rand certainly had a large influence on the movement, but I’m not sure how large, percentagewise.

            I think it’s a very small percentage of libertarians or ‘libertarians’, but, much like militant vegans and followers of certain exercise routines, if you meet one in an online political discussion, they’ll let you know.

            I don’t know about that. I think the important issue is the the overall escalation of the levels of force applied plus the willingness to use overwhelming amounts of force (which is a hallmark of the military).

            Overwhelming force, be it massive armed SWAT teams or ‘shock-and-awe’ style military attacks, are used because you want someone to realize that their cause is hopeless and surrender. (In the case of shock-and-awe, not the guy on the receiving end the bomb, but all of his buddies.) In the past, they’ve sent in bomb disposal robots with communication devices to persuade people to surrender. If you send the robot in with an actual bomb, you’re not looking to get the guy to surrender.

            From all accounts, the suspect wasn’t putting anyone in imminent danger, unlike the other two people killed (from the perspective of the police). I’m not entirely sure that what the Dallas police did was the wrong thing, but if it wasn’t a unique one-off situation it is a massive paradigm shift in law enforcement. I don’t want to see the FBI calling in an airstrike against the next Waco cult.

          • Lumifer says:

            @ Civilis

            Overwhelming force, be it massive armed SWAT teams or ‘shock-and-awe’ style military attacks, are used because you want someone to realize that their cause is hopeless and surrender.

            No, I think the point of the overwhelming force is to achieve your goals quickly and decisively with minimal losses. Surrender is nice, but all-enemies-are-dead is almost as good an outcome. It’s not psychological warfare.

            In this case, I think, what mattered is that cops have a certain… disinclination to try to capture a cop-killer alive and I don’t believe it’s a new phenomenon.

          • Anonymous says:

            No, I think the point of the overwhelming force is to achieve your goals quickly and decisively with minimal losses. Surrender is nice, but all-enemies-are-dead is almost as good an outcome.

            An inappropriate attitude for a police department to cultivate.

          • Lumifer says:

            An inappropriate attitude for a police department to cultivate.

            This is entirely correct and one of the reasons why militarization of police is a really bad idea.

            There is a bit of tension between “serve and protect” and “suppress and eliminate” : -/

          • John Schilling says:

            Explosive weaponry is a whole different order of magnitude due to the risk to bystanders. Yes, guns place bystanders at risk as well

            Minimum safe distance for detonating a pound of bare C4, per UN guidelines, 100 meters

            Surface danger zone for 7.62mm NATO M80 ball ammunition, 3662 meters.

            It is not unreasonable to expect that the police may be able to evacuate an area ~100 meters around a barricaded sniper. Clearing out everyone two miles downrange against a miss or ricochet from a SWAT countersniper rifle seems a bit less likely.

          • HeelBearCub says:

            @kerinah:
            That is helpful to understood your position. I have to say your position strikes me as, well the phrase that springs immediately to mind is, “morally repugnant”.

            Now, I’m not sure if we have the same understanding of exactly what happened. I see a fleeing, unarmed suspect running away exactly as fast as you expect an overweight 50 year old to run, suspected, at that point, of nothing more than running away from the cops and resisting arrest.

            He is 10 feet away from the officer when he opens fire, discharging 8 rounds aimed at his back, hitting with 5. Firing the last round when he is at least 20 feet away after a pause when it is first apparent that the suspect has been struck.

            There is no justification for that use of force. None. You can’t shoot people in the back merely for fleeing and resisting arrest, the fleeing being coincident with a stop for a malfunctioning taillight and nothing else.

            If we are at loggerheads over that, then every other conversation on this topic will be fruitless.

          • keranih says:

            I don’t think we are looking at the same thing.

            There absolutely are times when it is justified – and a service to humanity – to shoot a fleeing suspect in the back. If you don’t agree that such situations exist, then we are at loggerheads, but I think that most people would agree that there are occasions when that is the correct action.

            There is also a range of situations when it is reasonable but in error for a cop (or any other person) to think that shooting a fleeing suspect is correct. Where the cop’s assessment of fact is incorrect, but his judgement, assuming the facts to be correct, is right. This is a spectrum with more fuzz around the edges than most of us would like.

            I think we would agree there, yes? (please tell me if any of my suppositions are mistaken)

            However, there are things which are further out from being reasonable errors and are just flat wrong judgement. Under the actual conditions, despite how the suspect had already attempted to flee, had been (unsuccessfully) grappled with, and had attempted to flee againdespite that, there was apparently no justification for that cop shooting Walter Scott.

            Walter Scott was absolutely in the wrong for rabbiting, and for not taking his arrest like a grown up, and running away like an idiot teenager. But that doesn’t make him a threat to the cop or the public that needed shooting to stop him. The process by which the cop could have made a snap judgement about whether that threat did exist should have returned the assessment that no, he’s a stupid idiot who made me really mad, but he’s not a deadly threat.

            I can see, as I said, how the situation of a running confrontation could have led to the lapse in judgement that made the cop think that shooting Scott was appropriate. When the adrenaline monkey is on your back, jumping up and down, screaming and chewing on your head, you can can do really stupid things.

            However, I do not put that error in the same bucket as the (alleged) coverup and lying. To me, the lying and coverup are horrific, full stop. Cops will make errors. Errors are made all the time by many people. Failure of integrity of public servants – particularly cops who make errors is, to me, the far, far bigger problem.

            Had the coverup not occurred, I would have said, clearly not right, horrible training, why did he think that and would expect the cop to face the same charges as a homeowner who shoots a shadowy figure running away from a shed. Which is, imo, very different from driving up to someone on the sidewalk and shooting them through the window.

            I get the sense that you might not agree here. If you feel that putting more weight on the sin of deliberate lying and false witness than on the sin of impulsively striking in the heat of anger is “morally repugnant”, *shrugs* okay.

            I’ve been called worse.

          • onyomi says:

            Re. police militarization, a big part of it is giving a job to veterans, no? The war on drugs and the war on terror seem to thereby interlock in a cycle of crap. The problem is, what do we do with all those men now that manufacturing jobs are gone?

          • onyomi says:

            @Keranih,

            Setting aside whether it would be a service to humanity (because, frankly, I have a dim enough view of humanity to think there lots of people whose death would be a service to humanity but whom we would nonetheless not be justified in executing), what is a hypothetical scenario when shooting a fleeing suspect in the back is justified?

          • sweeneyrod says:

            @onyomi

            Someone running towards a building containing a detonator for a bomb they set up that will kill hundreds of people?

          • HeelBearCub says:

            @kerinah:
            I did not call you morally repugnant. I would ask you to consider the difference between that and what I actually said.

            I said your position, as I understood it, struck me as something for which the first phrase that came to mind was morally repugnant.

            And if you are a cop who shoots a fleeing suspect in the back for no reason other than that they are fleeing, that is horrific.

            The lying and the coverup is part of the indication that the horrific thing happened, but it is not the horrific thing in and of itself.

          • Lumifer says:

            @ HeelBearCub

            Are you talking, basically, about appropriate and inappropriate emotional reactions?

          • keranih says:

            @ onyomi –

            You really need a lawyer for parsing some of the details, and IANAL.

            But sticking to more realistic scenerios than the bombers – although that is more realistic than I wish it was –

            If Ted Bundy had rabbited from the traffic stop in FL where he got arrested, I doubt we would have heard a lot of disagreement with the cop’s judgement to shoot him. Not for the traffic stop, but because it was reasonable to think that if he got away, he would hurt other people.

            Likewise, a person fleeing the scene of a rape or murder and reasonably suspected of having committed one of those crimes, or a person under the influence of drugs who had attacked another person and then fled.

            If someone was going through a crowd and throwing acid/bleach on the faces of kids in school uniforms, and then fled, and did not respond to demands to halt, then shooting that person would be justified, so long as bystanders were not endangered.

            As I understand the general principles, one can use deadly force to protect life (limb/eyesight/etc), but not property.

            A lawyer could probably give exact examples of the cases for each of these.

          • HeelBearCub says:

            @Lumifer:
            You are talking about the fact that we appear to be debating the word “horrific”?

            I think that is essentially semantics. It’s not the actual debate that is occurring.

            The real debate is: a) whether a hypothetical act was immoral and extremely harmful, and b) how sure we are that such an act was committed.

          • Lumifer says:

            @ HeelBearCub

            Re horrific: reading your debate “externally”, it does seem that this is a pretty important word for you. Basically you found it unacceptable (thus, “morally repugnant”) that other people don’t find horrific what you find horrific. And “horrific” is an emotional reaction.

            The real debate is: a) whether a hypothetical act was immoral and extremely harmful, and b) how sure we are that such an act was committed.

            “Immoral and extremely harmful” are very vague words. If you want a reasonable discussion you need to specify what they mean to you. As to whether such an act was committed, the act was certainly committed, but are you talking about intent? Or something else?

            By the way, the cop in question is awaiting his trial for murder.

          • keranih says:

            @ HBC –

            You will note that I spoke of “labeling the action of assigning weight to different actions” as “morally repugnant”, and that I did not actually accuse you of name calling. As far as I’m concerned, we’re still discussing this courteously.

            People react badly and attempt to overup all sorts of things. The coverup is its own error, and must, imo, be held separate from the original action. That a coverup was attempted does not indicate the level of horror of that original action.

            And I generally agree that running away from the cops – while a cowardly and infantile action overwhelmingly associated with criminals who are attempting to evade the consequences of anti-social actions – is not cause for the use of deadly force, there are some actions by criminals which justify them being shot while attempting to flee.

            People do get caught beating people, or stabbing people, or raping them, or shooting them, at which point they often attempt to flee. This is not the same as running away from a traffic infraction, however, and the cops should not treat the two as the same thing. And generally they don’t.

          • onyomi says:

            @Keranih,

            Generally speaking, the legal line (and, imo, probably the ethical line in all but the most extreme cases) is that you can shoot someone who’s in the process of committing violence if they refuse to stop. That is, if you see someone stabbing or raping someone else and they refuse to stop, you can reasonably shoot them in the back (or front).

            That said, even a reasonable certainty that someone, if you don’t shoot them now, will probably commit a rape or murder sometime in the non-immediate future, is not enough justification to shoot them now. It’s just too problematic to execute people for things *you* think they’ll eventually do, since, well, who are you to be judge, jury, and executioner?

            Ethically speaking, it does seem like if a known serial killer is about to get away, it becomes a bit of a grey area, but legally speaking, I’m pretty sure I don’t want to empower the police to execute people they judge will later prove a threat, even though they’re not a threat right now.

          • HeelBearCub says:

            @kerinah:
            Go back and re-read. You did in fact accuse me of name calling ( “I’ve been called worse”), which is the only reason I made a point of distinguishing between the two.

            Legally, I don’t think you have a leg to stand on. I believe the general principles that govern legal use of deadly force by officers are “proportionality” and “necessity”. Your examples go to proportionality, but not necessity.

            In any case, we have neither of those in the Scott case, and I don’t see the how the Scott case merits examining the cover-up as exacerbating, even by your own standards. In what way would the absence of a cover up have made the killing of Scott warranted?

          • HeelBearCub says:

            @Lumifer:
            Kerinah is the one who wanted to parse what, precisely, was “horrific” about the shootings of Sterling and Castile. He is the one who made a point of splitting away the moral component from the gruesomeness component.

            I am interested in the moral/ethical component, and from that the practical component of what happens when we allow the state to commit these kinds of immoral acts with relative impunity. This is why I refocused the debate as I did.

          • keranih says:

            @ onyomi –

            Again, IANAL. This 2015 article (congruent with the Walter Scott case) and this older Volokh post both allow that it is permissible to use deadly force against someone who has just committed murder and is fleeing.

            Is this the most charitable option, which is to yell for the suspect to surrender themselves, and accept that the suspect may be lost if they do not yield to authority/their own better angels? No, it is not.

            (In Ghettoside, it was postulated that part of the reason for high crime rates among African-American communities was the lack of trust that the community had in the police, due to the police failing to solve a significant fraction of murders. I think “we don’t talk to the cops because they don’t find the murderers because we don’t talk to the cops to help them find the murderers” is a self-reenforcing cycle, but the point is that failing to stop bad guys has repercussions in the broader community.)

            This does not mean that shooting a fleeing suspect is equivalent to assembling evidence to convince a jury…but it’s not nothing, either.

            I completely agree that it is extremely worrisome, how much latitude we give police officers to use force (deadly and not) against citizens. I agree with those who would limit the scope of things for which one can be arrested – but I suspect there’s quite a lot of territory in play between different people’s idea of what should and should not be actionable.

            But the place to work on that is at the level of declaring things to be legal or illegal, not at the street level where we tell cops to stop people from breaking traffic laws, or withholding child support, or failing to register their vehicles, or stop selling addictive drugs.

            We would *like* that there be a zero level of fatal encounters between police and lawbreakers. I don’t think it’s rational to *expect* that. The rub is how we decide what’s acceptable and what’s not.

          • What specific acts are we protesting, to what specific end?
            It sounds like the Dallas police department massively reduced excessive force complaints without causing an increase in crime. I’m all on-board for that.

            Unfortunately almost all of these discussions seem to involve a “y’all racist!” line that disables any rational thought process on my end. Ex: The only active BLM activist I know protested 4th of July because it’s an example of white oppression.

            Then of course I am told that I need to be open to discussion of the unique experience of People of Color and blah blah blah.

            Other people I know with direct affiliation with BLM have similarly dim views of the movement. An example is a liberal activist protesting friend of mine, who did a protest with BLM, and hated it, since they deliberately provoked the police, who responded with aggressive crowd control measures.
            According to him, he’s never had bad dealings with police in all his years of protesting.

            This signals very strongly to me that the various people interested in controlling police are simply the opposite tribe in the culture wars and wish to do me harm. Why listen?

          • keranih says:

            @ HBC –

            For the record, and your closer reading, it’s ‘she’, not ‘he’.

            Actually, I think you’ll find that yes, it is in fact permitted for police to shoot fleeing suspects – most particularly when they are armed, but that is not a requirement. The necessity is to prevent harm to others, as might be expected from a person who just shot a store clerk, or who had fired on police. For police, self-defense is only part of the equation – there is also the rest of the community. So your stance that it is “never ever permissible” and “always unacceptable” is not correct.

            You also seem to be attempting to assume that I see the Walter Scott shooting as justified – ie, excusable, or even warranted – ie, necessary, when I have not said any such thing.

            Something does not have to be appropriate or correct in order for attempting to cover up that thing be worse than the original action. I think that shooting Walter Scott was an error, and while I could see how that error could be made, my understanding/empathy for the officer doesn’t change that error to a correct decision. As it stands, the error is likely to be found a criminal fault, but the jury will decide that.

            As noted elsewhere – the officer involved was arrested for murder four days after he shot Walter Scott. This is not a case that was swept under the rug –

            – in no small part because of the bystander video –

            – but it might have been, because of the officer’s alleged lying and coverup. That is the most serious immoral action, imo.

            The shooting of a fleeing suspect might be morally right or it might be morally wrong. Dishonesty in reporting what happened is morally wrong, full stop.

            And so long as we confuse moral/immoral with pretty/horrifying, we will not come any closer to making correct choices when it is our turn to make a call.

          • HeelBearCub says:

            @kerinah:
            Sorry to have mis-gendered you. I defaulted you to male, unfortunately, so I apologize.

            I did not, in fact, argue that there can never be a reason to shoot a fleeing suspect. I said that it was wrong to shoot Scott:

            I see a fleeing, unarmed suspect running away exactly as fast as you expect an overweight 50 year old to run, suspected, at that point, of nothing more than running away from the cops and resisting arrest.

            That is the facts of the shooting as I see them. That is what I am categorizing as both immoral and illegal. You seem to have no issue with it. It seems to me that this is either extremely motivated reasoning or something else that I am at a loss to understand.

            Do you think that the officer could reasonably believe that Walter Scott was going to attempt kill someone?

          • keranih says:

            HBC –

            (on edit: apology accepted. Not a huge deal, but thanks.)

            That is the facts of the shooting as I see them. That is what I am categorizing as both immoral and illegal. You seem to have no issue with it. It seems to me that this is either extremely motivated reasoning or something else that I am at a loss to understand.

            I’m really at a loss at how you got “you have no issue with this” when I have been saying – at considerable length – the exact opposite. I’m really freaking annoyed that you’ve scolded me to go back and re-read when you’ve missed something I’ve said over and over again in *counts* three different posts.

            (It is possible to find something bad, and something else EVEN MORE BAD, which I supposed could lead people reading in haste to assume that the first was no biggie.)

            Again, to go back to the heart of the matter – you described two recent ‘caught on film’ shootings as “horrific”, because of (as you later said) the misuse/abuse of force. However, it is not at all yet clear that misuse/abuse of force was involved in those. So maybe “horrific” wasn’t the right word to describe them, particularly if your definition of “misuse of force rests” on shooting a misdemeanor fleeing suspect in the back.

            In particular, if we talk about errors, crimes, and misuse of power by the government, I think we should be very clear about whether something is done under the color of law or is done outside of accepted legal action. I think it’s pretty clear that the Walter Scott shooting was treated as an unlawful action from the start, so it is not correct to speak as “the state” is here conducting “immoral actions” without repercussions.

            One of the major ways that needful police and government reform is being delayed is because people are mixing together actions by police, actions by non-police, actions that are legal and justified, actions that are illegal, actions whose legality has not yet been determined, and demanding that the public as a whole accept the judgement that ALL of these are “horrific”.

            This is bad, this is not accurate, this is not rational, and I wish we would do less of this.

          • HeelBearCub says:

            @kerinah:
            Are you then retracting this?

            In my mind, Walter Scott being killed did not rise to the level of horrific due to his death being a (very likely) misuse/abuse of force. To my thinking, the running/grappling/running nature of the interaction left some room for considering the shooting itself to be at the edge/just over the edge of appropriate escalation of force.

          • Julie K says:

            I wonder how it would have worked out if the response to the various recent police-shooting-civilian incidents had been a non-racialized protest movement, instead of “Black Lives Matter.” I’m guessing it would have been more bipartisan (the right wing is not necessarily in favor of police corruption), but with less participation and media attention (toxoplasma effect).

          • John Schilling says:

            I wonder how it would have worked out if the response to the various recent police-shooting-civilian incidents had been a non-racialized protest movement

            Protests about Ruby Ridge, Waco, and other abuses of Federal law enforcement were non-racialized, but were politicized in other respects. Roughly speaking, the police are not going to go around shooting mainstream members of the dominant culture in a democracy, because duh. Which means whoever they do shoot will be part of most people’s outgroup, and any complaints will tend to politicize along ingroup-outgroup lines.

            On the other hand, the 1990s protests did result in the Feds reining in at least the more lethal of their excesses (now they just settle for taking all your money). Hypotheses that come to mind:

            1. Playing the race card is more polarizing than most other ingroup/outgroup distinctions in the US, hindering #BLM in building a consensus.

            2. #BLM has picked the wrong target – the police aren’t preferentially shooting black people, they are preferentially shooting poor people, and getting that detail wrong costs them support and effectiveness.

            3. Seventy-six people including more than a dozen unambiguously innocent children were killed at Waco, which is more of an atrocity-level Bad Thing than any of the #BLM shootings.

          • Hector_St_Clare says:

            Truthfully, I’m most likely to vote Gary Johnson (or else write someone in, or most likely of all stay home) this year. This isn’t because I like libertarianism. As I mentioned above I’m essentially ‘full communist’ on economic issues, and conservative on some of the cultural issues (abortion, immigration, generally sympathetic to ethnonationalism and suspicious of cosmopolitanism, etc.). That having been said, my number one issue is foreign policy (especially given the events of the last few years), and I think Gary Johnson is the most likely candidate to pursue a genuinely non-interventionist, antiwar policy.

            Which is basically what was mentioned above, scaled up to the national level: Johnson is a consistent libertarian who wants to allow free countries to be free and unfree countries to be unfree, and that’s essentially the deal breaker for me.

        • James Picone says:

          I think there’s a steelmanned claim here, but applied to the neoliberal part of the right – the cut-company-tax, remove-regulations, cut-taxes-on-the-wealthy thing. I don’t know what the best American example would be, but in Australia our previous (and likely next, although counting isn’t finished yet) PM, Malcolm Turnbull, is a good example. Went to the election promising a small tax cut for people earning more than $80,000/year and a significant company tax cut staggered over the next decade; obviously some libertarian influence, but not really a libertarian.

          He was also in charge of the Communications portfolio when the Liberal government started deliberately cocking up the National Broadband Network project, changing it from a fibre-to-the-premises thing to a fibre-to-the-node thing, which seems suspiciously like it was intended to cripple it so it wouldn’t eat Rupert Murdoch’s cable network – Rupert Murdoch being a significant donor to the right. There’s been a semi-recent scandal where the Liberal party looks like it’s funneling taxpayer money into Liberal Party coffers through a privately-owned company for collecting voter information named Parakeelia. There’s been legal chicanery around preventing the contractors running our tropical concentration camps being held liable for the deaths and injuries occurring under their watch. The Liberal Party’s climate change policy, which they refer to as ‘Direct Action’, is for polluting companies to make bids on how much CO2 they could not emit if the government gave them $x million, and then the government gives the ones they like $x million. Given that the leader of the Liberal Party at the time that policy came up doesn’t actually accept the science there, that looks suspiciously like they’re just setting up a mechanism to give taxpayer money to mining companies/power companies, look a bit like they’re doing something about climate change (which is an important issue to voters), and are able to cancel it at any time. Keep in mind that the previous government had set up an emissions trading scheme for CO2, which was dismantled in favour of giving-taxpayer-money-to-companies.

          That’s just what I can come up with now, and is all in the last three years. There’s a long and storied history of politicans that talk libertarian-ish talk on taxation and markets in practice setting up policies that are essentially kickbacks and quid-pro-quo to the companies they like; and that leads people to distrust libertarian talk. Sort of the same way people here are mostly skeptical of social justice talk because they’ve seen a history of hypocrisy there.

          And yes, some of that is because the right-wing is a coalition and the neoliberal end can’t always get what they want. Some of it is people with unsophisticated views making policy, and the sophisticated people/views don’t have a similar appearance of hypocrisy. But that applies just as much to the skepticism of social justice / left wing views we see around here, so I’m sure you understand why that’s not convincing to people in that headspace.

          • Tibor says:

            I always wonder whether it is better to have a left-wing party elected which will try to implement more socialist policies and everyone will recognize those policies as socialist or a right-wing party which will also implement more socialist policies (even if it is usually a different flavour of socialism) but whose policies will be regarded as “free-market” because their politicians use the free market rhetoric in their speeches.

            In the latter case, it is the libertarians (or “neoliberals”) who get the blame for the failure of those policies and the popular view shifts even more to “more state, more regulation, tax the rich”. A good example in Europe are the bank bailouts. Since for most people banks=capitalism=free market, bailing out banks, even though it is as far from laissez faire as flamethrowers are from pacifism, gets associated with libertarians. Somehow even bailing out Greece, which is a country and not a bank [citation needed], has the same effect (here I cannot even imagine how one can make the conclusion that a group of governments and international organizations effectively bailing out another government has something to do with the free market).

            If it is the left-wing party who is in power, libertarians are less likely to be blamed if its policies fail and the people then perhaps get more interested in the actual free market ideas.

          • onyomi says:

            This reason, along with, you know, Iraq, is why I consider Bush Jr the most net harmful president of my lifetime.

  34. Howie says:

    I think it’s worth an update to this post to clarify that the IGM Forum link really does not show whether economists are pessimistic about basic income. Instead, they disagreed that one very specific proposal to replace the entire “all transfer programs (including Social Security, Medicare, Medicaid, housing subsidies, household welfare payments, and farm and corporate subsidies)” with a $13k/year UBI for citizens over 21 would be beneficial.

    Many of the people who disagreed might be optimistic about, for example, a UBI with a different structure, a UBI + maintaining health programs, or a UBI in case of technological unemployment. The survey could just as easily have been described “economists are very pessimistic about the effects of eliminating Medicare and Medicaid.”

    Reproducing the whole question below for people who didn’t click through:

    “Granting every American citizen over 21-years old a universal basic income of $13,000 a year — financed by eliminating all transfer programs (including Social Security, Medicare, Medicaid, housing subsidies, household welfare payments, and farm and corporate subsidies) — would be a better policy than the status quo.”

    Among a panel of top economists:
    0% strongly agree
    2% agree
    19% uncertain
    42% disagree
    16% strongly disagree
    9% no opinion

    • cbhacking says:

      Or as
      “Economists are very pessimistic about removing all government aid for people not old enough to legally drink alcohol”, or “… about the ability of people currently dependent on government support to survive on $250/week”, or something like that.

      The proposed scenario is really obviously flawed (single-family homes? People who can only afford healthcare because of Medicaid? Retirees who haven’t been saving up an extra $13k/year and now get more than that from SS and Medicare?). It’s also strangely specific about some things (why is 21 the age requirement? Why $13k, is that really just what happens when you take the cost of those programs divided by your seemingly-arbitrarily-chosen population subset?) while also missing some important issues (is the UBI taxable income? Is SS and Medicare still taxed the same way even though the justifications for that system make even less sense now? Does this come with any other of the common UBI-related suggested changes, like abolishing or at least lowering minimum wages?)

  35. Houshalter says:

    >In order to counteract my (and maybe your) usual bias: here’s somebody fired for doing a study that found that some people were racist.

    This page now seems to require a subscription. Google Cache link: http://webcache.googleusercontent.com/search?q=cache:VkRcR4MieuMJ:www.theaustralian.com.au/higher-education/uq-economist-paul-frijters-quits-over-strife-from-race-study/news-story/72123ed709a0b4ed05a9584e9d4f8c44+&cd=1&hl=en&ct=clnk&gl=us

    Someone archive it if possible before google drops it.

    • Creutzer says:

      Note that this is from Australia, which, given its immigration policy, is likely to have a different cultural climate with respect to relevant matters.

      • James Picone says:

        For reference, racism in Australia is mostly about Asians and not black people; we don’t have many of the latter – we killed off a fair chunk of the Aboriginals and didn’t do the whole slavery thing, and we’re quite close to Asia in an economic sense as well as a physical sense. We’ve recently picked up the no-muslims thing as well, as demonstrated by Pauline Hanson standing for the senate on the policy “No more Muslim immigration” and not “no more Asian immigration”.

        Interestingly that switch happened well after we started throwing mostly-islamic, mostly-middle-eastern asylum seekers into tropical death camps. Very prescient.

  36. boy says:

    Re: Omar Mateen’s sexuality

    It wouldn’t be surprising at all to me if gay men in the area had false memories of seeing Mateen on Grindr and whatnot given that I’m sure there are plenty of similar looking Middle Eastern men out there, and the “homophobe turns out to be secretly gay” narrative is one many people are drawn to, gays in particular.

    But Snopes doesn’t address the fact that one of his fellow students in the police academy claims Mateen tried to flirt with him back in the day. Allegedly Mateen told him “If you were gay, you would be my type.” Seems to me that unless this guy is lying for some reason, Mateen was probably gay or bi.

  37. Earthly Knight says:

    Company that handles tech company interviews makes a feature that changes what gender an interviewee’s voice sounds like, to see if women get more tech jobs when the company thinks that they’re men. To the surprise of nobody who is paying attention, there is no anti-woman bias found and in fact women do slightly better when they are known to be female.

    The study does nothing to rule out the possibility that gender cues other than voice pitch and timbre were responsible for women receiving lower ratings. For all we know, the interviewers may have responded poorly to the use of stereotypically feminine diction or (more likely) to the expression of stereotypically feminine personality traits. It’s an interesting result, but both the author and all the press coverage I’ve seen wildly overstates the conclusions it allows us to draw. We should be more circumspect!

    • suntzuanime says:

      Well, if it’s true that women actually do better when talking in a woman voice, that seems to rule out Highly Sophisticated Sexism, since surely they wouldn’t be harder to detect when unmodulated. I guess it’s possible that employers get weirded out at hearing a male voice with stereotypically feminine diction and/or personality traits? Perhaps instead of disproving sexism, we have proven transphobia?

      • Earthly Knight says:

        Well, if it’s true that women actually do better when talking in a woman voice, that seems to rule out Highly Sophisticated Sexism, since surely they wouldn’t be harder to detect when unmodulated.

        Not necessarily. It could be that having a feminine personality is a negative regardless of voice pitch but that this is partly counter-balanced by the interviewer recognizing you as a woman. E.g.:

        –“Lacks confidence, not very assertive, too focused on interpersonal”: -20 points
        –Interviewer positively disposed towards women: +5 points
        –“We need more women in the industry”: +5 points

        I suppose this hypothesis is a bit lacking in parsimony, but it otherwise doesn’t sound crazy to me.

        • suntzuanime says:

          “Lacks confidence, not very assertive, too focused on interpersonal” sound like legitimate, non-sexist reasons to disprefer a candidate. Like, you might worry that it was code for sexism, but that was sort of the point of the experiment, right?

          • J Mann says:

            Well, that begs the question of whether interview confidence, assertiveness, etc. correlate with being a good tech employee – we could tell the story either way, I think

          • Earthly Knight says:

            This depends on whether lacking confidence, not being assertive, and being interpersonally focused predict poor job performance (or, more precisely, whether the interviewer justifiedly believes that they do). I have no idea whether this is true.

          • gbdub says:

            Keep in mind that an interview doesn’t assess job performance per se – it assesses how good you are at convincing the interviewer that your job performance would be satisifactory.

            Being confident and assertive would almost certainly help in that regard – most of the objective evidence of performance is on your resume and was assessed before the interview, the interview itself is all about selling it.

          • Randy M says:

            Also, someone being confident is weak evidence that they think they will be good for the job, which is weak evidence that they will be good for the job. Or at least that they have heard the oft repeated advice to be confident.

          • Earthly Knight says:

            Right. So if the following are true:

            (1) Women tend to come across, in virtue of biology or socialization, as less confident than men.
            (2) Interviewers give lower ratings to less confident applicants, other things being equal.
            (3) Men’s excess confidence does not reflect a difference in underlying abilities.

            …then there will be sexist discrimination against women in hiring, discrimination which can’t be thwarted by voice modulation. Again, I have no idea whether (1), (2) and (3) are true. But this hypothesis is consistent with the experimental results.

          • PDV says:

            Given the high dropout rate they see (and the studies mentioned), (1) and (3) are almost certainly true.

    • LPSP says:

      Literally my thoughts exact. I’m messed around with enough voices using filters to know that there are big differences in pacing, intonation and word choice in men and women, and that they sound odd when coupled with the intuitively-off pitch and timbre. Men have larger ranges in pitch but shift mechanically between monotones, women tend to move fluidly from word to word in a narrower range.

      • “I’m messed around with enough voices using filters”

        Wonderful. Have you considered filtering software for WoW, something that would give the dwarven characters in your raid proper deep dwarvish voices and produce similar effects for other races?

        • LPSP says:

          I’m confused, I’ve never played WoW. Never saw the appeal in it.

          • A raid is a group project, where a bunch of players cooperate to defeat bosses too strong for one to deal with. There is software that lets players in a raid communicate with each other by voice while doing so.

            It works fine as long as nobody is trying to stay in character, imagine that the game situation is real. But if you are trying, the fact that a male character is speaking with a female voice, that there is no difference among the voices of dwarves, gnomes, night elves, etc., is a problem.

            I was suggesting a solution.

          • LPSP says:

            Oh I know full well what Raids are. Friends of mine shared a flat with me in uni, played it constantly. I just didn’t see the relevance of WoW.

            Help me if I’m wrong, but are you suggesting I make software for roleplaying purposes? Because I’m flattered but I really didn’t mean anything so advanced in my comment. I’m only talking about stock warping effects you can find in common editors like Audacity, things anyone could use with 15 minutes of experience. (And of course, the crucial point I was trying to make is that women and men still sound different when modified; if you just change the pitch and nothing else for instance, men sound incredibly clipped at a woman’s pitch and women sound, best word I can find, “warbly”.)

          • I’m suggesting that someone should make voice altering software for gaming purposes. I suggested it for you only because you mentioned some familiarity with such software.

            From what you say, it would have to be more sophisticated than just altering pitch.

          • Loquat says:

            It would probably be fairly straightforward to come up with a unique filter to apply to each race – the Turian filter from Mass Effect, for example, is apparently pretty simple to duplicate. WoW has kind of a lot of playable races for that, though, and I don’t recall noticing much in the way of noticeable audio filtering on the game’s voice-acting. NPCs of different races seemed to differ more by accent and vocabulary.

            Now, if someone making a new MMO wanted to apply that idea from the start, that could be interesting…

          • LPSP says:

            It’s certainly a very good idea, although as Loquat points out it’s sort-of already a thing. The revolutionary application is handing the voice mod to the end user, rather than simply presenting the end results of an actor’s performance.

            Are you a player of WoW yourself, Mr. Friedman?

          • I’m not currently playing WoW, may possibly start again with the next revision, at least for long enough to see if I like it. But I did play for quite a long time.

          • Aegeus says:

            I think any attempt at realism in voicing will inevitably be undercut when the deep dwarven voice is shouting “50 DKP minus!” at the top of his lungs.

        • Anonymous says:

          Is there any overlap at at between even medium-core raiders and rp-ers?

          • Not that I can observe.

            I had thought of commenting that my proposal was only relevant to people who enjoy role playing, and most WoW players don’t seem to. On the other hand, even a small minority of players could be a lot of people.

          • Tibor says:

            @David: The lack of RP players was the reason I quit WoW after 2 months of playing, I think. Some other parts are quite fun, I liked the crafting system and the auction hall but after a while it gets repetitive. I guess it can be nice if you meet a group of RPers.

            I have also been interested in starting a pen and paper RPG with someone, I even know the right people. The only problem is that we are each hundreds of kilometers away from each other and skype sessions lack the charm…it is also a lot of work for the DM and currently nobody seems to be enthusiastic enough to do that work, especially not for a skype session.

          • I’ve been pushing role playing in a different context, the SCA, for something will over thirty years. Ir seems to be having some effect at this point, but not very much. If you would like to join the SCA and come to Pennsic, I have an encampment within which everyone has agreed to stay in persona when within the encampment boundaries.

          • Tibor says:

            @David: I imagine I would enjoy that, but it’s about 6500 km away 🙂

    • The Nybbler says:

      Nobody believed that voice pitch and timbre were responsible for women receiving lower ratings. The initial hypothesis was that sexism was responsible for women receiving lower ratings; that is, that interviewers were rating women lower because they were women, not because they had women’s voices. The women’s voices were merely the mechanism by which the interviewers could tell they were women.

      It’s certainly possible that changing their voices was insufficient to prevent interviewers from determining that men were men and women were women and then being sexist against women as a result. But that hypothesis certainly seems much less likely than the hypothesis that sexism was not actually the cause of the lower ratings.

      • Did they try the experiment with men being given women’s voices? That would test for the possibility that a conflict between voice tones and other signals resulted in a candidate being downrated.

      • Earthly Knight says:

        You’ve identified one way by which sexism might operate:

        Female applicants have female voices -> Interviewers recognize applicants as female by their voices -> Interviewers are ill-disposed towards women -> Interviewers tend to evaluate female applicants more negatively

        The study allows us to rule this pathway out. But here’s another way sexism might operate:

        Female applicants tend to express feminine personality traits -> Interviewers are ill-disposed towards feminine personality traits -> Interviewers tend to evaluate female applicants more negatively

        Note that the second pathway does not require the interviewer to correctly identify the gender of the applicant, yet is still a manifestation of sexism.

        • HeelBearCub says:

          The big takeway to me was that Women and Men who participated in more than two interviews were, on average, rated equally.

          It’s not clear to me, but the difference in overall ratings of men and women might simply be a result of fewer women staying on the site. The total composition of the female pool might skew to “less experience” (because they are dropping out, which the article mentions). Rating could be more of a function of experience on the site than anything else.

        • The Nybbler says:

          It’s not clear to me that being ill-disposed towards “feminine” personality traits is sexism. “I don’t like personality trait X, candidate is a woman and therefore likely to exhibit trait X, candidate gets downgraded” would be sexism. So is “Candidate exhibits personality trait X, therefore is probably female so I’ll downgrade her”. But “I don’t like personality trait X, candidate exhibits it in the interview, candidate gets downgraded” is not clearly sexist at all, even if trait X is more prevalent among women.

          • In re personality traits being more important than gender:

            In the Terman study (from memory), it turned out that people with more female personalities (I think the big deal was conscientiousness) lived longer, regardless of whether the people were male or female.

          • Nornagest says:

            Conscientiousness in the Big 5 sense is sex-correlated? I thought different aspects of it were differently correlated, leading to small differences at the top level. Of course, I suppose one of those aspects could be doing the work.

            I do remember Neuroticism and Agreeableness being higher in women.

          • I’ve not feeing ideally conscientious at the moment, but I was referring to something I read in The Longevity Project.

            I don’t promise that the book used conscientious in exactly the same way that the big five does.

          • Earthly Knight says:

            It’s not clear to me that being ill-disposed towards “feminine” personality traits is sexism. “I don’t like personality trait X, candidate is a woman and therefore likely to exhibit trait X, candidate gets downgraded” would be sexism. So is “Candidate exhibits personality trait X, therefore is probably female so I’ll downgrade her”. But “I don’t like personality trait X, candidate exhibits it in the interview, candidate gets downgraded” is not clearly sexist at all, even if trait X is more prevalent among women.

            This is going to depend in part on whether trait X predicts poor job performance. It’s also going to depend on the etiology of the interviewer’s dislike for trait X– if he looks down on people who express humility because he was raised in a society where humility is disvalued because it’s seen as a feminine attribute, that’s sexism, alright.

          • Julie K says:

            It’s not clear to me that being ill-disposed towards “feminine” personality traits is sexism. …

            But “I don’t like personality trait X, candidate exhibits it in the interview, candidate gets downgraded” is not clearly sexist at all, even if trait X is more prevalent among women.

            Does it *matter* if anti-trait-X-ism is sexism? Okay, legally and culture-war-ly it does matter, but should it matter?

        • “Female applicants tend to express feminine personality traits -> Interviewers are ill-disposed towards feminine personality traits -> Interviewers tend to evaluate female applicants more negatively

          Note that the second pathway does not require the interviewer to correctly identify the gender of the applicant, yet is still a manifestation of sexism.”

          I’m not sure how you are defining sexism.

          Women are, on average, worse than men at a fair number of athletic activities. Suppose you are recruiting for a basketball team on the basis of various measures of athletic ability, including height, but somehow are kept ignorant of gender. You are ill-disposed towards feminine physical characteristics such as being short because they result in people being less good basketball players. Is your recruitment pattern a manifestation of sexism?

          If your answer (like mine) is “obviously not,” why is your case different? By hypothesis, the interviewer prefers traits that correlate with having a Y chromosome–possibly (although not necessarily) because they also correlate with being good at the job he is recruiting for. If my first case wasn’t sexism, why is this one? Even if he’s wrong, his prejudice isn’t preferring men to women, it’s preferring (say) more assertive personalities to less.

          • HeelBearCub says:

            Sexism is not passing the ball to Nancy Leiberman because she has breasts.

            Yes, breasts may be correlated with poorer basketball performance, but knowing someone has breasts doesn’t tell you anything about their basketball talent.

            Not picking Nancy Lieberman for your team because she has breasts (and therefore no one will pass her the ball) is also sexism (indirect sexism, but still sexism).

            Breasts are correlated with all sorts characteristics that can lead to poorer basketball performance, but knowing the player in front of you has breasts doesn’t tell you how good a basketball player they are. If she has an excellent handle, a good jump shot, good lateral quickness, amazing court vision and supreme passing skills, you would be well served to pick her up as your point guard irrespective of her breasts.

            So, the question is, are people making inferences about ability because they note something that they think correlates with ability or are they directly judging ability.

            And here we get into tricky territory, because something like “seems not confident enough” may correlate differently with programming ability in female and male programmers.

            I actual don’t think the results in this “man with one study” actual support that kind of interpretation, but that seems to me like the steel-man of Earthly’s position.

          • Earthly Knight says:

            I’m assuming that feminine personality traits do not predict poor job performance. These being tech interviews, I thought this was a reasonably safe assumption, but if knowing that someone is humble gives useful, non-redundant information about their abilities, I agree that it would not be sexist to use it as a factor in hiring.

          • suntzuanime says:

            That might explain why women do better with undisguised voices. If a woman is humble, well, you know how women are, but if a man is humble, there must be something wrong with him.

          • Earthly Knight says:

            It’s certainly possible that interviewers react more negatively to “men” with gender-discordant personality traits than to women with gender-typical personality traits, yeah. Gender roles hurt men too, I guess?

          • suntzuanime says:

            It’s not a matter of it being a gender discordant trait, it’s just that if everyone knows that men will brag about their tiniest accomplishments, seeing your interview candidate not bragging will let you conclude that they haven’t accomplished even the tiniest of things, but only if they’re a man. Interviewers are trying to infer hidden characteristics from visible ones, and different characteristics might have different evidentiary statuses for different genders, legitimately.

          • Earthly Knight says:

            What you say is possible. But I don’t think I share your faith in the perspicacity of interviewers.

          • suntzuanime says:

            You don’t need much faith when all you’re asking for is better than chance.

          • Earthly Knight says:

            That’s not true. The difference in interviewer ratings between masculine men and effeminate men will still need to appropriately reflect the difference in underlying abilities. Assume for purposes of illustration that being competent in javascript and gender expression are probabilistically independent. Now, suppose that competent masculine applicants receive an average rating of 7/10, incompetent masculine applicants receive an average rating of 4/10, competent effeminate applicants receive an average rating of 4/10, and incompetent effeminate applicants receive an average rating of 1/10. Then expressing masculine personality traits had better predict future job performance at least as well as being competent in javascript, otherwise, there’s still gender role discrimination at work.

          • The Nybbler says:

            The slight gain for women when their voices were masked wasn’t significant. I think you’re probably going through a lot of effort to explain noise.

          • “So, the question is, are people making inferences about ability because they note something that they think correlates with ability or are they directly judging ability.”

            What does “directly judging ability” mean? In the interview context, you are making inferences about ability, which isn’t observable, from observable characteristics that you believe correlate with it.

          • Anonymous says:

            @David Friedman

            I think you could take that position further and argue that it’s never possible to do anything but infer. When you hire someone with a stellar track record, you are treating this as evidence that they will continue to do a good job when working for you. But since you can’t see the future, you obviously can’t know this for certain.

          • Earthly Knight says:

            @ The Nybbler

            That’s okay. The study we’re talking about used a convenience sample, so the effect it found is probably a will-o’-the-wisp to begin with.

    • Orphan Wilde says:

      What evidence would convince you there is no significant degree of sexism against women in technology? Is there any evidence that could?

      More: Is there any evidence that would convince you that you should start arguing that there is no sexism in technology?

      • sweeneyrod says:

        If 50% of software developers were women, that would be pretty strong evidence against sexism.

        • Creutzer says:

          You mean against anti-female bias.

          • sweeneyrod says:

            If approximately 50% of developers were women, ~50% would be men (presumably). So it would be evidence against anti-male bias as well.

          • Creutzer says:

            Only if you assume that the “natural ratio” (determined by ability and interest in the subject matter) in the absence of bias would correspond to the population gender ratio. This is exactly what is under debate.

            The steelmanned version of Orphan Wilde’s question is: Given the observed ratio, what evidence would convince you that it is due to a difference in the natural ratio and not driven by bias?

          • sweeneyrod says:

            Let me turn that question round. Is there any ratio that could convince you that bias does exist?

          • Creutzer says:

            I don’t think the observed ratio is much evidence for anything, insofar as what it is evidence for depends very much on your priors for innate ability, inclination, and bias.

            That is, after all, why people look at distributions of test scores to get an idea of ability and perform experiments with blinding to measure bias. Inclination is the hardest to measure, unfortunately. It is far from impossible to show evidence of bias with blinding studies and convince people: It seems quite clear, for example, that there was gender bias in the hiring of orchestra musicians before the wide-spread introduction of blind auditions.

            Your turn.

          • Orphan Wilde says:

            sweeney –

            Yes. A 100% ratio might suggest something seriously askew. Anything short of that?

            Talking as a developer – developers are, almost uniformly, broken people. I seriously believe some kind of brain damage is necessary to do this job. And I seriously believe that men, owing to the absence of a backup X chromosome, are just more likely to have the kinds of brain damage that lets somebody program.

            Programming, to a normal person, would be like reporting to an autistic asshole who pores over every piece of work you hand in for the smallest pedantry that can be complained about, then telling you, in an offputting mixture of excessive detail and “RTFM” vaguery, how you fucked up. Compilers are the most obnoxiously stereotypical nerdy person, and it takes somebody with the same personality type to deal with them. (Or a sufficiently broken upbringing that being treated that way by an inanimate object is fine.)

          • Cerebral Paul Z. says:

            OW: I used to like to say about my own job, “This is not work that just any idiot can do. It takes a certain special kind of idiot.” Incidentally, you’ve provided a fine example of Althouse’s Law: it’s OK to posit an intrinsic difference between men and women so long as you make it clear that women are better.

            Before long, development tools will have improved to the point where loudly sighing and saying “No, everything’s fine” will become a valid debugging technique.

          • lvlln says:

            100% ratio either way would certainly be strong evidence of sexism, but only when paired with the observation that there exist some members of both male & female genders who have both the qualifications and the desire to go into the industry. Those 2 facts would imply that the acceptance rate of one gender was 0% while of the other was >0%, which would imply some sort of sexism in hiring practices.

            Things are simple in absolute cases like that, because we know that the ratio of women who have the qualifications and the desire to enter the industry to the men who have the qualifications and the desire to enter the industry is not 0. We also know that it’s not infinite. But we don’t know much else about that ratio, and (0, infinite) is a really wide range.

            I would guess intuitively that the ratio is somewhere between 0.05 and 20, which would imply that any % in the tech industry 95% would imply some sort of sexism in hiring practices. But that’s based entirely on my intuition and should carry zero weight – for all I know (and AFAICT, for all ANYONE knows), the true ratio of women:men who are qualified and desiring a job in the tech industry is 1:99 or 99:1. In which case even 1% or 99% gender ratios wouldn’t imply sexism in hiring practices.

            TL;DR: The % gender makeup of the tech industry can’t by itself provide any evidence for or against sexism in hiring practices. It necessarily needs to be paired up with knowledge about the ratio of women and men who are qualified for and are desiring to work in the tech industry. AFAICT, we don’t have any good data on those population numbers, and without that data, asserting that any % (other than 0 or 100) of gender makeup in the tech industry implies sexism in hiring practices is simply wrong.

          • “Let me turn that question round. Is there any ratio that could convince you that bias does exist?”

            An interesting question, not limited to the particular context.

            When my sister went to Bolt (Berkely law school) about fifty years ago, women made up about ten percent of the class. One year, of the top two students in each of the three years, five of the six were women. I take that as pretty strong evidence for bias in admissions–that the women who were able to get in (or chose to apply–the bias could be by the applicants) were substantially more able than the men.

            Currently, law schools are about fifty/fifty by gender, which is consistent with the claim that the earlier ratio was due to bias of some sort.

          • sweeneyrod says:

            @Orphan Wilde

            Why does a 100% imbalance provide evidence of sexism, but a marginally smaller imbalance doesn’t? Doesn’t it make more sense to say that a 100% imbalance strongly favours the sexism hypothesis, a 95% imbalance favours it somewhat less, and so on, until you get to a 50% balance, which doesn’t favour it at all?

          • Orphan Wilde says:

            sweeney –

            No.

            A bias, in the case of this ratio, is a movement away from a natural ratio. For any values except 0-1 and 1-0, there is an equal amount of space to the left or to the right of any given coordinate (namely, infinite). Only for 0-1 and 1-0 is the bias space actually restricted.

            It’s easy to see using binary ratios; 1-1 is a 50/50 split. 10-1 is a 75/25 split; it’s a binary order of magnitude greater. 100-1 is a ~87/13 split; it’s a binary order of magnitude greater. And so on and so forth.

            Since the potential impact of bias isn’t actually impacted by non-endpoint ratios, we can’t narrow down what the present bias actually is, and thus we have no information about the bias. All evidence about what the bias actually is comes, not from the ratio, but from our knowledge of the world.

        • Orphan Wilde says:

          No it wouldn’t. As pointed out below, that would only be evidence against sexism in the case that that is the ratio of software developers we should expect absent sexism.

          • sweeneyrod says:

            A ratio that closely matches the population can be explained by two hypotheses — either there are no reasons that either gender would be disproportionately represented; or that there are multiple opposite reasons that would cause disproportionate representation if it wasn’t for the fact they happened to cancel out. The first theory is much simpler, and so more plausible. So in a world where the ratio of genders of programmers matched that of the population at large, the first theory would likely be correct, and there would be little sexism at work.

          • Orphan Wilde says:

            If your counterfactual universe has humans exactly as humans exist in this universe, and there are biological reasons for the gender balance as it exists in this universe, then the more complex explanation is correct. If your counterfactual universe has humans exactly as humans exist in this universe, and there aren’t biological reasons for the gender balance as it exists in this universe, then the simpler explanation is correct.

            Your scenario hides all the complexity of the evidence is consideration behind a “What if things were different” – well, they would be different. But we might not be talking about humans-as-we-know-humans, or even gender-as-we-know-gender. For all we know, you’re talking about a 50/50 split between things that don’t exist in this universe; your hypothetical darkens understanding, rather than illuminating.

            Or, in short, the facts in evidence remain in evidence.

          • John Schilling says:

            The first theory is much simpler, and so more plausible

            Occam’s razor applies only to unnecessary, hypothetical assumptions. If we are talking about the real world, or at least the real Western world, we must necessarily include the very non-hypothetical assumption that any vaguely prestigious or desirable field with a >50% male workforce will have its hiring practices deliberately attacked until the workforce is no more than 50% male. That feminist activism is more powerful in a particular field than sexism/sexual dimorphism, counts as one hypothesis in my book.

  38. Julie K says:

    Re: refugee children: Has a 6-year-old refugee been living in a war zone all his life, or were the first few years of life peaceful, and then the crisis came along that resulted in his family becoming refugees?

  39. Pete says:

    Regarding the refugee children study, I don’t really see a problem reconciling this with the “childhood stresses of poverty”, because IMHO they’re totally not relevant.

    In that study, you’re not comparing the effect of random kids from [poor_war_torn_country_x] spending their ages 1-5 in that place or USA. You’re looking at a very biased subset of this population, namely, families who could afford to cross the ocean to get to USA, who had the initiative to attempt that, and likely the local connections to do it safely. As the study abstract says “consistent with this finding if parents but not country are important for early childhood development”.

    I believe that we can reasonably assume that these families were, on average, significantly above most locals in wealth, education and socioeconomic status; so we *DON’T* “expect them to be living in a terrible deprived environment before immigration”. A family that’s “locally wealthy/globally poor” can, and will be expected, to care well for their children, and for preschool kids (unlike e.g. college age) the environment is just as good as they’d get in a richer country.

  40. sohois says:

    Re: the sugar study on mice

    I don’t have the skills to judge the accuracy of the study itself, but I can’t say that it’s super confusing to read such a thing; I have in the past read theories that are justified by this finding and would seem to provide an explanation. It goes thusly:

    The body is not a perfect thermodynamic machine as many traditional diets speculate, it does not simply take energy in and give energy out. Rather, it’s a mass of many different and competing systems, that have their own triggers for action. The sensation of taste can be one such trigger, causing certain enzymes or intestinal actions to take place that convert energy into fat or any other mechanism that could increase weight. So when you taste something good, parts of the body prime themselves to gain weight somehow.

    I’ve not really seen any evidence for this beforehand, but I think it’s a reasonable sounding theory that could be justified assuming this new study is accurate.

    • PhoenixRite says:

      ^ This.

      Another possible factor that came to mind: Even if the quantities of sugar water consumed were identical, could something be going on with “binging” vs. “sipping” behaviors by the mice, and the distribution of calorie intake over time, digestion, glucose level, etc. being affected by taste? I can munch on 300 calories of carrots one at a time all day, and then eat 300 calories of chocolate in 30 seconds.

    • Anonymous says:

      The sensation of taste can be one such trigger, causing certain enzymes or intestinal actions to take place that convert energy into fat or any other mechanism that could increase weight.

      This is more or less the basis of the Shangri-la diet (though its creator considers sweetness to be a sort of non-flavor).

  41. MetaJetta says:

    …. it means that an initial study about something in psychiatry conveys literally zero evidence

    That’s why meta-analyses are the always the best place to start looking if you want to know more about a given topic: https://twitter.com/metastudy

    • The Nybbler says:

      As the Devil’s Dictionary might put it

      Meta-analysis: A study designed to tell you where the publication bias lies.

      • suntzuanime says:

        A good meta-analysis these days will include a funnel plot or similar way to check for publication bias.

        Of course not all meta-analyses are good.

  42. Murphy says:

    re:Refugee children

    Could it be a case of survivorship bias?
    if a lot of the least healthy 0-6 year old children die might that bias the average outcomes of the survivors upwards and possibly hide negative effects from other things in the averages?

  43. JGFC says:

    ‘to the surprise of nobody who is paying attention’

    evidently I’ve been under a rock. Anyone got any links?

    • Jacob says:

      Bump. This result surprised me as well.

    • Virbie says:

      These aren’t directly related to tech but I think they’re the kind of thing Scott is referring to, since women in tech and women in STEM are often conflated (perhaps fairly) with each other:

      http://www.pnas.org/content/112/17/5360.abstract 2-1 hiring bias in favor of women for STEM faculty

      https://slatestarcodex.com/2016/02/12/before-you-get-too-excited-about-that-github-study/ Study claims that women on Github are better contributors but not recognized as such when their gender is revealed. Study is pretty flawed and doesn’t quite prove what it says on the tin, but tech news predictably runs with it.

      • Earthly Knight says:

        I want to emphasize that the Ceci and Williams study is large, well-conducted, and thorough, while the github and interviewing.io studies may as well be pieces of performance art. They shouldn’t really be mentioned in the same breath.

    • Jill says:

      I must have been under a rock too. Certainly Silicon Valley does not hire and promote women equally to me– except for a few tokens. But then Silicon Valley is reputed to be particularly sexist, as well as being absolutely the most ageist place on earth.

      • sweeneyrod says:

        Particularly sexist compared to what? (This isn’t a trap where I’m going to snarkily respond with some comment about Saudi Arabia, I’m just interested in the claim that Silicon Valley is especially sexist).

        • Jill says:

          Sexism Valley: 60% of women in Silicon Valley experience harassment

          https://www.theguardian.com/technology/2016/jan/12/silicon-valley-women-harassment-gender-discrimination

          WHAT SILICON VALLEY THINKS OF WOMEN
          http://www.newsweek.com/2015/02/06/what-silicon-valley-thinks-women-302821.html

          The Brutal Ageism of Tech
          Years of experience, plenty of talent, completely obsolete

          https://newrepublic.com/article/117088/silicons-valleys-brutal-ageism

          • sweeneyrod says:

            But what percentage of women in e.g. journalism, or medicine, or … experience harassment?

          • Jiro says:

            That doesn’t answer the question. Compared to what?

          • Orphan Wilde says:

            Eh. Women are so accustomed to being treated better than men that being treated like a man is interpreted as sexism; they’re being treated unusually poorly, from their perspective. Which is why the Internet seems so hostile to so many women; stripped of their gendered protections, they’re treated the same, and they discover it kind of sucks.

            This isn’t unique to women. Men interpret women being treated like men as sexism, too. Yes, there are studies; no, I won’t link them, because I’m lazy and I honestly don’t think any of my intellectual opponents on this matter will be convinced by any degree of evidence, as evidenced by the fact that they immediately started looking for reasons this study doesn’t matter.

          • Whatever Happened to Anonymous says:

            I mean, I am somewhat sympathetic to your position, but “Trust me guys, I’ve totes got studies, and those who disagree with me are too mindkilled to believe evidence anyway” isn’t a great sales pitch of your ideas.

          • Orphan Wilde says:

            Whatever –

            It’s more “I’m too lazy and don’t care enough to sell this idea, so if you’re interested, go to the idea store, check out the labeling, and buy it if you want.”

            There’s evidence both ways. It’s better if people do their own investigation anyways.

      • The Nybbler says:

        (Warning: culture-warry)

        This is IMO why the outcome of that experiment is not surprising. We in tech have been subject to a constant drumbeat about how racist, sexist, and ageist we are. And yet, despite the demonstrable demographic skew, only a few of us actually see this evidence of pervasive sexism, racism, etc. We’re told by these few that horrible things happen all the time but we just don’t see them. If we fail to buy that, we’re told we are ‘unconsciously biased’ against women and minorities based on studies which amount to mining the noise (and are often completely inapplicable to our field) and this barely measurable bias results in huge disparities in outcome. If we won’t buy that, we’re told our normal (or typically nerdy) behavior constitutes sexism and racism by microaggression and is driving women put of the field. And if we fail to accept that, we’re told our very dissenting presence is itself unwelcoming, so we should be fired.

        And then we look around at fields where men behave exactly the opposite of ways these few would have us behave…. and we see those fields have more women.

        Conclusion: The claims of pervasive sexism are just as unfounded they appear. So when a study comes around confirming that, it’s no surprise.

        • Garrett says:

          I work in such a company. My last department-level all-hands meeting (hundreds – thousands of attendees) spend the first half of the hour talking about diversity. The second half was spent on department goals, financials, etc.
          There were lots of assertions that diversity was valuable, but no useful evidence of what kind of diversity mattered, or what evidence existed to support its emphasis.

          • Evan Þ says:

            As a limited counterexample, my last mandatory HR training here at Very Large Software Company actually did go into why diversity matters: diverse perspectives help us keep in mind different sorts of users. For example, if we’re designing a user interface, a white American man might naturally interact with it in one way, an Asian woman might naturally interact with it in another way, and so on – and, ideally, the interface should support as many different interactions as it can and ensure that those it can’t still don’t cause disaster. I thought that was a surprisingly on-point and useful training.

            There were also several other sections about harassment, prejudice, etc; they weren’t quite as well-reasoned, but nothing was really egregious there.

          • John Schilling says:

            For example, if we’re designing a user interface…

            “For example”, or “In this one unique case and I can’t think of any others”?

          • Evan Þ says:

            User interfaces might be the clearest example, but it also applies to anything where people might interact with the product in different ways, or use it in different use-cases. For another example, if there’s one small additional feature which would open up a whole variety of new use cases for (say) Middle-Eastern women, then it’s good to have someone on the team who’ll think of that so we can add that small feature with high return.

      • Lumifer says:

        You know what’s ridiculously sexist? Schools!

        “76 percent of public school teachers were female” (source) which looks to me like a VERY unbalanced gender ratio.

        So if you want to judge sexism by outcomes (percent of female employees), I’d like to discuss Silicon Valley alongside school teachers, nurses, etc.

        • Jill says:

          So you think that large numbers of males are just dying to become school teachers and are prevented from doing so by a sexist system?

          And all those male doctors would really have rather become nurses and are prevented from doing so by a sexist system?

          Right.

          • Jiro says:

            I don’t know about nurses, but it’s certainly true that men who want to become teachers face sexism. Google is your friend: http://abcnews.go.com/Health/story?id=5670187 https://www.theguardian.com/education/2006/jun/09/schools.uk

          • God Damn John Jay says:

            Nurses and Doctor’s don’t really seem like comparable professions to me. There are a lot of nurses who are one generation out of the underclass. Everyone I know who went to medical school came from an extraordinarily well off / academically elite family, most of them having doctors for parents.

          • Do you think large numbers of females (with qualifications equal to the male candidate pool) are dying to become entry-level coders and are prevented from doing so by a sexist system?

            As you say, right. There are issues and nuances to explore in how careers get coded in particular ways and how those ways can change, but just holding up numbers as evidence of sexism in one case and then laughing off worse numbers because they tell a story you don’t like is not credible.

            And, for the record, how many cases of men being discouraged or openly prohibited from normal interactions with children (girls in particular) would it take for you to consider that there might actually be sexism in the teacher ratio numbers?

            There are many other reasons too, of course, like the fact that male teachers tend to get rapidly promoted to administrators, but again, how many cases of this would you need before you’d consider that there might be a problem here?

          • Winter Shaker says:

            I understand that our host’s position on the ‘women being driven out of tech by gender discrimination’ is that whatever is happening to cause there to be disproportionately few women in tech is happening well upstream of them applying for jobs in Silicon Valley or equivalent. I know he doesn’t want this post shared about too blithely, but you’re a regular here; you’ve probably seen it anyway. Section VIII of this sets out the argument. Trouble Walking Down The Hallway is also worth a read.

          • An unbalanced ratio means sexism, ipso facto, or it doesn’t.

            Should be relevant for voting booths:
            http://www.cawp.rutgers.edu/sites/default/files/resources/genderdiff.pdf
            We just need to make voting more male-friendly. Perhaps beer chugging and barbecue outside!

  44. LPSP says:

    I may be poor at reading research lit, so I couldn’t find any explicit detail of what sort of intermittent fast was performed in the study linked. Can anyone fill me in?

  45. Wrong Species says:

    Basic income proponents: how much taxes are you willing to pay in order to make it work? Because the only way it can work without eliminating those other programs that help the poor is through massive tax increases far higher than you are used to, something to the effect of trillions of dollars. People often play down the negative effects of tax increases but I think at that level you have to consider it a likely possibility.

    • Anonymous says:

      At least as much as the basic income.

      • suntzuanime says:

        Bingo. People who talk about it costing “trillions of dollars” don’t take into account the fraction of those dollars that are fully illusory.

        • They aren’t “*fully* illusory.” They are illusory on the average but real on the margin.

          Before the demogrant is passed, I pay (say) $20,000/year in income taxes on an income of $80,000. After, if I have the same income, I pay $40,000/year in income taxes, get a $20,000/year demogrant. That, I presume, is why you call the tax illusory.

          But before the demogrant, if I earned an additional thousand dollars I got to keep $750 (25% flat tax for simplicity). After the demogrant, if I earn an additional thousand dollars I get to keep only $500 (50% flat tax). So my disincentive to earn has doubled, which means I probably won’t be earning $80,000/year.

          • PDV says:

            “Flat tax for simplicity” is doing all the work there. This would obviously not be a flat tax, it would be progressive; that’s why some forms of it are called the Negative Income Tax.

          • suntzuanime says:

            “Disincentive to earn” is a silly way to frame taxation. You’re not being punished for working, you’re just being rewarded less.

            I agree that the secondary negative side effects of taxation still apply, but I don’t think most of the people who are like “oh no, taxpocalypse” have objections that sophisticated.

          • “This would obviously not be a flat tax, it would be progressive; that’s why some forms of it are called the Negative Income Tax.”

            That is not correct. A negative income tax can have a flat rate. It just applies that same rate to the negative income of people whose income is less than their personal deduction.

            Personal deduction $10,000, rate 50%, income zero. Taxable income -$10,000, tax -$5,000.

          • nm. k. m. says:

            Main problem with current welfare scheme in my country is that in some cases, poor people will have absolutely smaller income if they start working. They will lose benefits (structured on the assumption that if you are not unemployed anymore, you are monthly salaried, because that’s how things were in th 1980s), but today in the relatively unskilled jobs you’re paid on an hourly basis and the hourly rates are kind of low, and you don’t get even a guarantee that manager gives you as many hours of work as you’d wish / could / need to support yourself. Sometimes even the monthly salary might be smaller than the benefits.

            Get a job? You don’t get to keep anything, you *lose* your net total income (instead of keeping less than in some hypothetical scenario). You’d need pretty amazing work ethic not to care about that. That’s even worse disincentive to earn, and if negative income tax can get rid of that at the cost that the incentives are reduced for well-off people, well maybe it’s worth it.

          • Anonymous says:

            The only way to get rid of those pathologies is to consolidate the dozen different welfare programs, into one cash based system. But we can’t do that because then the middle class can’t enact in tedious detail who exactly they find deserving and what precisely they deserve.

    • Anonymous says:

      Laffer limit, obviously.

      • James Bond says:

        Growth will be slower at the laffer limit. So while it might maximize taxes for the year, it will hurt the economy in future years, ruining everyones standard of living.

        • Luke the CIA stooge says:

          ^^^This.

          I don’t know how many mentions and pieces on the Laffer curve I’ve seen that don’t mention the most basic idea that as the time-frame approaches infinity the fall point of the curve should get closer and closer to zero as the increased growth from lower taxation compounds.

          to use the classic example a taxation rate of 100 % could probably generate major revenue in the first year if you sprung it suddenly, how ever this would drop off to zero in the following years as people adjust their behaviour.

          Even assuming you get the 100% the first year within five years a 25% tax rate Will generate more revenue as the 100% tax rate will not generate any revenues the following years and 25×5>100.

          Thus over long time horizons a sustainable low rate will generate more revenue than a high rate. With “sustainable” being the key word.

      • “Laffer limit, obviously.”

        Obvious only if you put zero weight on the welfare of taxpayers.

    • keranih says:

      Not a proponent exactly – I lean against but find the argument that this is the most effective politically possible idea increasingly persuasive –

      I fully expect that I would not receive any net income (ie, that “my” GBI would be taxed right back away again) and that at the very least in the short run, the tax burden would go up to cover the transition.

      My hope would be that as (probably over a generation) people adapted to the idea that free independent adults were expected to use the GBI to create their own safety net, and that excess requests from the taxpayer would be denied (while charity, friends and neighbors applied themselves to direct intervention) we would see an increase in more sound economic decisions and that the general economy would pick up, demand for tax-payer-funded payouts would go down, and eventually the tax burden would decline.

      Of course, I also think it would be foolish to implement GBI without severely reducing the legal minimum wage, eliminating all or nearly all entitlements or other assistance, and changing some of the tax code, etc, etc.

      Done right, with backbones of steel and hearts of granite, I think this could work. Done badly, it will be far worse than what we have now.

    • Since UBI is universal , you can arrange for the median taxpayer to see no nett difference.

    • Well, I think a 40% flat tax would work pretty well. If we then use 2/3 of taxes collected for UBI we’d still have $1.19 trillion for the rest of the government with a UBI benefit of $7374 per head. So someone making 0 per year gets a check for that amount, someone making $18.436 pays no taxes, someone making $36,872 pays 20%, and someone making $100,000 pays 32% net taxes. And we have the benefits of a flat tax like shifting income between years not making a difference in taxation and a whole other lot of simplification.

    • Quixote says:

      current program costs, plus a 4% increase in marginal tax rates for income in excess of 10m, plus removal of mortgage income deduction, plus removal of ag subsidies, and a progressive inheritance tax on estates greater than 10m that scales from 25% at 10m to 65% at amounts greater than 200m.

    • James Picone says:

      Basic income proponents: how much taxes are you willing to pay in order to make it work? Because the only way it can work without eliminating those other programs that help the poor is through massive tax increases far higher than you are used to, something to the effect of trillions of dollars. People often play down the negative effects of tax increases but I think at that level you have to consider it a likely possibility.

      Quite a lot more, on the condition that higher brackets had equal or greater increases, and steps were taken to close taxation loopholes (particularly ones relating to the intersection company tax). I’d be unhappy if my income tax rose 20%, but I think a basic income scheme + healthcare + disability scheme would be worth it.

      Honestly my living conditions would barely notice an additional 20% of my money going to the government, and I know several people who would definitely notice an $x000/month basic income.

  46. benwave says:

    So, as far as I can see it the UBS would be the same as a UBI only the amount would be strictly tied to GDP? Or are there other differences that I’m failing to see?

    Assuming that that IS the case, I guess it would give everybody an incentive to increase GDP as a whole, although it would give for most people a very small incentive. Is there any research into how very small incentives change behaviour, something which could be relevant to this situation?

  47. Jaskologist says:

    Apropos of recent discussions about steganography (hiding messages in normal-looking files), see this. He does it in less than 150 lines of Python, and the comments showcase some other methods as well (such as easy ways to distribute zip files disguised as images over Imgur).

    • suntzuanime says:

      Neat! You wouldn’t want to use that method to sneak data past anyone who was actually paying attention, though. The data is just sitting there in plain sight undisguised, it’s just ignored by most image renderers. I wouldn’t put your subversive literature on imgur using this method.

  48. Randy M says:

    Oh God oh God oh God functional brain imaging studies are awful –

    I would like to see more discussion of what this means. What bodies of knowledge are based on these kinds of studies, and how much of that need to be rethought?

    • Earthly Knight says:

      Scott Lilienfeld has a book on this, called “Brainwashed: The Seductive Appeal of Mindless Neuroscience.” It’s alright, a bit dumbed down for my tastes.

    • Anonymous says:

      I’m as clueless as you, but have you noticed how all articles about the brain go something like “area X is responsible for speech because we notice activity there during speech”? No prediction, pure observation and in very broad terms too.

    • Joy Livingwell says:

      Anyone have the math chops to know how this issue affects the new study that identified 180 brain areas based on Human Connectome Project data from 210 healthy young men and women?

  49. I’ve read The Big Fat Surprise, which claims that the anti-dietary fat health claims were based in very shoddy research, and a high fat diet (and the fat should pretty much be animal fats) is very good for people. (Olive oil and tropical nut oils aren’t bad.)

    Most of the problem was a panic about men were having heart attacks at surprisingly low ages which led to jumping to the conclusion that dietary cholesterol led to blocked arteries– and then the panic was amplified by lobbying and sanctimoniousness.

    Anyway, this is the sort of thing I’d like to believe (the chapter on “the” Mediterranean diet is especially hilarious), but I’m having trouble gathering the patience to read a very long critical review which is, to put it mildly, not organized in terms of the importance of the arguments. Also, the claims of “plagiarism” from Taubes (using the same obscure sources isn’t plagiarism) make me think worse of the author of the critique.

    So, has anyone looked into this? Experimented with high-animal-fat diets, and if so, how did they work out for you?

    • BD Sixsmith says:

      I think Mr Yoder’s suggestion that Ms Teicholz has lifted material from Taubes is correct and relevant if, as he claims, she uses the same incorrect reference. This suggests that her research is sloppy at best.

      I think Christopher Snowdon was fair when he argued that Teicholz jumps dogmatically and opportunistically from evidence that saturated fat may not be as unhealthy as was thought to the idea that “saturated fat is not a risk factor for anything and should instead be viewed as a disease prophylactic”. In her book, for example, she claims that “a snack of full-fat cheese is better than fruit”. Better in what sense? What if I’m short of Vitamin C? What if I need potassium? What if I have been exercising and want something sweet? This is just weird carnivorous partisanship.

    • Lumifer says:

      I’ve poked around this subject. tl;dr It’s complicated :-/

      Basically, a diet high in saturated fat will (for some sort of average “will”, there is considerable individual variation) raise your LDL (bad) cholesterol, but it will also raise your HDL (good) cholesterol and will drive down triglycerides. This will give you a weird lipid profile which will make an average doctor unhappy and cause him to insist you need statins for the rest of your life. However there is considerable uncertainty in the medical profession about the consequences of having such lipid profile. There was an interview (I’m too lazy to go find it) by some high-profile paleo blogger with a well-known mainstream cardiologist and when pushed the cardiologist admitted that the high-LDL high-HDL low-trigs profile is not the same thing as the usual troublesome high-LDL low-HDL high-trigs profile and no one really knows how will it work out.

      And that’s just the lipid profile. There are also arguments about the effects on being prone (or not) to inflammation, about the potential to develop (or fix) the metabolic syndrome or, more specifically, glucose metabolism problems, etc. etc.

      See the tl;dr above.

      • Yes, this happened to me. Before my high fat diet (2011) :
        Whole Cholesterol 150
        Triglycerides 98
        HDL 33
        LDL 98

        Now (2016)
        Whole Cholesterol 242
        Triglycerides 73
        HDL 55
        LDL 172

        I’m not sure if I should be happy or concerned.

    • Randy M says:

      I eat several eggs a week, a fair bit of butter, red meat, & coconut oil (sat fat).
      My cholesterol numbers are dead center of the range, but I’m only mid 35.

    • Loquat says:

      I’ve been more or less low-carb and high-fat for a few years now – eating plenty of eggs, full-fat dairy, and animal fat helped me drop around 20 pounds and my cholesterol numbers are still well within the official “good” range. (Much to the annoyance of my husband, who prefers a more typical lower-fat diet and has noticeably worse cholesterol numbers, plus pre-diabetes.)

  50. Jacob says:

    >Review: “Nominal agreement between initial studies and meta-analyses regarding the presence of a significant effect was not better than chance in psychiatry, whereas it was somewhat better in neurology and somatic diseases.”

    Sounds like a lot of research is essentially Winner’s curse . Psychology comes out the worst in the study, but none of the other fields look very good either.

  51. onyomi says:

    Have not listened to the linked podcast, but the blogpost on accepting a mentally disabled child and the story about a Belgian town with a tradition of taking in mental health “refugees” seem like they might be of interest.

    Somewhat related cultural practice: I’ve heard in India they have a tradition of giving free food and, sometimes, lodging to anyone who shows up at the door claiming to be a spiritual aspirant. This seems to result in a lot more spiritual aspirants (and claims to be such), which most of the people doing this probably think is a good thing.

    • leooboiko says:

      In a lot of older, smaller societies, it was customary to give food and shelter to travelers and roamers of all kinds. In my (third-world) country, this used to happen in small towns within living memory – when I was a kid, vagrants would clap at home every so often to ask for lunch – but it was already in decline in the 90s, and now it stopped.

      I think the ethical framework underlying this practice is a variation of gift culture, based on the idea that “there’s such a thing as a free lunch”. More specifically, you don’t ever worry about whether the person asking for food deserves it, or whether they really need it or are treacherous fakes; whether they’re just lazy bums, taking advantage of your hard work, or else if they’re criminals, rapists etc. The only relevant points are that you have food, they don’t, and they’re asking some; they don’t have to pass an examination.

      I often wonder whether such a culture of “hospitality” could be resurrected in global, post-industrial, increasingly urban societies, and how.

      • onyomi says:

        My grandmother, who was a child during the Great Depression, claims that her relatively well-off family set up benches in their yard where they would daily offer free sandwiches to traveling “hobos.” She said that, as a child, her parents even let her talk to these itinerant men and sit on their laps. There was apparently no fear that they were crazy, violent criminals.

        A big part of the reason, of course, was presumably that a lot more otherwise sane, hardworking, respectable people became destitute during that period, whereas now, the threshold for “not destitute” being much lower, we tend to assume (obviously not always fairly) there’s something wrong with homeless people.

        But what is more surprising to me is just the high-trust nature of society as recently as 80 years ago. Sure, more nice, trustworthy people were homeless back then, but surely there were also *some* homeless criminals, psychotics, etc. Yet apparently my grandmother’s parents were just really not worried about that?

        • Ninmesara says:

          To me, offering hospitality/a meal to a “traveling hobo” feels very different than offering the same to a local homeless person who is expected to stay around. It seems that when you offer a local person hospitality you become more or less responsible for that person, and supporting that person becomes a long-term commitment. When you do it for a stranger, it is just a one time thing and your “responsibility” end when the live. But this is probably highly individually or culturally dependent.

          But now I’m curious: did your family use to set up a similar scheme before and after the Great Depression for local homeless people?

          • onyomi says:

            I do not think they did this before or after the Great Depression. I think it was perceived as a special circumstance when the presumption that honest, hardworking men should be able to find jobs and take care of themselves was temporarily suspended.

        • Error says:

          Perhaps the change has something to do with an increasingly urbanized society. In a rural village, it’s pretty safe to assume that anyone you don’t recognize is an itinerant. You can give them food and be fairly confident they’ll move on. In a city, though, a habit of giving out food to strangers may quickly turn you into the local de-facto soup kitchen, simply by word of mouth. That’s much less sustainable than giving travelers a free meal once in a while.

      • I’ve also wondered whether default hospitality to travellers is partly caused by people living in small stable communities without much contact with the outside world– travellers relieve boredom.

        The situation is kept stable by travel being fairly difficult– you don’t get overwhelmed by large numbers of travellers.

      • Lumifer says:

        You need what’s known as a “high-trust society”. Such societies tend to be highly homogeneous.

        • onyomi says:

          This is one of the things that drives me crazy about bland comparisons of the US to places like Sweden. It’s like, sure, what works for an extremely homogeneous population of 10 million people in an area the size of California should apply just perfectly to 300 million very diverse people inhabiting an area 30 times that size. Or, yes, it must be because of the way they do public schools or gun control that the crime is so much lower… nothing, again, to do with the small population of high homogeneity…

          • Anonymous says:

            Well, if someone believes diversity is good, then what works for 10 million homogeneous Sweden should work in a more diverse place as well.

          • onyomi says:

            Diversity means difference. Even if you think that it should be an end in itself and is inherently desirable, it doesn’t change the fact that any law, policy, program, etc. is going to have a harder time being well-tailored to a large, diverse group of people than a small, homogeneous group of people. Because diverse people, by definition, have different wants, needs, and priorities.

            It’s like saying, “having all different sizes and shapes of bodies is a good thing, which is why I believe in one-size-fits-all clothing.”

        • leoboiko says:

          The society I was describing is Brazil, by the way. It’s not quite homogeneous. <- this is the biggest understatement I've ever said in my whole life.

          • onyomi says:

            That is interesting. Do you find Brazil to be high-trust, even though it’s diverse, or does not being high-trust in general not prevent people from offering charity to strangers?

            By the way, I don’t think diversity and high-trust are necessarily incompatible, though I think there might need to be some degree of shared values, even if the language, food, ethnicity, etc. are different.

          • leooboiko says:

            @onyomi: The latter. As much as I value our diversity, I’ll concede the data point that we’re definitely a low-trust society.

            I think Ninmesara is onto something, in that this kind of support was meant for “vagrants”, specifically, like the traveler’s hospitality we read about in the Greek classics. Mom would probably refuse to keep feeding the local homeless if they showed up often. Perhaps the death of this custom is correlated with the end of the “vagrant” type; the kind of person who would roam between cities will now flock to the metropolis, attracted by low-requirements jobs.

      • Jiro says:

        I’m reminded of the idea that having kinship obligations to everyone like this serves as a poverty trap. Anyone who got some money would find themselves obliged to give it all away by giving people hospitality, with the result that needy people get hospitality, but nobody gets to escape poverty.

        • onyomi says:

          “having kinship obligations to everyone like this”

          Fortunately now the government has determined we have kinship obligations to everyone, so now we can trap entire populations in poverty.

          Sarcasm aside, it seems in general that charity is a double-edged sword, and the best way to gain the benefits without so many of the negatives, whether administered publicly or privately, is to apply individual discretion and judgment (which private charity probably does better–that said, private charity may be less reliable in less homogeneous, high-trust populations).

        • Nornagest says:

          I’m told there’s a thriving town in Mindanao whose industry is Manny Pacquiao.

          He still seems to be doing okay, but, of course, most successful people are not as successful as Manny Pacquiao.

        • My impression is that kinship obligations aren’t all that rare. What’s interesting is how the kinship obligations are regulated, how much is owed, and how much people who are doing well are allowed to keep.

          Also, I’ve wondered whether there’s a problem with the world that an obligation of some generosity gets turned (in some ideological circles) into an obligation of unlimited generosity.

        • Sfoil says:

          It’s not just an idea. I ran into this pretty frequently as a commander in the Army–some young soldiers couldn’t save anything because they always had a wife’s uncle or something who needed money. Career men had usually cut these people off but the decision was harder for junior soldiers who might need help from extended family in the near future.

  52. Ninmesara says:

    Scott, where exactly does it say that patients with beta hydroxylase deficiency are psychiatrically normal? I can’t find it in any of your links. On the other hand, this paper says that they have findings consistent with ADHD:

    In addition, affected individuals had a temporal-attention deficit when they were not on treatment. During an attentional-blink task, participants were asked to identify two digits, separated by a variable number of letters. Attentional blink refers to the deficit in processing the second digit when it is presented within 200-400 msec of the first. Accuracy in identifying the second digit was impaired in affected individuals not on treatment but performance improved with droxidopa treatment.

    This is hardly dramatic, but probably far from “normal”. It is more or less consistent with research on the role of the Locus Coeruleus on arousal and attention (for example, this paper, cherry-picked out of a google search, and I only read the abstract). Note that in these patients the distinctions between tonic and phasic norepinephrine don’t matter, because they have none.

  53. Manya says:

    Everyone knows that “millennials” are far left, but the truth is more complicated – really into gays, marijuana, and immigration, but not much different than older generations on support for the poor or on racial issues (wait, really?)

    So, basically, more libertarian? That would be nice.

    (I’m way too lazy to actually analyze data, but that’s what it sounds like at first glance. Er, looks like.)

    • gbdub says:

      I’m kind of wondering why Scott picked out just “support for the poor” and “racial issues”, because there are a couple other interesting ones where millenials are average to conservative when controlling for other factors: Big Business, Gun Regulation and Abortion Rights.

      Gun regulation and abortion rights seem to imply a bit of a conservative swing on those issues in the newest generation – the former could definitely be described by a libertarian tilt, but what about abortion?

      The racial issues one isn’t really surprising to me. Racial justice was the big issue of the last couple generations, as gay rights are for millenials. The boomers were the radicals on race, and I see little reason why millenials would be more liberal there – this isn’t saying that millenials are less liberal on racial issues, just that they mostly match older generations.

      • Mercer says:

        Yeah, the first thing that came to mind for me with regards to Millenials on race was not “Wow, what a small shift leftwards” but “Well, how much more left could they have gone?”

  54. Ninmesara says:

    Many people here seem to support a Universal Basic Income. What is the most convincing defense of basic income you’ve seen? I’d prefer peer reviewed papers, but random blog posts are ok. I’d be interested in a comparison with the benefits of a “normal” welfare state, with social housing, free healthcare and soup kitchens.

    • Quixote says:

      I don’t have a post to link but I think people typically come from a few main points:

      1) incentive effects. Any “need based” program will have either cut offs or phase outs which mean that at the margin someone would be worse off (or in the case of phase outs not much better off) for acquiring more income. This reduces incentive to work and to improve ones situation. This is bad for GDP long term and if you believe that people in pressing need can sometimes be shorttermist bad for people as well. Reducing perverse incentives is a big gain.

      2) reduction in dead weight loss.
      A little preamble is needed here. People often talk about costs in terms of taxes or absolute dollars, but from a macro perspective this isn’t what really matter. If I make 200k and spend it all that’s 200k into the GDP. If gov can costlesly move 50k from me to someone else and they spend all of it that’s still 200k into GDP. If te gov costlesly moved 100k the math works out the same (200k into GDP) even if my tax bill is now double.

      For most social programs the gov cannot costlesly move money from point A to point B. It’s actually very expensive. If a program is need based there needs to be an infastructure that assesses that need, people who review income statements look at bank acounts etc and determine that the need is real. This infastructure is not directly economically productive, they don’t make anything. The cost is a loss to society. Then you have people to review, you need internal monitors to ensure they are not corrupt. You need an audit function. You probably need an appeals process to deal with mistakes And you need a legal function to try the appeals. You probably also have a wide range of record keeping and compliance costs. All these costs are dead weight losses to society.

      A basic income doesn’t make decisions and so doesn’t have any of the above costs. When an economist (as opposed to a journalist) says a basic income would cost less they are referring to this savings and the avoidance of the economically non productive transfer desicion apparatus.

      3) there are also a lot of moral and aesthetic arguments. But my competitive advantage is Econ so ill leave te above two.

      • The incentive effects are still there with a demogrant. To fund it you need higher marginal tax rates.

      • “People often talk about costs in terms of taxes or absolute dollars, but from a macro perspective this isn’t what really matter.”

        A nice illustration of what is wrong with the term “macroeconomics.” The point you are making is straightforward price theory, aka microeconomics. It’s “macro” in the sense of considering the whole population rather than a single individual, but that’s normally true of price theory–the world wheat market is an example of microeconomics.

        Why I have long argued that the terminology ought to be “price theory” vs “disequilibrium theory.” Or even “equilibrium” vs “disequilibrium” theory.

  55. wintermute92 says:

    I’m unsurprised by the replication failures cited here, and feel vindicated by one of them.

    On the obesity thing: didn’t one major study (JAMA?) use strange weight-group bucketing that lumped “moderately underweight” in with “low weight healthy”? As I heard the story, they caught some healthy weight people in their “overweight” bucket, caught a bunch of chemo patients, eating disorder suffers, and other high-risk patients in their “healthy” bucket, and concluded that ‘healthy’ weights are more dangerous. (Also, failed to adequately control for common causes like “is on chemo” when looking at death risk and weight…)

    Selfishly pleased to see the “eyes make you prosocial” studies go down. Not for any object-level reason, but because that’s the result that first made me pre-replication-crisis suspicious of these kitschy social-psych results, and now I feel prescient.

  56. Spurlock says:

    In the sugar study, do we somehow know that the genes that were modified only affect sugar-affinity? It seems like there could be a genetic sequence that both predisposes you to weight gain (or weight gain from sugar) and also causes you to like sugar.

    • Aido says:

      From the article:

      “But here’s the coup de grace. To make sure the mice weren’t just resistant to obesity in general for some reason that has nothing to do with palatability, they altered the sugar solutions by adding a very small amount of fat emulsion, so that the solutions were once again palatable to all groups of mice. Now, suddenly, the mice that couldn’t taste sweetness got just as fat on the sugar solution as normal mice! “

  57. The Nybbler says:

    Did no one link the meat shower to the well-known exploding cow phenomenon?

    https://en.wikipedia.org/wiki/Exploding_animal

    • houseboatonstyxb says:

      There were exploding sheep in Far From the Madding Crowd with a lot of detail, including where to puncture them to prevent the explosion.

  58. Nornagest says:

    Why don’t people just Aumann-agree on how the conflict would probably go, and skip the part where they actually waste all of their resources fighting each other?

    Wasn’t that the plot of a Star Trek episode?

    • Kenziegirl says:

      A Taste of Armageddon from the original series. IIRC the “virtual war” had gone on for so long because it was relatively painless, so they didn’t have enough incentive to come to the negotiating table and hammer out a peace agreement. Kirk, bless him, said that’s idiotic and basically escalated the conflict, forcing their virtual war into the real world. Yes, I am a nerd 🙂

      • LHN says:

        Just how Kirk was able to reconcile his actions there with the Prime Directive (which, contrary to reputation, he was usually very concerned with abiding by) was left as an exercise for the viewer.

        • suntzuanime says:

          I thought the Prime Directive didn’t apply to sufficiently advanced cultures? They were on a mission to open diplomatic channels with them, it wasn’t Kirk who decided the Prime Directive didn’t apply.

          • Said Achmiz says:

            Indeed, the Prime Directive was for non-warp-capable cultures.

          • Dr Dealgood says:

            In TNG the Prime Directive meant that the Federation couldn’t get involved in the Klingon Civil War, at least not until the Romulans had already been shown to backing one faction.

            So at least some parts of the PD apply to warp-capable powers.

          • Evan Þ says:

            … or the Prime Directive had been expanded between Kirk and Picard’s times. (It pretty clearly was from a Doylist perspective, at least.)

    • FullMeta_Rationalist says:

      Why don’t people just Aumann-agree on how the conflict would probably go, and skip the part where they actually waste all of their resources fighting each other?

      Scientists should just Aumann-agree on the laws of physics. Given the replication crisis, experiments are clearly a waste of resources.

  59. Israel Ramirez says:

    I have a unique perspective on the sugar study. Back in the 20th Century when I was an active researcher, I had proposed that combining a sweet taste and carbohydrate calories leads to increased ingestion of sugar.

    My evidence was obtained by letting lab rats drink an artificially sweetened solution (saccharin) while I infused carbohydrate solutions into their stomachs (yes my experimental setup was as ugly as it sounds). Rats that had calories in their stomach when they drank the sweetened solution eventually drank substantially more than rats given plain water in their stomachs. The effect persisted for a time even after I stopped supplying carbohydrate.

    Liking for sweet taste, I had claimed, was partly innate and partly learned. After repeated pairings of sweet taste and calories, sweet taste strongly drives eating more sweet.

    So I would have predicted the result reported in that study.

    Anthony Sclafani’s genetic technique is certainly more elegant that the ones I had been using but he confirmed my earlier claim.

  60. Oort says:

    It’s not exactly “fully automated.” Their robots only replace the cooks, not cleaning, customer service, or management.

  61. Mercer says:

    On the Ross Douthat article, anyone else amused to see such Mencian undertones in the New York Times? The cosmopolitan tribes religion described as “Liberal Christianity without Christ”, their view of history as “a powerful caste’s self-serving explanation for why it alone deserves to rule the world”. He might as well be using the word “Brahmin”. I don’t know much about Ross Douthat, is this typical of his writing?

    • Anon. says:

      He has referenced both Scott and Moldbug in the past. He takes some ideas from that area, but he’s not a “full convert”. OTOH he is a convert to Catholicism, which plays a pretty big role in his writing.

      Also the “Liberal Christianity without Christ” thing isn’t exclusively Mencian, it’s a criticism that goes back 200 years.

      Amusingly, the NYT commentariat seems to absolutely hate his guts. I don’t think I’ve ever seen a positive comment on his articles.

      • Douglas Knight says:

        200 years? really? Liberal Christianity had hardly been invented back then, let alone had Christ removed. Quakers were liberal, but the Puritans were at the far opposite extreme. Ben Franklin was post-puritan, but not post-liberal-christian. Maybe Thomas Jefferson was post-liberal-christian, but since he was not post-puritan, that means something different than what Moldbug is talking about.

        • Anon. says:

          Maybe 200 years was a little bit of an exaggeration, but by the middle of the 19th century you can find these arguments (against atheists who embrace Christian morality, and the politics that results from that) both in Germany and England. A bit later (1888) you can find Nietzsche perfecting it in the section on George Eliot (who was influenced by Germans in the first place) in Twilight of the Idols.

          The specific form of contemporary liberal Christianity is irrelevant, the underlying basis is the same.

          • Mercer says:

            I attributed the notion to MM even though I read Twilight of the Idols recently, probably just because since MM introduced me to the concept I unconsciously decided it was his.

            Dyou recall what article he mentions Scott in? I remember I was originally linked to SSC through some piece I stumbled across on RealClear, and now I’m thinking it might have been one of his? I read a lot of things without necessarily remembering the author. Googling “Slatestarcodex Ross Douthat” gave me nada.

          • Anonymous says:

            Mercer: FWIW, he links directly to this blog in the NYT article in question – ctrl+f “outgroup”.

          • Douglas Knight says:

            Thanks for the example of Nietzsche!

            Moldbug disagrees with you: he says that the underlying form is important. I think that means that he disagrees with Nietzsche, or at least means something much more specific. The word “liberal” is important to his formulation. But I don’t know what Douthat means. He may well mean something closer to Nietzsche and the word “liberal” may not be so important to him.

        • ChetC3 says:

          200 years? really? Liberal Christianity had hardly been invented back then, let alone had Christ removed.

          Yes really. The “Christianity without Christ” criticism goes back to the 18th century at least.

          Edited to add an example. The phrase “christianity without christ” used in a book from 1772 with an authentically interminable 18th title:Three Sermons preached before the University of Cambridge occasioned by an attempt to abolish subscription to the XXXIX articles of religion; and published at the blah blah blah.

          • Douglas Knight says:

            Hallifax uses the same phrase, but appears to me to mean exactly the opposite. I’m not sure because I have trouble reading that document, both because of the typography and the abstraction.

          • ChetC3 says:

            You’re right, it doesn’t seem to be a clear example of the phrase being used in the same sense. I’ll need to keep digging. I know Unitarian and Universalist churches were already established among ancestrally Puritan New Englanders (like John Adams and his family) by Revolutionary days, but I don’t have good sources for that at my finger tips. Of course, Unitarian and Universalist churches weren’t then what they are now.

          • Douglas Knight says:

            Yes, I think Hallifax is talking about liberal Christianity. In which case, he isn’t talking about “liberal Christianity without Christ.” The liberal element is important to Moldbug, although I don’t really know that Douthat means anything by it. Maybe Hallifax is correctly predicting how liberal Christianity will transform, but I doubt it.

            I usually date the New England Unitarians to 1820, but that’s when they split from the rest of the Congregationalists, so they’re really quite a bit older. But the rump Congregationalists are today the next most liberal sect. The transformation of the Puritans into the modern Congregationalists is a big part of liberalization. But the Unitarians liberalized first and I think that they were the elite branch from the beginning, and Douthat is talking about elites.

      • Quixote says:

        As part of the commentariat that “hates his guys” I may as well state my reason. I actually think a lot of his articles are thoughtful and well argued. But the problem is that they mostly have conclusions that are disconnected from the main argument. They often follow te below structure

        Intro
        Some human interest flavor
        Nuanced argument
        The phrase “and therefore”
        We should act in the evolving economic interest of major donors to the Republican Party.

        The last line is fixed. It’s always the same regardless of the issue or the arguments in the “nuanced argument” section.

        • E. Harding says:

          “We should act in the evolving economic interest of major donors to the Republican Party.”

          -Is it my role to spell out Moldb*g is no Sheldon Adelson?

          • null says:

            This is irrelevant because the previous comment is talking about Ross Douhat.

            It is also curious which major donor you have selected to represent Republican major donors.

          • E. Harding says:

            “This is irrelevant because the previous comment is talking about Ross Douhat.”

            -This is way more true, then. But “major donors to the Republican Party” were Brexit fans?

            “It is also curious which major donor you have selected to represent Republican major donors.”

            -He’s one that Harry Reid respects. And he’s no Democrat.

  62. Tseeteli says:

    Re: Basic shares & changing definitions of poverty

    I think we need to split out two “poverty” concepts that have gotten entangled.

    One problem is absolute deprivation; people have so little stuff that they starve, freeze, or suffer some other physical harm. The resource-level needed to avoid this should be pretty much fixed over time. And this problem seems like something that we’ve mostly solved.

    A second problem is that there’s a level of relative deprivation that creates major psychological stress. The amount of resources that people need to avoid these problems seem to be a function of expectation and culture.

    My issue with basic-income proposals is that people talk about them as if we’re solving problem #1, when really, it’s problem #2 that seems to be the major constraint.

    We could (and generally do) ensure that people have enough resources to be physically fine. If our goal is ensuring psychological health (a worthy goal) then I think that the social-context we’d set up around the transfer payments is just as important as the actual amount of money that gets transferred.

  63. haishan says:

    I don’t understand why there’s not more support, even from weird nerds like the Wait List Zero people, for a free market in kidneys. At minimum some sort of reimbursement to kidney donors. Iran allows donors to be paid for donating kidneys, and they quite literally don’t have a transplant waitlist. Meanwhile, here in the US, we spend something like $40 billion of taxpayer money every year paying for dialysis for patients on the waitlist. My back-of-the-envelope calculation suggests that the government could pay $100,000 for a kidney, spend another $100,000 per person to vet them and make sure they’re not being exploited or whatever, and still save tens of billions of dollars.

    I get that normies are squicked out by paying for organs. And maybe it’s just not politically feasible. But those of us who don’t have moral objections to it should at least be trying to shove on the Overton window, no?

    • Jiro says:

      Markets don’t work well when the price that the sale items would fetch on the open market is very much less than the price that someone not in immediate need of money would have to be paid to be induced to sell them. This also applies to the sale of other things than kidneys.

      • haishan says:

        Do you have any evidence that this would be the case in a kidney market? In Iran, which is the only legal market on the planet, the market clears. And, again, in case of market failure the federal government would likely save money subsidizing the purchase of kidneys up to a very high price (dialysis costs Medicare almost $100k per patient per year).

    • sweeneyrod says:

      What are the impacts on health of having only one kidney (honest question)? Also, how fungible are kidneys? If having only one kidney e.g. reduces life expectancy by 5 years, people might question the ethics of encouraging poor people to sell their kidneys.

      • Knoll says:

        From WaitListZero’s site:
        “Due to the screening process, living kidney donors are healthier than the general population, and they remain healthier after they donate. The vast majority of donors live healthy, normal lives essentially unaffected by the risks of donation.”

      • haishan says:

        AFAICT, there’s very little in the way of long-term risk, and life expectancy and risk of renal disease in particular appear to be unaffected. There are some short-term risks during and just after surgery, as with any operation, which might raise some ethical concerns. Then again, there are also ethical concerns with the status quo — way more people die waiting for a transplant than would die or be seriously harmed donating.

        One of the proposals for nonpecuniary compensation that I’ve read about is to guarantee free health insurance for life for living kidney donors. That would likely allay any concerns about long-term consequences.

    • haishan says:

      A correction: Medicare spends $40 billion a year paying for dialysis in total. About $10 billion of that is for patients currently on the transplant waitlist; some unknown amount of the rest is for patients who were on the waitlist but became too sick to receive a kidney. That latter would also be saved, in the long term, if we established a market and eliminated the wait.

    • James Picone says:

      Same basic reason people feel squicked out by the merchant offering a dying traveller in a desert water at an exorbitant cost example from some post here a long time ago. While the merchant makes the traveller strictly better off – they get one more option then they had before that they don’t have to take – it triggers fairness concerns. People have a broad heuristic for finding ‘exploitative’ scenarios because of the incentives; sure maybe that traveller is better off, but maybe we don’t want a cottage industry of people encouraging travellers to walk off into the desert with insufficient water who can then be sold water by merchants who then pay a kickback to the people who encouraged the traveller to wander off into the desert.

      The Chinese government has previously been accused of harvesting organs from executed prisoners; that might give you some of the flavour of the incentives people are worried about.

      • Larry Niven had a series of stories along the lines you suggest, and I have a JPE article on the subject.

        • Jiro says:

          I think that article misses one point: Even in the absence of incentives, we wouldn’t want to punish crimes by giving them all a 1/X chance of execution, X depending on the severity of the crime. The rationale is that doing that would produce an average punishment that is what we want.

          Why in the world would we want the average punishment to match something?

          Obviously deontologists, for instance, would not allow such a thing, and utilitarianism should not just be taken as granted. But even given utilitarianism, surely there are methods of aggregating other than just averaging out the punishments? Perhaps you want each prisoner to be punished by exactly Y, and whether a punishment system matches the desired outcome should be measured by the sums of the deviations from Y for each individual–in that case, execution-average punishment obviously would be a bad idea, and that’s a lot closer to how most people think.

        • James Picone says:

          That’s an interesting article; but I kind of feel like “Maybe perfectly rational self-interested actors is not a good model for criminals” is a hypothesis that can explain the same observation. If, for example, criminals/basic human psychology underweights probabilistic influences, then making stuff consistent could be a substantially stronger effect on the perceived strength of the punishment (that is, the average human probably doesn’t perceive a 50% chance of getting -10 to be equal to a 100% chance of getting -5, and I think it’s plausible that serious criminals are extra-bad at that).

          “Why don’t we go for the strongest punishment possible, then?” because people like not executing people for minor offences a lot; money isn’t the only cost. Also false positives; presumably a perfectly-rational agent can go “Well there’s an x% chance I’m accidentally convicted of theft, maybe that sets limits on how nasty I want punishments to be”.

          EDIT: That said it’s 4 AM, I have somewhere between no and limited economics, and I didn’t read terribly closely; so apologies if this is covered or obviously stupid.

  64. Taradino C. says:

    Trump/Palin 2016: We’re gonna build a bridge to nowhere, and nowhere’s gonna pay for it!

    • The Nybbler says:

      What I took from that tweet is that Trump now opposes Newton and by extension, gravity. Levitation for everyone!

  65. Here’s my weird story about a little stable weight loss:

    I was trying out the bit in Eric Franklin’s Dynamic Alignment through Imagery about the area where the ribs attach to the breastbone. I realized I had a good bit of tension there which seemed to be related to anxiety, and I was able to let go of the tension. All I expected was that I’d be less anxious, and at this point, I can’t remember whether my anxiety level changed.

    What did happen was that I got clearer about satiety. My previous spectrum was [hungry, food is interesting, stuffed]. Now it’s more like [hungry, food is interesting, comfortably fed, stuffed]. I lost 10 pounds with no effort. and they’ve stayed off.

  66. Le Maistre Chat says:

    “Treasure-hunting is big part of Tibetan Buddhism, and monks inspired by mystical revelation will often go out and unearth treasures or manuscripts hidden by past saints.”

    Thanks, Scott. You’ve solved the long-standing mystery “Why is Monk a D&D class?”

    • Nornagest says:

      D&D monks are more Shaolin than Tibetan… but I suppose wuxia convention would have just about every religion kicking ass.

  67. Alia D. says:

    Re the fMRI link, give that and http://www.pnas.org/content/early/2016/06/27/1602413113.full it looks like we’re going to have to understand brain structure and background noise a lot better before we can build on fMRI studies

  68. BBA says:

    Felix Salmon argues that we need more elitism. Honestly my every instinct is in his favor and I agree with many of his premises (government by referendum can be disastrous, ask any Californian) but I’m still not convinced he’s right.

    • E. Harding says:

      I see nothing wrong with Californian referendums.

      • PDV says:

        For an example, the Bay Area housing crisis is in large part maintained by a referendum. Property taxes cities can collect are drastically limited, so the city governments have no incentive to allow value-creating development; their tax receipts will keep rising at exactly 1% per year, every year, no matter what gets built on the land. This creates perverse – and totally foreseeable – consequences.

        Government by referendum frequently has this problem; it leads to nice-sounding laws with crappy consequences. (See also: preliminary studies on the effects of the top-two primary indicate it did exactly the opposite of what it was intended to do; it has increased polarization.)

        • E. Harding says:

          At least Top 2 leaves California with only viable competitors in general elections, and there being fewer candidates unopposed.

        • Glen Raphael says:

          @PDV:

          [California] city governments have no incentive to allow value-creating development; their tax receipts will keep rising at exactly 1% per year, every year, no matter what gets built on the land.

          If you’re referring to Prop 13, this is incorrect. Property tax gets reassessed after any new construction and whenever the property changes ownership, which means if something gets built that makes the property worth a lot more then the government gets a lot more tax money.

          Indeed, the fact that property taxes for current owners can only increase by 2% annually (starting from a base 1% of assessed value) would seem to give government MORE monetary incentive to want new developments built that they can charge higher taxes on. Doesn’t it?

          • Nornagest says:

            Indeed, the fact that property taxes for current owners can only increase by 2% annually (starting from a base 1% of assessed value) would seem to give government MORE monetary incentive to want new developments built that they can charge higher taxes on. Doesn’t it?

            Yeah. But they don’t. So what gives?

            I think the issue might be that we’re treating “the government” as an agent here, but the actual agents are functionaries at various levels that may not have the same incentives. Every other element of the city bureaucracy has an incentive to allow construction because that means they’ll get paid tomorrow, but the people running the various inspection and permitting departments have an even stronger incentive to justify their existence, and the people with the power to actually change this arrangement were voted into office by soccer moms who want their property values to keep going up.

      • BBA says:

        WARNING: This comment may contain a chemical known to the state of California to cause cancer and/or birth defects or other reproductive harm.

    • ThirteenthLetter says:

      It’s the usual thing where most people wouldn’t truly mind rule by elites if the elites weren’t such tremendous incompetent assholes.

      • Jill says:

        Perhaps. But since power corrupts, families and groups who remain in power for generations tend to always be assholes.

        • Lumifer says:

          I think the controlling term here is “incompetent”, not “assholes”.

          • Jill says:

            Maybe for you, but not for me. Assholes in power can be very competent at keeping and increasing their power and money and screwing everyone else over. But that doesn’t mean that doesn’t mean that the people who are being screwed over want those people in power.

          • Evan Þ says:

            +1 to Jill. Some hereditary elites are actually incompetent (e.g. Nicholas II of Russia), but others can be very competent but selfish prigs. AFAIK the average hereditary elite person was somewhere in-between the two.

    • HeelBearCub says:

      I’m not sure he is using “elite” in exactly the same manner is those who criticize rule by “elites”.

      I think this is the heart of his commentary:

      … the parties themselves were very, very serious about the job of governing. Even when they were in opposition, they were still an important part of the democratic process. It was their job to run the country, and they approached that job as professionals, trying to do the best they could …

      My emphasis added.

      There is a spot going around for Johnson/Weld which has an embedded tag line “Good government is easy.” This drives me nuts. It’s like saying “Good programming is easy” or “Good engineering is easy” or “Good science is easy.” It is patently untrue.

      Doing government well takes professionalism, that is the kind of “elitism” we need, and the article argues for.

  69. Cody says:

    What a brilliant look at the state of our media/information consumption! What beautiful philosophy!

  70. Nomghost says:

    Re: Voice modulation study.
    I wish they would go into more detail about the modulation they’re using. They imply it’s more than a simple pitch shift, which is good, because if you watch this Youtube video at 1.5 speed you notice that the voices are still heavily laden with linguistic markers commonly identified as female (or perhaps as belonging to a certain kind of performative homosexual identity.

    The two example videos they give show exactly that. Even if a listener didn’t know they were talking to a woman, they’d still know they were talking to someone who used vocal fry, who stressed certain syllables and used little dips in pitch in the middle of words, etc. If their study hinges at all on the supposed gender-masking capabilities of the voice modulation (which I think it does), then they should at least try to rigorously demonstrate in a double-blind test that the modulation software works.

  71. Dan T. says:

    The Australian article is paywalled so I can’t read it.

  72. John Schilling says:

    Re: Used soviet tanks for the price of a nice car.

    If your budget will stretch to cover a somewhat nicer car, there are some more interesting options available. As with the tanks, be sure to consider the operating expenses and limitations.

  73. Sfoil says:

    Re: surplus Soviet tanks.

    You do have to arrange, or at least pay for, for the shipping yourself, which is a pretty substantial cost. Also, spare parts. Also, their weapons have to be removed or demilitarized. I’ve read in the past that the armor is also “demilitarized” by drilling holes through it, which doesn’t make any sense to me because an inch-thick steel plate with a hole drilled through it will stop a bullet just fine everywhere except the hole itself.

    The tanks themselves are probably pretty worthless except as show pieces. You’re paying a lot of money in shipping and fuel costs to haul around possibly tens of tons of steel armor, and they’re ridiculously cramped because of how much room is dedicated to the main gun and ammo storage. Stuff like the BRDM, BMP, and MTLB, on the other hand, have better parts availability, way more interior space, AND out of the box amphibious capability. You should have no trouble “driving” a functional vehicle of these types across calm water, although they’re not as seaworthy as a typical bass boat.

  74. Jill says:

    Trigger Warning. Views expressed in this post may be Left of Center. Please do not read this post if such views may horrify you.

    I find myself asking “What would I have to believe in order to believe that the refugee children study should have come out similar to the Cherokee study?

    Yes, as Scott mentioned “we expect them to be living in a terrible deprived environment before immigration but a much better one afterwards” is one thing, as Scott mentions. And also we would have to believe that the stresses and life conditions of living as a poor person in the U.S. on an particular Indian reservation, are equivalent to the stresses and life conditions of living in some kind of crisis in some country and then needing to come to the U.S. in order to get relief from that crisis.

    There could be numerous differences though. For one thing, do we know how poor these refugees were, if they were poor at all, financially or culturally? And do we know if they lived in a “China Town” or “Little Italy” sort of place once they arrived in the U.S. i.e. if they were still surrounded by people of their native culture or not?

    Certainly refugee stress could possibly be acute stress, which might be quite different from the generations-long chronic stress of financial and cultural poverty on an Indian reservation.

    We would have to have the typically American beliefs that money is all that matters and that people outside our borders are poor. So that the effect of coming to the U.S. is mostly just the effect of having more money. So when the Cherokee folks got more money by having a casino, then that would be equivalent to refugees having more money because they were in the U.S. But did they indeed have more money– or a higher standard of living, or living conditions that were better– in the U.S. than in their country of origin? Of course they did in the recent past, but what about prior to the crisis that caused them to immigrate– during the child’s pre-crisis life if they were 6 years old at the time of immigration? Maybe they were doing pretty well, culturally and/or financially, before the crisis hit them?

    This reminds me of people comparing black people here in the U.S., with all of the police bias and other discrimination here and the historical baggage of slavery, the war on drugs that was in some ways a war on black people etc.–

    Nixon official: real reason for the drug war was to criminalize black people and hippies
    http://www.vox.com/2016/3/22/11278760/war-on-drugs-racism-nixon

    comparing them to e.g. Chinese or Vietnamese immigrants who had a long history of being culturally rich in their own countries for generations, where they were not a discriminated against minority, and where they had numerous constructive role models who had lots of self discipline, were not addicts etc.– in other words, no cultural poverty whatsoever.

    • Jiro says:

      Trigger Warning. Views expressed in this post may be Left of Center. Please do not read this post if such views may horrify you.

      Stop doing this. This is insulting.

      • Zz says:

        I also find it very difficult to read any comment prefaced by that kind of petty sniping.

        Jill, I consider myself to be broadly leftist and I’ve recently stopped reading the things you post here. I know you aren’t posting for my exclusive benefit, but you ought to be aware that you’re probably putting off as many left-leaning people as right-leaning.

        • Jill says:

          Is there another Left leaning person, other than the 2 of us? Just kidding. I think I saw 1 or 2 others.

          Zz. that’s fine if you don’t read my posts. Maybe if fewer irritable and easily put off people, of any political stripe, will stop reading my posts, I’ll be better off. I’m gotten somewhat bruised, metaphorically speaking, posting on majority Right Wing site.

          And I am certain that no one has been as offended by my trigger warnings as I have been by some of the comments that have been directed toward me. If I can handle bruises, other people should be able to handle slight irritations.

          • sweeneyrod says:

            I’m “left leaning” (by virtue of being British). Your trigger warnings aren’t offensive, just somewhat cringe-inducing.

          • Zz says:

            Zz. that’s fine if you don’t read my posts. Maybe if fewer irritable and easily put off people, of any political stripe, will stop reading my posts, I’ll be better off.

            The thing is, I’m not irritable. What I am is conflict-averse, in the sense that I don’t like being part of an exchange where anyone is being insulting to anyone else.

            I’m sorry people have been rude to you, sincerely. I think people have been a lot more unkind to you than was ever necessary (or useful, except as a means of tribal points-scoring). But opening your posts with that trigger warning reads to me (and presumably to some others) as: “Yeah, that’s right, I’m a liberal. Come at me you conservative jerks!” That kind of challenge is likely to be accepted by people who, unlike me, aren’t conflict-averse, thereby increasing your chances of having the very kinds of interactions you’re trying to reduce.

          • Winter Shaker says:

            I guess I also count as left-leaning-because-British, and I too would suggest that putting in that sort of trigger warning comes off as patronising at best, and is more likely to harm your position than help it. You have been met with insults you didn’t deserve, but putting a pre-emptively insulting comment in a top level post before anyone even can respond to you is kind of against the spirit of this place.

          • Tibor says:

            So what is the purpose of that trigger warning? It is obviously not meant sincerely, motivated by concern for someone who literally might experience a psychological trauma from reading your post.

            The only likely interpretation of it is that it should be read as “eat this you stupid right-wingers!”. If you sincerely want to discuss things with people, this is extremely counterproductive. If you want to score tribe points and sneer at people, well, then it is not but then you might consider a forum where people don’t at least try to be charitable.

            I do agree that there are right-wingers here who make similarly irritating claims like you do. The problem is not criticizing the views you disagree with. The problem is treating them in a condescending way like you do or concluding that since they don’t agree with you they are obviously evil bastards. Comments appear here every so often which say something along the lines “well, the real reason leftists do X is because they hate freedom!” and I am as annoyed by them as I am by your post. But seeing those comments does not give you the license to do the same. If everyone started behaving that way, this would quickly turn into a standard facebook- or even worse twitter-like “debate” which is usually not worth taking part in.

          • Tibor says:

            @sweeneyrod,@Winter Shaker: I wonder what being British has to do with being left-leaning 🙂 I get that the contemporary UK has rather more left-leaning policies than the contemporary US but it does not follow that being British makes you left-leaning by default.

          • James Picone says:

            @Jill here’s another left-of-centre person who thinks you should stop doing the trigger warning thing.

          • sweeneyrod says:

            @Tibor

            Our whole political spectrum is a fair bit further left than the US’s, as far as I know. David Cameron could probably stand as a centrist Democrat in the US.

          • I recommend The Moon Is a Harsh Mistress for the concept of “funny once”.

            Also, one of the problems with being insulting and/or annoying is that you’re likely to put off people who you weren’t aiming at.

          • Tibor says:

            @sweeneyrod: I’m Czech. The Czech political spectrum is at least as left-wing as the British* but I am a crazy libertarian who believes that anarcho-capitalism is a potentially great idea.

            *well, it’s complicated – prostitution is basically legal (it is not illegal to be a prostitute, it is illegal to employ prostitutes…also, since it is such a grey area and prostitution is not legally defined as a profession, it is not taxed), nobody is trying to ban porn like in the UK, drugs are decriminalized like in Portugal (unlike in the rest of Europe), so that could be very vaguely described as “more left-wing” (also more libertarian), but gun laws are more liberal than anywhere in the EU and much more liberal than in the UK and there is a flat income tax, so that would be “more right-wing” (also more libertarian :D). At the same time (not libertarian any more), healthcare is about as socialist as in the UK, the state is the majority shareholder in the biggest electricity company, the railways are almost entirely state-owned (there are a few private railroad companies but all the tracks are owned by the state) and in fact the state even owns a brewery (Budweiser…not the American Budweiser…they keep having lawsuits about the name, but actually the German name of the town where the Czech Budweiser is made is Budweis). I should add that I think the beer from that brewery is not very good but I am biased towards a (private :)) ) brewery from my own town, Pilsner Urquell, which is the original Pils-type beer 🙂 I get the impression that Margaret Thatcher managed to privatize stuff like that (electricity companies, railroads etc.) in the UK, so in this way UK is probably more “right-wing”.

          • Hector_St_Clare says:

            @Tibor:

            The Czech Republic sounds like a pretty cool country, from what I’ve heard. One interesting wrinkle (and I don’t know how you would put this into your left-right spectrum) is that they appear to have the second lowest level of economic inequality in the world, tied with Sweden and behind Denmark.

      • Wrong Species says:

        Can we please stop responding to Jill? It isn’t helping.

    • Douglas Knight says:

      where they were not a discriminated against minority

      It’s these little details that make me think Jill a false flag operation.

      • Jill says:

        Douglas Knight, if you think I am a false flag operation, whatever that is, feel free not to read any more of my comments. Whatever it is, it sounds like an insult.

        It’s taking a lot less time to read this board lately, because I am no longer reading any comments by consistently insulting people.

        It’s amazing how incredibly insulting some people can be, and the whole time they seem totally sure that they are just being “rational”, as they are somewhat civil to people who agree with them, and insult everyone who disagrees– or appears to. The insulting folks have no idea what anyone really thinks or whether they disagree or not– since, the moment the other disagrees in any minor way, they are called a Communist.

        • Ilyusha says:

          Jill observes “It’s amazing how incredibly insulting some people can be, and the whole time they seem totally sure that they are just being “rational””

          It’s less amazing when one peruses rationalist doctrines that tolerate, excuse, justify (and even glorify) demagogic modes of rhetoric that are self-descriptively exclusionary, vicious, and vituperative:

          “Regulars … often engage each other in a vicious and vituperative manner that you should not try to imitate because you are not an accepted part of the group yet.”

          As Noam Chomsky reportedly has remarked:

          Already we can see who is going to be privileged by this narrative and who is not.”

          Uhhh … maybe it’s not you who’s privileged, Jill? 🙂

          Affirmation  The social epidemiology of ideology-driven abuse has motivated Scott A to observe:

          “Any community that is more than 20% [abuse-tolerant] quickly becomes 100% [abuse-tolerant] through a neighborhood-segregation-style effect.”

          Yep. Not an outcome desired by any rational person.

          • Jill says:

            Thanks for the info, Ilyusha. Yes, you are right, it is certainly not me who is privileged on this site. No wonder Diesach blew a gasket and said something or other that resulted in being banned.

          • HeelBearCub says:

            Jill,
            I’m guessing you know the AA definition of insanity, as I’ve seen you reference the serenity prayer.

            Isanity is to do the same thing over and over and expect a different result.

            As a fellow lefty on this board I will ask, do you think this aphorism might be useful to you in terms of how you approach dialogue here?

          • Jill says:

            Bear, yes I’ve considered that. However, I do not expect a different result. I know what this board is like. And I prefer to be authentic, rather than to try to shelter Right Wing people from the world. People want to live in their own bubbles. How would I be helping them or myself if I would cater to that? Each to their own.

          • Nornagest says:

            @Ilyusha — You’re John Sidles, aren’t you? I suspected earlier, but now I’m sure.

            If you’re going to keep trying to evade your ban, you might have better luck if you got rid of some of those stylistic tics.

          • Jill says:

            I do think that that is one reason why the country is where it is today. There are some very nice and good people who are conservative and Right Wing. But there are others who are very verbally abusive, arrogant and condescending.

            Most of us on the Left put up with that, just as if what they were doing were normal. Calling Obama the Anti-Christ, saying he wasn’t born in the U.S., saying he is a Muslim allied with terrorists. The fact that we put up with so much is why we may end up with President Trump in November.

            I know that the Left has some extremists of our own who seem to jump into calling people racist for little or no reason. And I do not approve of that either.

          • FacelessCraven says:

            @Jill – “The fact that we put up with so much is why we may end up with President Trump in November.”

            You’re saying the reason Trump may win the election is because the left puts up with rampant incivility from the right, presumably rather than pushing back? Not sure I’m reading this right, apologies if not.

          • onyomi says:

            I really don’t think you have a leg to stand on if you want to claim that the left “puts up” with more rhetorical abuse than the right in the United States. Maybe puts up with different kinds of abuse, but certainly not more. And certainly not less likely to lash out than the right wing.

            My entire facebook feed the past week has been nothing but Pokemon and BLM stuff. How do you think these friends would react if I were to post some sort of pro-police message (not that I want to, but hypothetically)? Being silenced for fear of intense social repercussions for expressing your views is a form of “putting up” with.

          • Jill says:

            Faceless Craven, yes, I think that is so to some extent. The bar for what is treated as acceptable from politicians, and in political discussions, has been lowered a lot over time. And Trump just pushed it down further.

          • Jill says:

            Onyomi, too many of us, whether Right or Left, let ourselves be silenced. So almost no one knows what anyone else really thinks any more, except when the other person agrees with them.

            It reminds me of the song This Masquerade.

            https://www.youtube.com/watch?v=i4s9wVRIXeU

            Lyrics

            https://play.google.com/music/preview/Tvn3qhxeapw2sp5hphka36ad7pu?lyrics=1&utm_source=google&utm_medium=search&utm_campaign=lyrics&pcampaignid=kp-lyrics

          • I’m not sure whether this counts as pro-police, but it might be interesting to post– it isn’t pro-police in general, it’s about the Dallas police having improved their policies and behavior quite a bit.

          • FacelessCraven says:

            @Jill – Thanks for the clarification. If you’d like to see a lengthy discussion on a similar topic from several months ago, I found one here. Beyond that, I think I’ll wait for the open thread.

          • onyomi says:

            “Onyomi, too many of us, whether Right or Left, let ourselves be silenced.”

            If this is what my Facebook friends sound like when they’re silenced, I’d tremble to imagine what they sound like when they’re expressing themselves.

        • Nornagest says:

          A false flag operation is an action by one party designed to look like it originated from another party, typically in order to stir up resistance to it or otherwise make it look bad. They’re common in conspiracy theories (e.g. “9/11 was an inside job”), but they do happen in real life every so often. In the propaganda realm, this is called “black propaganda”, and it’s discussed extensively in that Linebarger psyops book I recommended to you way back when.

          You may be more familiar with the phrase “concern trolling”.

          • Jill says:

            Thanks for the explanation “false flag.”.

            Thanks for reminding me of the LInebarger book. I thought I’d requested it on inter-library loan. I’ll have to call them and see whether my request was lost, or if it was received, as I have not heard back.

        • I think what “false flag operation” here means is that he thinks you are a non-leftist pretending to be a leftist in order to make leftists look bad.

          I think that is very unlikely. Lots of online posters make their own side look bad by posting insults and assertions instead of arguments, I suspect because their real objective is to feel good not to promote their cause.

          But I don’t think you are one of them. As best I can tell, you are honestly trying to argue for your views but in the process revealing a striking ignorance of the positions you are arguing against.

  75. Stuart Armstrong says:

    >LWer and Future of Humanity Institute scholar Stuart Armstrong is in the news for a paper written together with Google AI scientists detailing an exciting new avenue for working on AI safety based on designing intelligences that will not resist their own shutdown.

    Thanks for the shoutout! Just wanted to mention those ideas derived originally from the less wrong post here: http://lesswrong.com/lw/jxa/proper_value_learning_through_indifference/

  76. If you want to kill two birds with one stone, make the disbursement of your universal basic share once a year, solely from the voting booth. 100% turnout among those who need it, and a nice predictable yearly spending boom.

    • Evan Þ says:

      But then, a lot of people wouldn’t save enough money to pay September and October rent ten months later. And even if you use the same trick to increase turnout in May primary elections, people might not be able to save for five months.

      • Julie K says:

        If the main goal is to increase turnout, even the motivation of getting the monthly disbursement at that point ought to do it.

    • Is turnout actually *that* important?

      I’m imagining someone losing a year’s income because they’re sick on election day. Even if there’s an exemption, there’s still a risk of losing the income if the authorities don’t think the reason for not voting is good enough.

  77. DensityDuck says:

    RE: robot hamburger stand

    On the one hand, this is what victory in the “fight for 15” looks like.

    On the other hand, maybe people are underestimating the desire of rich folks to have poor people dance for their amusement.

    On the other other hand, “all burgers cooked and served by humans, no automation” will become a selling point in the mid-range burger industry (Five Guys, In-N-Out, Habit, etc.)

    On the gripping hand I really want someone to fix this up in a truck so that there can be a Burger Van just like the Ice Cream Van…

  78. akarlin says:

    Great moments in Donald Trump tweeting.

    No, I want to be associated with interesting quotes, and people, I have almost fourteen million people between Instagram and Facebook and Twitter, and all of that and we do interesting things. And I sent it out, and it certainly got your attention didn’t it?

    So, what’s the problem? 😉

  79. onyomi says:

    Police shootings won’t stop till we stop using police departments as revenue generators.

    Though I think the drug war is the bigger elephant in the room, I do think using police as revenue generators creates all kinds of bad incentives.

    I feel like there should be some way to disconnect police and local budgets from revenue collected enforcing laws, because this creates an obvious bad incentive. Not sure what, though. Maybe send all local enforcement revenue into some bigger state or federal pot from which the money is then redistributed according to population or something?

    • Anonymous says:

      Another factor that seems to be rarely mentioned are veterans’ preferences. If you have a large influx of ex-soldiers is it any surprise that you get a militaristic culture instead of a protect and serve one? That’s even before getting into latent brain injuries and/or psychological problems.

      • FacelessCraven says:

        got any data on the rates of shootings (all types, justified and unjustified) by veteran vs non-veteran cops? Google gives me nothing but current affairs news.

        • AlphaGamma says:

          And note that some non-cop veterans I’ve spoken to have said that they operated under stricter rules of engagement in Iraq/Afghanistan than some American police departments appear to be operating under.

          • Fahundo says:

            some non-cop veterans I’ve spoken to have said that they operated under stricter rules of engagement in Iraq/Afghanistan than some American police departments appear to be operating under

            I’ve only been to Bahrain but I’ll second this.

    • CatCube says:

      It could be simply put back into the state or federal treasury as a revenue source, to be parceled out by the legislature or Congress like taxes, etc. That would make it too distant from the people taking it to care about trying to maximize their intake, while the legislature distributing it won’t care enough to incentivize takings because it’ll be a rounding error to their budget.

      As a matter of fact, I was under the impression it took special wording in the laws permitting this to prevent this from happening, as monies coming in go to the general treasury as a default.

      • My understanding of the history of civil forfeiture, from something Bruce Benson wrote, is that a lot of states had laws designed to eliminate the incentive by sending revenue from forfeiture somewhere other than to law enforcement. The federal government then subverted that system by allowing federal law enforcement to adopt forfeiture actions by local law enforcement, channeling the revenue to the federal agency which then shared it with the local agency. That change was followed by a sharp increase in forfeitures.

        • brad says:

          If you go even further back, civil forfeiture seems to have arisen as an outgrowth of the customs system, which was outright designed as a system of revenue both for the government(s) and as pay for the officers enforcing the policies.

  80. Ilyusha says:

    SSC‘s notable neglect continues of cognitive research like Stewart and Thompson’s “Does comedy kill? A retrospective, longitudinal cohort, nested case-control study of humour and longevity in 53 British comedians” (2015).

    Of the 23 comedians adjudged to be very funny (score 8-10), 18 (78%) had died versus 12 (40%) of the rest; mean age at death 63.3±12.2 versus 72.3±14.7 (p=0.079).

    Within comedy teams, those identified as the funnier member(s) of the partnership were, on an adjusted basis, more than three times more likely to die prematurely when compared to their more serious comedy partners (HR 3.52, 95% CI 1.22, 10.1; p=0.020).

    In view of these epidemiological findings, elementary considerations of rationalism and conservatism alike dictate that comedic cognition is sufficiently mortal in effect, and high-risk in practice — and even alarmingly degradative of empathic capacity — as to raise the question:

    Ought humor to be assiduously minimized in one’s personal relations, and to the maximal degree feasible, purged from rationalist / conservative on-line forums?

    More broadly, what (and how) do rationalists think of humor?

    • Jill says:

      Very interesting data there. I imagine it’s a hard job to be a comedian. Maybe as hard as being slightly Left of Center and posting on this board, LOL.

      Seriously though, being a comedian is probably a high stress job, dealing with hecklers, dealing with having jokes fall flat sometimes in front of large audiences etc. I don’t think they get paid much either, or treated well by those who hire them, on average.

      Having humor in one’s life seems an entirely different thing, which is rather uplifting.

      • Ilyusha says:

        A pretty good film, that is all about comedic cognition. is Punchline (1988). This film stars Sally Fields as a middle-aged housewife, and Tom Hanks as a young MD pediatrics resident, who both pursue stand-up comedy as a path toward personal integrity. Their quests succeed about as well might be expected (no spoilers will be given).

        Punchline mostly flopped at the box-office, for the simple reason that audiences wrongly expected it to be funny.

        Watched as a serious case-study in the slow, risky, passionate evolution of individual comedic cognition, Punchline comes across as a d*mn good movie, that was about three decades ahead of its time.

    • Held in Escrow says:

      Does this adjust for the whole “died before they got unfunny” factor though?

  81. onyomi says:

    If paleo isn’t working for you yet, make sure you’re eating an authentic paleo diet!

  82. G says:

    To the surprise of nobody who is paying attention, there is no anti-woman bias found and in fact women do slightly better when they are known to be female.

    I was paying attention and found this surprising. Is there a rigorous study consistent with this result? They don’t give any numbers if graphs supporting this, and other easier-to-control studies (like resume response) do find anti-female bias.

  83. 0xBEEF Supreme says:

    It’s kind of shameful that somebody at Motherboard managed to write a whole article about rigor in UFOology without mentioning Stanton Friedman. This isn’t new. Friedman’s been debunking UFO hoaxes and maintaining the validity of other reports for years. API essentially doesn’t mention Friedman on their website. I have this suspicion they’re ignoring the past in order to have the opportunity to repeat it.

    Incidentally, I don’t believe in alien flying saucers. I’m undecided about Nazi UFOs.

  84. Tyrant Overlord Killidia says:
  85. PV van der Byl says:

    Re Ross Douthat: very good assessment about “global citizens,” but Ross should have added Enoch Powell to Kipling, Lawrence, and Burton. In addition to ancient languages such as Latin and Greek (Professor, University of Sydney at age 25) and Modern Languages such as Portuguese and Welsh (well, sort of Modern), Enoch mastered Urdu, the language of hundreds of millions of people from the sub-Continent. Some “racist.” He was the real cosmopolitan!

    • Snodgrass says:

      It is absolutely possible to be completely fascinated by people you also regard as intrinsically inferior; people get completely fascinated by dogs, cats and wild birds. “These people who cannot be convinced to work eight-hour days have a five-hundred-year tradition of amazing love poetry” (paraphrasing Burton) or “these desert savages are such spectacular warriors that they can defeat squares of well-armed British infantry using medieval equipment” (paraphrasing Kipling) are perfectly consistent things to say.

  86. Albatross says:

    On 6 year old immigrants: 1.) Real formal education begins after six. 2.) The third world toddlers receive better training prior to age six, the helplesses of American toddlers is unique. An immigrant six year old can cook, shop and has street smarts. And 3.) Immigrant six year olds not only more competent than native peers in all non-education aspects, they are old enough realize the opportunity they are being given. A six month old has no concept anything else exists. The six year old knows failure has consequences. Cherokee/American toddlers is a different comparison apples to oranges.

  87. Danny Vartan says:

    I think you should have defined “better” in your post about intermittent fasting. The benefit of intermittent fasting is that it makes you calmer and clearer, not that it helps you lose weight. Since stress contributes to disease I’d agree with your friend.

  88. James says:

    On the topic of conflict, sieges may represent a good microcosm/thought experiment. Actual battles are rather rare in at least the European Middle Ages–many campaigns consisted of siege after siege. The reason is exactly what you describe–the armies acknowledge the outcome of the event, and the obvious loser opts to not lose by simply not fighting. Real pitched battles occurred when either 1) both sides thought they could win, or 2) one side surprised the other. Sieges, on the other hand, occurred because towns, cities, and fortifications can’t run away. They’re right there, and can only be conquered, victorious, or destroyed (by either side). It was MUCH safer for the attacker, because as long as you could stay out of bow shot there really wasn’t much the city could do to hurt you. The defender always had the hope that an army could relieve them, or that the attacker would run out of supplies/men/will to fight before the city did.

    Basically, a siege was a battle of wills with weapons serving as proxy, set up by the immobility of cities, both sides having a reasonable chance of victory, and the apparent safety of the attacker. If you get those conditions, conflict is likely to occur. Add a healthy pinch of one group or the other feeling that living under the rule of the other is intolerable, and the likelihood increases.

    Take Creationism. The Creationists can’t budge–that’s what “dogmatic” means. Scientists feel reasonably safe–we have, after all, absolutely overwhelming data collected over a few centuries. Both sides think that victory is possible. And both sides think that life after the victory of the other side would be intolerable. Ergo, conflict. In contrast, there’s not much friction between mainstream science and, say, Phantom Time advocates. The advocates of Phantom Time have no hope of victory, and can jink and juke to their hearts content. Science, meanwhile, has declared victory so sees no need to continue the conflict.

  89. Douglas Knight says:

    Possible implications for artificial sweeteners?

    Also, they really should have had an artificial sweetener arm to this study. That’s a much simpler study to run. But they could have mixed that up with the knockout mice, too. (They even mention that the knockouts don’t taste artificial sweetener.) If artificial sweeteners caused weight gain in mice, would someone have noticed? Would this paper have cited them?

  90. James Lohner says:

    “Company that handles tech company interviews makes a feature that changes what gender an interviewee’s voice sounds like, to see if women get more tech jobs when the company thinks that they’re men. To the surprise of nobody who is paying attention, there is no anti-woman bias found and in fact women do slightly better when they are known to be female.”

    For someone who wasn’t paying attention, could you please link to an overview of the relevant literature?